Womens/Development EXAM MASTER

Lakukan tugas rumah & ujian kamu dengan baik sekarang menggunakan Quizwiz!

Question A 26-year-old woman at 29 weeks gestation presents with a 1-week history of vaginal discharge and dysuria. On genital exam, she has mucopurulent cervicitis. A cervical swab is obtained and results are positive for Chlamydia trachomatis infection. What is the best course of treatment for this patient?

Correct answer: Azithromycin Explanation Azithromycin is safe for use in pregnancy (Category B) and it is effective against Chlamydia. Though doxycycline is effective against Chlamydia, it is contraindicated for use in pregnancy (Category D). Though metronidazole is safe for use in pregnancy, it is not effective for treatment of Chlamydia. Levofloxacin is effective against Chlamydia, but it is contraindicated in pregnancy (Category C). Fluconazole is Category C for use in pregnancy and is not effective against Chlamydia.

Case A 25-year-old woman presents with pelvic pain for the last 3-4 menstrual cycles, but the most recent episode with increasing symptoms for approximately 2 weeks. She has tried over-the-counter pain relievers without relief. She admits deep dyspareunia, and she has been with her current sexual partner for about 6 months. She has no chronic medical conditions. There are no allergies to any medications. She denies urinary issues. She admits to nausea, but no vomiting or diarrhea. She reports a negative home pregnancy test this morning. Her last pelvic exam was over 1 year ago. LMP was 1 week ago. Question What physical exam findings would best narrow your diagnosis?

Correct answer: Cervical motion tenderness Explanation Cervical motion tenderness along with a history suspicious for pelvic inflammatory disease (PID) is a red flag for the diagnosis. While testing for sexually transmitted infections (especially Chlamydia trachomatis and Neisseria gonorrhea) are indicated, the diagnosis is typically based on history and physical exam. The Centers for Disease Control Sexually Transmitted Treatment Guidelines recommend empiric treatment for PID if 1 or more of the 3 following minimum criteria are present on exam: cervical motion tenderness uterine tenderness adnexal tenderness A high index of suspicion is needed, and treatment should be initiated while awaiting laboratory results. The examiner should further elicit this woman's sexual partner history and potential vaginal/cervical discharge. Other conditions, such as endometritis and endometriosis, could cause cervical motion tenderness. The CDC guidelines are helpful in selecting antibiotic therapy and whether to treat as inpatient or outpatient. Most women will find that pelvic exam is uncomfortable, but tenderness with movement of the cervix is not a normal finding and indicates inflammation in the uterus. The remaining physical exam findings listed are less helpful than a finding of cervical motion tenderness. Abdominal guarding is a common physical exam finding with many gastrointestinal, urinary, and pelvic disorders. Guarding refers to voluntary or involuntary contraction of the abdominal musculature in response to pain felt during palpation. It is not a specific finding for pelvic inflammatory disease, and it would not help narrow a diagnosis as much as cervical motion tenderness. A physical exam finding of an adnexa approximately 2.5 cm in size is a normal physical exam finding. When the patient is thin enough for a good estimate of ovarian size, an examiner may notice enlargement in the case of ovarian cysts, ectopic pregnancy, tumors, and other ovarian disorders. A finding of normal ovarian size does not narrow the differential for this patient. An anteverted uterus on physical exam simply indicates the angle of the fundus points toward the ventral side of the patient. This is also a normal finding, and it does not assist in arriving at a definitive diagnosis. Discomfort with speculum insertion can be related to many factors, including examiner skill, patient anxiety, vaginal inflammation, and many gynecologic disorders. This finding is nonspecific and also does not help with diagnosis.

Case Your patient is a 45-year-old woman concerned about the accelerated aging of her skin. 5 years ago, she underwent a total hysterectomy with oophorectomy. Since then, she gained about 20 pounds. She also has diabetes mellitus type II. On examination, you find a slightly overweight female (body mass index 26) with skin that has lost its elasticity and has reduced water-holding capacity, increased pigmentation, and decreased vascularity. Question What are her symptoms signs of?

Correct answer: Decreased estrogen Explanation Lower levels of estrogen are associated with skin aging, most probably because of telomeres shortening. The effects of reduced estrogen levels cause loss of elasticity, reduced water holding capacity, thickness, increased pigmentation, decreased vascularity in the skin, and facial hair. Some of those signs are found in your patient. Your patient has no ovaria, and testosterone is primarily secreted in the ovaries (small amounts are also secreted by the adrenal glands). Too much testosterone will cause excessive oiliness and acne rather than accelerated aging of the skin. Testosterone levels in this woman are decreased after the removal of the uterus and ovaries. Symptoms related to her skin are not characteristics of decreased testosterone levels. More often in decreased testosterone there will be a loss of muscle strength and mass, accumulation of fat, especially around the abdomen, osteoporosis, vaginal dryness, etc. High levels of estrogen can be found in women who are pregnant, extremely overweight, have diabetes, or have liver disease. They are sometimes associated with acne, red flushed appearance, spider nevi, etc. Skin changes in this patient cannot be attributed to high estrogen. Insulin resistance is a condition when insulin becomes less effective at lowering blood sugars. It can lead to weight gain and estrogen hyperproduction, resulting in skin changes attributable to high estrogen levels (acne, red flushed appearance, spider nevi, etc). Skin changes in this patient cannot be attributed to the high estrogen.

Case A 36-year-old couple comes to your offfice because they have been trying to conceive for 10 months without results. The woman's past medical history is significant for an elective abortion 8 years ago and an episode of vaginal candidiasis 10 years ago. The rest of the couple's past medical history is not relevant. Physical examinations of both the man and woman are normal. Question What will be your advice?

Correct answer: Evaluate semen Explanation Your patient had 1 pregnancy; therefore, she is probably able to conceive. In approximately 35% of couples, the male factor is responsible for the infertility. The best next step would be to perform semen analysis and evaluate the motility, ejaculate volume, concentration, and the morphology of sperm. Laparoscopy is an invasive procedure that should only be considered after other non-invasive structural evaluations of infertility. In general, infertility is defined as not being able to conceive after 1 year of unprotected sex. However, women who are older than 35 years and have not conceived after a 6-month period of trying to do so should be evaluated for infertility. Therefore, 'keep trying' is not the best answer. Ovulatory factors contribute to infertility in about 20% of cases. Because she was pregnant once, it is unlikely that your patient has ovulatory problems; therefore, hormonal analysis in this case should be considered after other tests come back normal. Postcoital tests examine the interaction between the sperm and the mucus to determine their compatibility. It is a poor predictor of pregnancy success, but it may help to determine the need for intrauterine insemination when other causes of infertility are excluded.

Case A 24-year-old woman, gravida 0, presents with heavy bleeding between her periods and cycles that seem irregular and are more than 40 - 60 days apart. She has never had regular monthly periods. She is also concerned that she has acne and hair growth on her face, chest, and abdomen. She would like to become pregnant. She denies any chronic medical problems, although she states that her previous physician advised her to lose weight because she has borderline diabetes. She is 5 feet 5 inches tall and weighs 240 lbs. Her gynecologic examination is unremarkable. Question What is the most likely explanation for her abnormal uterine bleeding?

Correct answer: Polycystic ovary syndrome (PCOS) Explanation This patient has polycystic ovary syndrome. PCOS, also known as Stein-Leventhal syndrome, is an endocrine condition present in 5 - 10% of women of reproductive age. It is the most common cause of infertility in women. It is referred to as polycystic because most women with the condition have a number of small cysts in the ovaries; however, it is the characteristic constellation of signs, symptoms, and biochemical aberrations, rather than the presence of the cysts themselves, that is important in establishing the diagnosis. These include dysfunctional uterine bleeding (DUB) due to estrogen breakthrough bleeding, hyperandrogenism, insulin resistance, and often obesity. Each plays a role in the evolution of an oligo-ovulatory state. It most commonly affects women ages 15 - 20 and is characterized by high estrogen and androgen levels resulting in virilization (hair growth and acne), chronic menstrual irregularities, and infertility. Although the exact mechanism is yet to be determined, it is thought to involve a disorder of the hypothalamic-pituitary axis, in which excess luteinizing hormone (LH) and androgen production cause virilization and anovulation (with cyst formation) in the ovary. Hyperandrogenism and virilization can be seen in the presence of a testosterone-secreting tumor of the ovary or adrenal, but this patient's long history of oligomenorrhea makes this less likely. Increased exogenous estrogen (associated with obesity) is likely to contribute to this patient's anovulatory state, but there is no evidence to suggest that she has been exposed to exogenous estrogen. A similar bleeding pattern can be seen with progestin-only contraception (e.g., Depo-Provera or a progestin-only birth control pill), but such hormonally-related bleeding would be due to progesterone breakthrough rather than estrogen breakthrough. This patient desires pregnancy and has not been exposed to exogenous progestin. Thyroid dysfunction may also cause anovulation through dysregulation of a feedback loop that results in increased prolactin levels; however, this patient's long history, which includes hyperandrogenism (acne and excess hair growth) and obesity, make this diagnosis less likely. Women with PCOS are at risk for cardiovascular illness and diabetes mellitus. They are also at increased risk for endometrial hyperplasia and endometrial cancer because of long-term unopposed estrogen stimulation of the endometrium.

Case A 51-year-old woman presents after discovering a new breast mass on her left side 2 weeks ago. It has not been tender. Her last exam and mammogram were normal 2 years ago. Her past medical history is significant only for being perimenopausal. She takes no medications. Your physical exam confirms a firm, non-tender, fixed 1.5 cm mass in the left breast. Right breast exam is normal. Question What is the next most appropriate step for the management of this breast mass?

Correct answer: Schedule a bilateral diagnostic mammogram. Explanation The next most appropriate step in the management of this patient is to schedule a bilateral diagnostic mammogram because the mass has characteristics suspicious for malignant breast disease (such as being unilateral, non-tender, and fixed). Especially in patients over 40, the workup of a breast mass should include mammography. A mammogram can further characterize the breast mass and potentially detect additional smaller lesions and lymph node involvement that were not detected by physical exam. Ultrasonography may provide additional diagnostic information in the evaluation of this mass. The imaging results can guide the next best treatment approach, such as fine-needle aspiration if the mass appears to be a simple cyst, or open biopsy with node sampling if the mass appears to be cancerous. Eventually, pathology of the tissue will be needed for definitive diagnosis. At this point, a referral for lumpectomy could have injurious effects regardless of whether the breast mass is benign in nature or if it is malignant and invasive. If the mass turns out to be a simple breast cyst, the patient would have been subjected to an invasive procedure when watchful waiting or needle-drainage may have sufficed. If the mass turns out to be invasive with lymph node involvement, the lumpectomy alone would have missed the opportunity to surgically sample lymph nodes, leading to an incorrect breast cancer staging or an additional surgery to remove additional cancer. BRCA testing is a serum test (for BRCA1 and BRCA2) to detect chromosomal mutations (on chromosomes 17 and 13) that are associated with an increased risk of breast and ovarian cancer. The results of a BRCA test will not influence the need to evaluate the symptomatic patient. Only about 5-10% of breast cancer cases have a known genetic mutation. If the patient tests negative for BRCA, they still have a chance that the breast mass is malignant; conversely, a patient who tests positive for the BRCA mutation(s) may develop benign breast masses. Reassurance of the patient may be helpful, but because of the suspicious characteristics of the mass and the patient's age, delaying evaluation could be deleterious to her outcome. Further workup is warranted immediately. Checking the patient's FSH level might yield some information as to her ovarian status. The FSH rises as menopausal ovarian production of estrogen and progestin declines, but the level does not directly influence management of the breast mass.

Case A 36-year-old P2 obese woman desires another pregnancy, and she seeks your advice for anything she should do prior to becoming pregnant. Her previous pregnancies and deliveries were uncomplicated, but the most recent one was 8 years ago. Her PMH is significant for insulin resistance and mild hypertension, which is currently controlled with lisinopril. Her current method of contraception is an oral combination contraceptive. Her body mass index is 31. Question What statement is the best advice in regard to reducing her birth defect risk in a subsequent pregnancy?

Correct answer: She needs to consult with a health care provider to control her hypertension on something other than lisinopril. Explanation This patient should be advised to change her antihypertensive from lisinopril, which is an angiotensin-converting enzyme inhibitor (ACEI). Lisinopril is considered a category D drug during the 2nd and 3rd trimesters of pregnancy, showing "positive evidence for human fetal risk". The class of ACEIs are "associated with fetal and neonatal injury, including hypotension, neonatal skull hypoplasia, anuria, reversible or irreversible renal failure, and death". If the hypertension is mild and this patient becomes pregnant, the healthcare provider may utilize close monitoring or initiate treatment with preferred agents, such as alpha-methyldopa, labetalol, or nifedipine. Decreasing caloric intake, especially below 1200 kcal per day, would not be recommended for this patient. It is estimated that normal pregnancy increases the daily caloric intake by 300 kcal. She should aim for a weight gain of approximately 0.66 pounds per week, with an overall weight gain of 15 - 25 pounds during a pregnancy, which is less than the 25-to 35-pound recommended gain for a woman of normal BMI. This patient's obesity is a risk factor for birth defects, but she should be advised to focus weight loss efforts prior to a pregnancy, not during one. The patient should not be advised to start oral medications for insulin resistance with the specific intent of decreasing birth defects. Though fetal malformations are strongly associated with maternal hyperglycemia, especially in the 1st trimester, there is no such association shown with the use of oral medications for insulin resistance for reducing the risk. This patient should be screened appropriately for development of gestational diabetes if she becomes pregnant. Oral drug therapy should be discontinued because there is insufficient data to establish the safety of these products. This patient does not need to discontinue her oral contraceptive for a several months' time prior to conception. The return to fertility takes roughly 2 weeks for most patients who have been using oral contraceptives, but birth defects are not associated with recent use of oral contraceptives. If the patient desires another pregnancy at age 36 and has an uncomplicated obstetric history, she should be advised to seek genetic counseling to fully explain her risks, given her advanced maternal age status (age >35 years old). Allowing the patient to make an informed decision is preferable to advising the patient to avoid pregnancy entirely.

Case A 26-year-old woman is in the 24th week of her first pregnancy. She is in fairly good shape, and the pregnancy is progressing well. However, a fasting blood glucose done in the office shows it to be 146 mg/dl. It is repeated the next day, and the value is 142 mg/dl. An oral glucose tolerance test is ordered, which comes back as abnormal. She is diagnosed with gestational diabetes and advised to meet with her obstetrician. Question What should be the recommended regimen for this patient?

Correct answer: Diet, exercise, and insulin therapy if blood glucose remains high despite diet control Explanation Diet, exercise, and insulin therapy if blood glucose remains high despite diet control is the correct response. Gestational diabetes mellitus (GDM) should be treated early and optimally to reduce risks to the fetus. In pregnancy there is an increased resistance to insulin which causes high levels of glucose, lipids, and insulin. Placental lactogen and high levels of estrogen and progesterone cause this scenario. Risk factors for GDM include family history of diabetes, prior delivery of large baby, age >25 years, obesity, steroid use, glycosuria in first antenatal visit, ethnicity like Hispanic, African, Native American, Asian or Pacific Islander, polycystic ovarian disease (PCOS), and prior fetal loss of unknown cause. Hyperglycemia is harmful to the mother and the fetus and can cause increased perinatal mortality. Preeclampsia and polyhydramnios have a higher incidence in GDM. Trauma during birth and operative delivery are some other adverse outcomes. Macrosomia, or larger than a normal fetus, is common. Metabolic abnormalities in the baby post delivery like hypoglycemia and hyperbilirubinemia, childhood obesity, attention deficit disorder (ADHD), and early diabetes may occur in the child. Universal screening of all pregnant women is recommended (though very low risk women may be excluded), usually at 24-28 weeks of pregnancy, except if risk factors are evident (e.g., obesity, prior history of GDM etc.), in which case screening should be done as early as possible. A random blood sugar of >200 mg/dL or fasting glucose of >126 mg/dL on 2 occasions is diagnostic of diabetes and does not require screening. Screening is done with a 50 gm oral glucose challenge test. A value of >140 mg/dL is positive, and a 3-hour glucose tolerance test is then done for definitive diagnosis. GDM is confirmed if 2 or more of the following are present: fasting glucose> 95 mg/dL, 1-hour glucose >180 mg/dL, 2-hour glucose >155 mg/dL, and 3-hour glucose >140 mg/dL. Referral to a nutritionist for strict diet control is a must to maintain euglycemia, prevent ketosis, and monitor adequate weight gain. Blood glucose should be measured before breakfast and 1 hour after each meal by the patient. Moderate exercise is encouraged. Fasting blood glucose should be <90 mg/dL and post-prandial should be <120 mg/dL. If blood glucose remains high in spite of these measures, then insulin is the treatment of choice. Oral hypoglycemic agents are not approved for use in pregnancy in the USA due to transplacental passage and fetal morbidity. If fasting blood glucose is high, then NPH insulin is used at bedtime in the dose of 0.2 units/kg body weight. If post-prandial glucose is high, then regular insulin is used before meals. If both are high, then a regimen of 4 injections a day needs to be started, including NPH before breakfast and at bedtime as well as regular insulin before each meal. Oral agents -- most commonly, metformin and glyburide -- can also be used in gestational diabetes, but these outcomes are not better than that of insulin, and in some cases may be worse. In the United States, as of 2018, insulin is still recommended as first-line therapy by the American College of Obstetricians and Gynecologists, as well as by the American Diabetes Association. The use of oral antihyperglycemic agents in pregnancy is permissible, but due to a comparative lack of evidence, these are typically reserved for those patients who refuse (or cannot afford) insulin therapy.

Question A 37-year-old G3P1102 presents at 8 weeks gestation for prenatal care. Her obstetric history reveals that she has a son with a lumbosacral meningomyelocele. What statement concerning this patient is true?

Correct answer: Folic acid supplementation must be initiated before conception to reduce the risk of neural tube defects. Explanation To derive the greatest benefits, folic acid supplementation must be initiated before pregnancy. Usually, by the time a woman recognizes her pregnancy, it is well past the development of the neural tube, which is complete at approximately 4-5 weeks post-fertilization. The CDC recommends a dose of 4 mg of folic acid daily for women who previously delivered a baby with a neural tube defect. The recommended dose for the general population is 0.4 mg or 400 mcg. The risk of having a second child with a neural tube defect is 3-5%, while the risk for any chromosome abnormality is 1/125 at term.

Case A 30-year-old woman has a routine cervical cytology (pap smear); it returns as negative for intraepithelial lesion or malignancy. She has had several normal pap smears in the past, with no abnormal ones. She has no past history of sexually transmitted infections. Question According to current guidelines from the United States Preventive Services Task Force (USPSTF), when should her pap smear be repeated?

Correct answer: 3 years Explanation According to the USPSTF, cervical cancer screening should begin at 21 years regardless of age at onset of sexual activity, and women aged 21 - 65 should be screened with cytology every 3 years. Women aged 30 - 65 years who want to lengthen the screening interval may be screened with a combination of cytology and human papillomavirus (HPV) testing every 5 years. The USPSTF recommends against screening for cervical cancer in women younger than 21 years or women older than 65 years who have had adequate prior screening and with no other risk factors for cervical cancer. Women who have had a hysterectomy with removal of the cervix and with no history of high-grade precancer or cervical cancer should not be screened. A repeat pap in 6 months might be indicated if her current pap smear result was abnormal or to follow up after a colposcopy. A repeat pap smear in 1 year would be appropriate in women with risk factors, such as human immunodeficiency virus infection (HIV). 2 years is the appropriate screening interval for younger women aged 21 - 29 according to guidelines from the American College of Obstetricians and Gynecologists (ACOG). 4 years is not a recommended screening interval for any age group.

Case A 14-year-old girl presents because she has not yet had her first menstrual period; she has no other health problems. Her mother states that her daughter suffers from what seems to be mild intellectual disabilities and attends special education classes. On examination, the girl is 4'5", her neck skin shows web-like folds on the side, her breasts are not fully developed, and the nipples are widely separated. No heart murmurs are detected; the lower limb pulse is weaker than that of the upper extremities. Question What is the most likely karyotype of this girl?

Correct answer: 45,X Explanation Turner syndrome is a leading cause of gonadal (ovarian) dysgenesis. It is caused by sex chromosomal abnormalities resulting in a girl/woman having only 1 X chromosome. The typical karyotype of Turner syndrome is 45,X, but it may also present with mosaic pattern 46,XX, 45,X, and sometimes 46,XX where the second X chromosome is nonfunctional due to an abnormality. Turner syndrome presents in 1 of 2500 live female births. It affects many systems in the body and can be diagnosed prenatally by amniocentesis. It causes short stature, webbed neck, widely separated nipples, and intellectual disabilities. Gonadal dysgenesis leads to amenorrhea; if a mosaic pattern is the case, premature ovarian failure occurs. The gonads are always streak gonads with a small number of ovarian follicles. The pattern 47,XYY is also called super male; it is characterized by intellectual disabilities and large testicles in boys/men. The pattern 47,XXY is Klinefelter syndrome, resulting in a tall boy/man with small gonads and mild intellectual disabilities. The pattern 46,XY is the normal male karyotype. The pattern 47,XXX, is also called super female; it results in tall, relatively normal girls/women who have a lower IQ than their siblings.

Case A 15-year-old girl presents with short stature and primary amenorrhea; her neck is short and broad, her palate is high-arched, and her genitalia are infantile. Her parents recall several episodes of otitis media during childhood. Her serum FSH is elevated and estradiol is low. Question What would a chromosomal analysis be most likely to reveal?

Correct answer: 45,X Explanation Turner syndrome, also known as gonadal dysgenesis, is a genetic abnormality characterized mainly by karyotype 45,X. Other variants include 45,X/46,XX mosaicism and structural abnormalities of the X chromosome (i.e., rings and X-fragments). These patients are phenotypically girls/women with a variety of somatic stigmas (e.g., short stature, low hair line, widely-spaced nipples, webbed neck) and cardiac and renal malformations. Most patients have absent or incomplete pubertal development and primary amenorrhea, although a few may reach menarche spontaneously. When the diagnosis is made, a careful renal and cardiac examination should be performed to rule out malformations (e.g., horseshoe kidney, duplication of the collection system, coarctation of the aorta, bicuspid aortic valve, and mitral valve prolapse). Treatment is directed towards estrogen replacement and management of other structural abnormalities. True hermaphroditism (46,XX/46,XY) is a rare chromosomal aberration; patients present with gradations between male and female external sexual organs (ambiguous genitalia) directed by the predominant gland. Internally, patients usually present with ovarian and testicular tissue in the same gonad (ovotestis), or an ovary on one side and a testis on the other side. Diagnosis is made by the presence of ambiguous genitalia; most individuals are raised as boys. Although it is unknown if the ovarian tissue undergoes neoplastic degeneration, the risk of malignancy in the testicular tissue is increased and gonadectomy is usually recommended. Around 80% of patients with Klinefelter syndrome are found to have the 47,XXY karyotype, although rarely other aneuploid aberrations (e.g., 48,XXXY, 48,XXYY, and 46,XY/47,XXXY mosaicism) may be seen. Patients with Klinefelter syndrome may present with small, firm testes with associated low testosterone levels, high gonadotropin levels, gynecomastia, azoospermia, and increased leg length. The severity of phenotypic presentation depends on the number of X chromosomes or the presence of mosaicism. Fertility may be possible in patients with mosaicism. Signs of Klinefelter before puberty may be very mild and go unnoticed, and the majority of patients are diagnosed during evaluation for infertility. Most patients fail to reach puberty, have small, firm testes, and signs of androgen deficiency. Diagnosis is made by lymphocyte karyotyping; treatment includes testosterone replacement. In some patients, spermatozoa may be recovered by testicular extraction for intracytoplasmic sperm injection (ICSI). Most children born after ICSI will have a normal karyotype. 47,XXX is a chromosomal aberration found in roughly 1 in 1,000 newborn girls. Somatic stigmata are absent in these patients and clinical features are very nonspecific (e.g., low birth weight, decreased head circumference). It is associated with varying degrees of intellectual disability and psychosocial adaptation problems. 46,XX constitutes the normal female karyotype.

Case A young couple presents for advice about infertility; they have been married for 4 years and have never used contraceptive methods. They are both 27 years old, and the wife has an 8-year-old daughter from a previous relationship. Both are very eager to have a child; the husband is somewhat anxious with the results of 2 semen analyses that show azoospermia. He states that when he was 14 years old he was evaluated for delayed pubertal development; at that time, he had a chromosomal analysis that revealed that the problem is genetic. He was prescribed testosterone, which he used for only a few months. He is 6'8'' tall and has unusually long legs. His facial skin is smooth; there is no beard. There is palpable breast tissue bilaterally. His right 5th finger is curved towards the 4th finger. His pubic hair is substantially decreased for his age, and both testes are small (approximately 5 cm) and firm in consistency. Question What is his karyotype likely to show?

Correct answer: 47,XXY Explanation Around 80% of patients with Klinefelter syndrome are found to have the 47,XXY karyotype, although rarely other aneuploid aberrations such as 48,XXXY, 48,XXYY, and 46,XY/47,XXXY mosaicism may be seen. Patients with Klinefelter syndrome may present with small, firm testes with associated low testosterone levels, high gonadotropin levels, gynecomastia, azoospermia, and increased leg length. The severity of phenotypic presentation depends on the number of X chromosomes or the presence of mosaicism. Fertility may be possible in patients with mosaicism. Signs of Klinefelter before puberty may be very mild and go unnoticed, and the majority of patients are diagnosed during evaluation for infertility. Most patients fail to reach puberty and have small, firm testes and signs of androgen deficiency. Diagnosis is made by lymphocyte karyotyping; treatment includes testosterone replacement. In some patients, spermatozoa may be recovered by testicular extraction for intracytoplasmic sperm injection (ICSI). Most children born after ICSI will have a normal karyotype. True hermaphroditism (46,XX/46,XY) is a rare chromosomal aberration; patients present with gradations between male and female external sexual organs (ambiguous genitalia) directed by the predominant gland. Internally, patients usually present with ovarian and testicular tissue in the same gonad (ovotestis), or an ovary on one side and a testis on the other side. Diagnosis is made by the presence of ambiguous genitalia; most individuals are raised as boys. Although it is unknown if the ovarian tissue undergoes neoplastic degeneration, the risk of malignancy in the testicular tissue is increased and gonadectomy is usually recommended. Turner syndrome, also known as gonadal dysgenesis, is a genetic abnormality characterized mainly by karyotype 45,X. Other variants include 45,X/46,XX mosaicism and structural abnormalities of the X chromosome (e.g., rings, X-fragments). These patients are phenotypically girls/women with a variety of somatic stigmas (e.g., short stature, low hairline, widely spaced nipples, and webbed neck) and cardiac and renal malformations. Most patients have absent or incomplete pubertal development and primary amenorrhea, although a few may reach menarche spontaneously. When the diagnosis is made, a careful renal and cardiac examination should be performed to rule out malformations (e.g., horseshoe kidney, duplication of the collection system, coarctation of the aorta, bicuspid aortic valve, or mitral valve prolapse). Treatment is directed towards estrogen replacement and management of other structural abnormalities. 47,XXX is a chromosomal aberration found in roughly 1 in 1,000 newborn girls. Somatic stigmata are absent in these patients and clinical features are very nonspecific (e.g., low birth weight, decreased head circumference). It is associated with varying degrees of intellectual disabilities and psychosocial adaptation problems. 46,XX constitutes the normal female karyotype.

Question An 18-year-old man presents with progressive growing of his breast tissue. Physical exam reveals that he is very tall for his age, with very long arms and legs and small testes. Blood tests indicate high levels of FSH. What is the genetic abnormality in this patient?

Correct answer: 47XXY Explanation 47XXY (Klinefelter Syndrome), described by Hans Klinefelter in 1942, consists of small testes, azoospermia, gynecomastia, sparse body hair, above-average height, eunuchoid body proportions, long legs and arms, and low testosterone levels and high levels of FSH and LH. [7] [8] Trisomy 18 (Edward Syndrome) is the second most common trisomy, and presents more commonly in female patients. The syndrome is characterized by intellectual disabilities, ventricular septal defects, patent ductus. duodenal atresia, imperforated anus, diaphragmatic hernia, polydactyly, microcephaly and microphtalmia, and rocker bottom feet. Most of them die in the 1st year of life. [3] In 45X0 (Turner Syndrome) patients present with unexplained growth failure, pubertal delay, webbed neck, preductal coarctation of the aorta, bicuspid aortic valve or hypoplastic left heart, low hairline, low-set ears, edema of the hands or feet, and elevated levels of FSH. [1] [2] Deletion of the short arm of chromosome 5 (Cri du chat syndrome), first described in 1963 by Lejeune et al., translates to "cry of the cat". A high-pitched cry is heard in affected neonates. Patients have multiple nonspecific characteristics, such as low birth weight, poor muscle tone, microcephaly, language difficulties, and profound retardation. Facial characteristics are round face, hypertelorism, low-set ears, micrognathia, a prominent nasal bridge, epicanthal folds, and facial hypotonia. [4] [5] Trisomy 21 (Down syndrome) is the most common trisomy. Features include hypotonia, brachycephalic head, epicanthic folds, flat nasal bridge, upward slanting, low implantation ears, single transverse palmar crease, and variable degrees of intellectual disability. There may also be defects of the endocardial cushion, Hirschsprung disease, and duodenal atresia. [6]

Question An obese woman pregnant with twins is concerned about gaining too much weight. How do you counsel her regarding the Institute of Medicine's (IOM) recommended weight gain in women carrying twins?

Correct answer: A 25- to 42-pound weight gain is recommended. Explanation According to IOM, obese women (BMI >29.9) pregnant with twins should gain 25-42 pounds, or only 11-20 pounds if pregnant with a singleton. A woman with a normal BMI (18.5-24.9) carrying twins should gain between 37-54 pounds, or 25-35 pounds in pregnancy if carrying only one baby. Pregnant women with a low BMI (≤18.5) should attempt to gain 28-40 pounds if carrying one infant, or 31-50 pounds if carrying twins.

Question A 23-year-old primiparous woman visits her primary care provider for follow up with her pregnancy. Today her gestational age is 25 weeks. Her pregnancy has been uneventful so far. She has no personal or family history of diabetes mellitus. Her height is 5 feet 9 inches and her weight is 140 lbs. She has no chief complaints. The physician would like to screen for gestational diabetes and opts to perform a 50-g oral glucose challenge test. Regarding performance and interpretation of this test, what is true?

Correct answer: A standard nonfasting 50-g glucose challenge test with a glucose level of ≥135 mg/dL after 1 hour is abnormal and warrants further testing Explanation Gestational diabetes affects approximately 5 - 6% of pregnancies in the United States. Most obstetricians adopt a policy of universal screening of pregnant women between 24 and 28 weeks gestational age using the oral 50 g glucose challenge test. The test is performed by administering a 50-g oral glucose load without fasting before the test. The test has a low specificity with handheld monitors; therefore venous blood should be used to obtain blood samples for glucose measurement. Most clinicians use a value above 135-140 mg/dL to prompt the need for follow-up testing (usually with 3-hour 100-g oral glucose testing, given after an overnight fast).

Case A 17-year-old girl presents with a 1-month history of vaginal discharge. The discharge is described as thin, greenish-yellow, and malodorous. She also admits to vaginal irritation and postcoital bleeding. She currently has 1 sexual partner and denies the use of barrier contraceptive methods. Speculum exam reveals erythematous vaginal mucosa, purulent, thin, frothy discharge, and a 'strawberry cervix'. Question Based on the patient history and physical examination findings, what is your most likely diagnosis?

Correct answer: Acute cervicitis caused by Trichomoniasis vaginalis Explanation The correct response is acute cervicitis caused by Trichomoniasis vaginalis. This patient has the classic findings associated with an acute trichomoniasis infection, which are purulent, malodorous, thin, frothy discharge with vaginal irritation, and punctate hemorrhages on the cervix (strawberry cervix). The case does not support the other diagnoses. Chronic cervicitis is typically noninfectious, and cervicitis caused by Gardnerella vaginalis is not associated with purulent discharge or strawberry cervix; no foreign body was identified on speculum exam, which would support a diagnosis of foreign body cervicitis.

Case A 17-year-old G1, P0 girl presents after being found in a crack house by local police. She was initially cooperative, but she is now experiencing severe abdominal pain, and she has developed vaginal bleeding. She says that she is pregnant, but she has not received any prenatal care. Examination reveals a blood pressure of 90/50 mm Hg, pulse of 120/min, and respiratory rate of 25/min. She is diaphoretic and clammy. Her uterus measures 25 cm from the pubic symphysis, but no fetal heart tones can be appreciated. There is a large amount of dark blood around the vagina. Ultrasound reveals an intrauterine fetal demise and a hyperechoic retroplacental hematoma. Labs are pending. Question What is the next best step in the management of this patient?

Correct answer: Administration of crystalloids and obtain type and crossmatch for packed RBCs Explanation This patient probably has a grade 3 placental abruption, characterized by external uterine bleeding and fetal demise, as well as maternal hemodynamic instability. Patients with placental abruption should have adequate intravenous access established. Crystalloid fluids are initially used to resuscitate the patient. Blood for transfusion should be typed and crossed (usually at least 4 units). Coagulation studies should be drawn to assess for signs of coagulopathy. Fibrinogen is usually given if the fibrinogen level is <100 mg/dL. Platelets may be administered if the platelet count is <50,000 /uL. In the case of fetal demise, vaginal delivery is generally preferred over cesarean section. Placental abruption (abruptio placenta) is defined as the premature separation from the uterus of a normally implanted placenta. As the placenta separates, there is a large amount of bleeding which irritates the uterus and causes uterine contractions, as well as fetal distress because fetal perfusion is compromised. The incidence of placental abruption ranges from 1 in 75 to 1 in 225 births. Placental abruption is classified by degree of separation. Grade 1 abruption presents with slight vaginal bleeding, mild uterine irritability, and a normal fetal heart rate tracing. Maternal blood pressure is normal, as is the maternal serum fibrinogen level. A grade 2 abruption is characterized by mild-to-moderate vaginal bleeding and pronounced uterine irritability, which may include tetanic uterine contractions. The patient may exhibit orthostatic blood pressure changes and often has an elevated pulse. The fetal heart rate usually shows some evidence of distress. Maternal fibrinogen levels are usually reduced to 150 to 250 mg/dL. A patient with a grade 3 abruption will have moderate-to-severe vaginal bleeding (unless concealed in the uterus), painful, tetanic uterine contractions, and hypotension. The fetus will be dead, and the patient is likely to exhibit a coagulopathy with fibrinogen levels less than 150 mg/dL, thrombocytopenia, and reduced levels of clotting factors. Abruptio placenta is associated with several risk factors. Maternal hypertension is one of the most commonly associated risk factors identified in patients who develop placental abruption, especially those with grade 3 abruption (where 40-50% of patients will have hypertensive disease of pregnancy). A history of prior abruption, tobacco abuse, cocaine abuse, poor nutrition, and chorioamnionitis are also associated with an increased risk for abruption. Uterine trauma is also associated with a risk for placental abruption, causing 1-2% of grade 3 abruptions. Patients with trauma in pregnancy may present with minimal physical evidence of trauma, but they will still have a significant abruption that can progress from grade 1 to grade 3 within 24 hours. Multiple gestations and polyhydramnios can also cause placental abruption if the uterus decompresses rapidly during labor. Placental abruption will classically present with painful vaginal bleeding in the third trimester of pregnancy (80% of patients present with bleeding), though the amount of bleeding may be concealed within the uterus (20%). Ultrasound is used to rule out placenta previa (the other common and dangerous cause of third trimester bleeding) and may identify placental abruption. Unfortunately, the ultrasound appearance of an abruption may lag behind the clinical degree of bleeding, so this imaging modality cannot rule out abruption. The prognosis and management of abruptio placenta depend greatly on the gestational age of the fetus, as well as the grade of the abruption. A grade 1 abruption with a term fetus can often be managed with a controlled induced delivery; since blood is a strong uterine irritant, many of these patients will deliver relatively quickly. The management of preterm pregnancies, however, is less clear and depends on the degree of abruption and fetal distress, with the risks of premature delivery measured against the risks of progression of the abruption.

Case A 24-year-old woman at 32 weeks of gestation presents with a fever, chills, generalized malaise, and vomiting. T=100.9°F, BP=110/70 mm Hg, P=100/bpm, RR 20/min On pelvic examination, she has acute fundal tenderness, the cervix is 2 to 3 cm dilated, she is 40% effaced, and vertex is at -1 station. Contractions are palpated and recorded every 5 to 10 minutes. Urinalysis shows no evidence of bacteria. On vaginal examination, membranes are ruptured. Question In addition to the administration of steroids, what is the most appropriate next step?

Correct answer: Administration of intravenous antibiotics and induction of labor Explanation This patient has preterm premature rupture of membranes (PPROM) and she is in active labor. PPROM is defined as rupture of fetal membranes prior to 37 weeks' gestation. Given her high temperature and fundal tenderness, her clinical presentation is most consistent with chorioamnionitis. In the setting of ruptured membranes, the diagnosis is more likely. Management: Immediate antibiotics for chorioamnionitis therapy should be started, and the commencement of labor induction or augmentation should be considered once the patient is stabilized. Chorioamnionitis, labor, or non-reassuring fetal heart rate testing mandates delivery at any gestational age. In the absence of labor, chorioamnionitis, or non-reassuring fetal heart rate testing, patients with PPROM can be expectantly managed until 34 to 35 weeks' gestation with corticosteroids and broad-spectrum antibiotics: Corticosteroids: A complete course is indicated from 24 to 34 weeks' gestation. Because the degree of lung development is critical for premature neonates, betamethasone or dexamethasone are given to stimulate fetal lung development before delivery. This reduces the risk of respiratory distress syndrome (RDS) and intracranial hemorrhage (ICH). A recent review of several studies found significant benefits and no increase in risk of infection for mothers or infants given steroids. Current recommendations are that all patients with PPROM before 32 weeks receive steroids. Broad-spectrum antibiotics: A 7-day course is given. Antibiotics prolong the latency period and improve perinatal outcomes in patients with PPROM. Fetal well-being is assessed daily with a non-stress test, and a follow-up biophysical profile as needed is used for ongoing monitoring following pharmacologic intervention. Fetal fibronectin is a glycoprotein that plays a role in fetal membrane adhesion. It can be detected in the cervicovaginal fluid in the late 2nd and early 3rd trimester, and it has been associated with preterm birth. Fetal fibronectin testing may be useful in women with symptoms and negative tests because the negative predictive value is greater than 95%. This may avoid unnecessary treatment. However, the poor positive predictive value creates clinical ambiguity in patients who test positive. It is a diagnostic tool and would not be useful in this case. Tocolysis with magnesium sulfate is only useful for the immediate 48 hours following membrane rupture while achieving steroid therapy. Treatment beyond this period has not found to be effective; in fact, it may be harmful. Tocolysis in this patient is contraindicated regardless of the number of hours following the membrane rupture; the patient's chorioamnionitis warrants fetal delivery.

Case A 28-year-old woman presents for a well woman examination. She complains about a foul smelling vaginal discharge. She thinks it may be greenish in color. She does have a history of sexually transmitted disease, including chlamydia. She has been treated for it twice. A Pap smear a year ago showed evidence of HPV infection, for which she had colposcopy and cryotherapy. Her last menstrual period was 10 days ago, and it was normal. Past medical history is otherwise significant only for obesity and intermittent low back pain. She takes ibuprofen on a regular basis. She is not allergic to any medications. Family history is significant for breast cancer (grandmother and aunt), diabetes, hypertension, and stroke. Social history reveals that she smokes 1 pack per day and has done so for the past 10 years. She admits to alcohol use and experimentation with marijuana. She is unemployed, but she is attending night school. She lives with her mother and 3 children. On review of systems, she does not have any abdominal pain, dysuria, hematuria, diarrhea, constipation, or fever. She has not noticed any concerning lumps during breast self-examination. She has used oral contraceptives in the past, but wishes to try a different method of birth control because she frequently forgets to take the pill. On physical examination, blood pressure is 140/90 mm Hg, and weight is 245 lb. Pulse is 70 and regular, and respirations are 12. HEENT is normal. Heart and lung examinations are normal. Abdominal examination is difficult due to her body habitus, no abdominal tenderness, and normoactive bowel sounds. Breast examination is normal. External genitalia are normal. On speculum examination, the cervix is parous, and there are no suspicious lesions. A Pap smear and cultures are obtained. Bimanual examination shows there to be no cervical motion tenderness; there is no adnexal mass or tenderness. Question What Pap smear result is potentially the most serious and requires more aggressive investigation and treatment?

Correct answer: Atypical glandular cells, not otherwise specified Explanation Atypical glandular cells, not otherwise specified (AGC-NOS) indicate a much higher likelihood of more serious underlying disease than atypical squamous cells of undetermined significance (ASCUS). AGC-NOS is found on less than 1% of all Pap smears, but on further study, a significant percentage (20 - 50%) of these patients will be found to have high-grade pre-invasive disease, carcinoma in situ, or invasive adenocarcinoma. Colposcopy is recommended for these patients, and depending on the findings, conization biopsy should be strongly considered. ASCUS Paps can simply be repeated once any underlying infection is treated, and does not immediately require colposcopy or more invasive investigation. A lack of endocervical cells is indicative of an inadequate specimen. The Pap can simply be repeated at an interval best determined by underlying risk factors, but there is no need for an immediate colposcopy. Gardnerella and Candida are frequently part of the normal vaginal flora, and they do not need to be treated unless symptomatic. Trichomonas is a sexually transmitted disease and is readily treated with metronidazole. While it is important to treat this infection, the more serious finding would be abnormal cytology. Having this infection does not increase risk of future cervical dysplasia; however, certain strains (16 and 18) of human papillomavirus do increase risk.

Case A 25-year-old woman has come in to see you today for her annual gynecological visit. You review her history and note the following: menses onset at the age of 12 years old, duration of menses is typically around 6 days in length and occurs every 30 days. She is G0P0 and has no history of abnormal pap smears or diagnosed STIs. The patient is a non-smoker, single, and is in a monogamous relationship with 1 partner for the past year. A pertinent positive the patient mentions is a whitish gray vaginal discharge that increases after intercourse and is accompanied by a distinct musty odor; she denies any pain from this discharge. Physical examination and a positive result of a whiff test solidify your suspicions. Question The patient most likely has what diagnosis?

Correct answer: Bacterial vaginosis Explanation The patient above is highly likely to be experiencing cervicitis caused by the disease state known as bacterial vaginosis. Many times this is considered a polymicrobial disease state and frequently is not viewed as a sexually transmitted disease. Its main etiology is an abundant overgrowth of Gardnerella and other anaerobes. These organisms produce a malodorous discharge that has a characteristic grayish to frothy appearance to it. An amine-like ("fishy") odor is created if the discharge is alkalinized with 10% potassium hydroxide (positive "whiff test.") On wet mount, the characteristic "clue cells" will also help the healthcare provider confirm the diagnosis. A patient with a Candidainfection will have a malodorous, white, curd-like discharge. It will also exhibit extreme pruritus and vulvovaginal erythema. A microscopic examination with potassium hydroxide will reveal both hyphae and spores. This is not consistent with the patient scenario above. Trichomoniasis cervicitis is known as "strawberry cervix" when examined by the healthcare provider. This type of pathology will have diffuse vaginal erythema accompanying red macular lesions as well as possible punctate hemorrhages that are identified on the cervix. It produces a malodorous frothy, yellow-green discharge as well as pruritus. Microscopic examination will reveal motile protozoan organisms with flagella. This does not fit the case discussed above. A patient who has a Gonorrhea infection will typically have signs and symptoms consistent with pelvic inflammatory disorder: this includes lower abdominal pain, chills, fever, menses disturbances, purulent cervical discharge, and exquisite cervical and adnexal tenderness to palpation. This pathology is more common in nulliparous, young, sexually active women who have a history of multiple partners. This is inconsistent with our patient. Herpes simplex type 2 (HSV-2) will have distinct lesions involving the genital and even the anogenital area. Patients will experience burning, stinging, and potentially even have a neuralgia type of pain. The actual lesions are usually small, grouped vesicles and could potentially look eroded. These lesions will be painful to the patient. Regional lymphadenopathy may also be present. This description is not comparable to the patient scenario.

Case A 32-year-old woman presents with a 2-day history of having a vaginal "bump"; the bump is painful to sit on. She has never had this problem before, and she has been monogamous with the same sex partner for 7 years. On physical exam, you notice a solitary 2 cm smooth, slightly tender mass at the introitus. A KOH/wet mount demonstrates squamous cells with no white blood cells (WBCs), hyphae, or motile organisms. Whiff test is negative. Refer to the image. Question What is the most likely diagnosis?

Correct answer: Bartholin gland cyst Explanation This patient most likely has a Bartholin gland cyst. The Bartholin glands are located at approximately the 4 o'clock and 8 o'clock positions at the introitus. Secretions help with vaginal lubrication. A cyst may form when the gland outlet is obstructed; infection occasionally develops, leading to an abscess. Both Bartholin cysts and abscesses can be painful. Initial conservative treatment consists of warm compresses to encourage drainage. If this is unsuccessful, incision and drainage are necessary. If Neisseria gonorrhea is implicated as a causative organism, appropriate antibiotic treatment should be prescribed. Vaginal candidiasis (also known as a yeast infection) would be characterized by symptoms of vaginal pruritus and discharge. Vulvar pain is common, and patients may note pain upon sitting, but a physical exam would reveal vaginal and vulvar erythema, thick white discharge, and hyphae on the wet mount. The fungal hyphae are especially noticeable when potassium hydroxide (KOH) is applied to the microscope slide to lyse the other cells. Vaginal candidiasis is not associated with a solitary vulvar mass. Herpes simplex, type 2 is typically the strain associated with genital herpes. A symptomatic patient with herpes may describe vulvar lesions causing pain with sitting. When lesions are visible, herpes is characterized by multiple painful vesicles and/or ulcerated lesions. Herpes lesions are typically much smaller than the described single mass. Lichen sclerosis is a chronic, progressive dermatological problem, often involving the vulvar and perineal epithelium. It is characterized by pruritus, dyspareunia, and fissuring of the skin. Lichen sclerosis is more common in post-menopausal women, and physical exam reveals thin, white-appearing tissue. Masses are not associated with lichen sclerosis. Vaginal intraepithelial neoplasia (VAIN) is typically located at the upper third of the vagina. Its presence is usually detected by an abnormality on the Pap pathology from asymptomatic patients. This patient's 2 cm mass at the introitus could potentially be a malignant tumor, but VAIN is an unlikely diagnosis. Bartholin gland carcinoma is a possibility, but it is very rare.

Case A 20-year-old woman presents to the medical office to start birth control pills. She is engaged to be married soon. She has never been sexually active. She states she feels healthy, and she denies any current complaints. A summary of her past medical history includes: Medications: None Allergies: Sulfa medications Surgical history: None Medical history: No known conditions. OB/GYN history: Menarche age 13. Regular monthly menses, with no menstrual complaints. Family history: Paternal grandfather has diabetes and hypertension. Maternal grandmother had a stroke. Maternal grandfather had prostate cancer. Father has hypertension and history of a heart attack. Social history: Patient works as a retail clerk part-time and is attending college. She currently lives with her parents. She denies the use of tobacco, alcohol, and recreational drugs. Question What component of the physical exam is the most important before prescribing birth control to this patient?

Correct answer: Blood pressure measurement Explanation It is most useful and important to obtain a blood pressure measurement prior to initiation of combined hormonal contraceptives. Routine screening, including physical exam and tests, is generally not recommended prior to initiating contraceptives. If the patient is found to have severe hypertension, combination hormonal contraceptives should be avoided. Likewise, if blood pressure measurements rise dramatically after using these contraceptives, they should be discontinued and alternate methods initiated. A bimanual pelvic exam has been somewhat controversial, with no compelling evidence to continue the practice in asymptomatic women. In fact, exams viewed as invasive or embarrassing (such as pelvic and breast exams) can be seen as a barrier for preventing unintended pregnancies; some women may avoid encounters to obtain contraceptives in order to avoid the exams. Breast exam is another routinely performed exam; it has limitations in the asymptomatic patient population for screening. It becomes much more useful in screening for breast cancer in older women, who are higher risk for breast cancer. A breast exam is not routinely recommended prior to initiating birth control. A cardiovascular exam is not recommended prior to initiating birth control. Even with some cardiovascular disease in this patient's family history, the cardiovascular exam is likely be normal and would not be useful in initiating contraceptives. The Pap smear (short for Papanicolaou) with vaginal speculum exam is primarily a test for cervical cancer and pre-cancerous dysplasia. Many years ago, it was thought that birth control increased risk of cervical cancer. That myth has been disproved. There is no compelling reason to obtain a Pap smear in order to initiate birth control. Routine cervical cancer screening guidelines should be followed, which would indicate that this patient should wait until the age of 21 years before beginning Pap testing. If the patient noted vaginal complaints or desired testing for sexually transmitted diseases, a speculum exam would be useful.

Question You note an irregular hard mass with ill-defined margins on the right breast of a 37-year-old woman. In addition, the nipple has bloody discharge. What is the most likely diagnosis?

Correct answer: Breast cancer Explanation Breast cancer is manifested as a single irregular, stellate, fixed, and firm nodule, as presented in the scenario. A mastitis is warm and erythematous along the ducts, and it is usually associated with breastfeeding. Paget's disease presents with a nodule at the epidermis of the nipple/areola, with an eczema rash, scaling, and excoriation. Fibroadenoma is a single non-tender, freely movable, round, and lobular nodule; it typically has clear margins.

Question What component of a patient's health history would be considered protective against breast cancer?

Correct answer: Breastfed all 3 of her children Explanation Breastfeeding is protective against breast cancer. Early menarche and excessive alcohol use are factors that increase susceptibility to breast cancer. Mother's age of menopause does not affect breast cancer risk.

Question A 22-year-old woman presents with increased vaginal discharge. She is sexually active with 2 male partners, and she uses birth control pill for contraception. Her last menstrual period was 12 days ago, and she has noticed an increased whitish vaginal discharge for the past week. Physical exam reveals a soft, non-tender abdomen. On pelvic exam, she has a light-yellow cervical discharge with erythema of the cervical os. There is no cervical motion tenderness and no adnexal masses or tenderness. Wet mount of the vaginal discharge reveals epithelial cells and WBCs, and no yeast or protozoa. Gram stain of the vaginal discharge reveals many leukocytes. A urine pregnancy test is negative. What is the best treatment for this patient?

Correct answer: By mouth, azithromycin 1 g PO, and ceftriaxone 250 mg IM once Explanation This patient has a mucopurulent cervicitis on physical exam, which is usually caused by Neisseria gonorrhoeae or Chlamydia trachomatis. In all practicality, a patient should be treated with antimicrobial agents that will cover both pathogens because there is a high risk of co-infection with both pathogens (at least 50%). Patients should be tested for infection with both chlamydia and gonorrhea before treatment, usually with an immunoassay. In addition, a Gram stain of the cervical discharge can help elucidate the responsible pathogen. In 60% of women with gonorrhea, Gram-negative intracellular diplococci may be identified. In chlamydial infections, the Gram stain will reveal many leukocytes (>10/high powered field) but no gonococci. These patients should also be screened for co-infection with syphilis and possibly HIV. This patient does not have signs of salpingitis or tubo-ovarian abscess (pelvic inflammatory disease). Specifically, she has no cervical motion tenderness or peritoneal signs, and no adnexal tenderness or masses. She will qualify for outpatient treatment of cervicitis and can avoid the more aggressive treatment used for salpingitis. In either condition, both of the patient's sexual partners will require medical evaluation and treatment. Additionally, if either Neisseria gonorrhoeae or Chlamydia trachomatis is identified with culture or immunoassays, infection with these pathogens must be reported to the local health department. Acceptable regimens for the treatment of gonorrheal cervicitis include one-time treatment with ceftriaxone 250 mg IM or cefixime 400 mg PO. Alternative regimens for gonorrheal cervicitis include ceftizoxime 500 mg IM; cefoxitin 2 g IM with oral probenecid 1 g; or cefotaxime 500 mg IM. Spectinomycin 2 mg IM is the recommended treatment for patients with penicillin or cephalosporin allergy; however, spectinomycin is not currently available in the United States. Previously, quinolones were an option for gonorrheal cervicitis, but these regimens are no longer recommended by the CDC due to increasing resistance to the quinolones in Asia and many metropolitan areas of the U.S. Additionally, all patients with cervicitis should receive a regimen to treat chlamydia. These regimens include azithromycin 1 g PO once or doxycycline 100 mg PO twice a day for 7 days. Metronidazole 2 g PO is the regimen used to treat vaginitis due to Trichomonas, not mucopurulent cervicitis.

Case A 26-year-old woman presents with 8 weeks of gastric reflux that does not improve with medication, as well as bloating, constipation, vaginal bleeding, and weight loss. Past medical history is significant for father with hypertension and coronary artery disease, mother with breast cancer, aunt with hypothyroidism, and sister with breast cancer. Complete physical examination is significant for an adnexal mass. CBC is significant for hemoglobin 11.5 g/dL and hematocrit of 38%. Urinalysis is negative. Question What would be the most appropriate tumor marker to order in this patient?

Correct answer: CA125 Explanation Cancer antigen 125 (CA125) is a tumor marker that can be used in the diagnosis of ovarian cancer. One test alone is not generally helpful because many things can cause an elevation in CA125, such as endometriosis, menstruation, pelvic inflammation, liver or kidney disease, and many other types of cancers. Serial measurements are advised. In this case, the physical exam findings, family history of breast cancer, and symptoms of bloating, dyspepsia, constipation, and weight loss make the possibility of ovarian malignancy very high. While not all types of ovarian cancer will have elevated CA125, epithelial ovarian cancer, which is the most common type of ovarian cancers, will show elevated CA125 in most cases. According to some guidelines, women at high risk should discuss having CA125 and ultrasounds done yearly. Cancer antigen 19-9 (CA19-9) is a marker used to detect pancreatic cancer. It can also be elevated in disease states such as biliary obstruction, cholangitis, liver cirrhosis, inflammatory bowel disease, and cystic fibrosis. Carcinoembryonic antigen (CEA) is most commonly associated with colon cancers, but it may be positive for vulvar tumors that originate in sweat glands. It is also positive in 35% of endometrial cancers and may be positive for ovarian cancer; however, CA125 is more sensitive and specific for ovarian cancer than CEA. Prostate-specific antigen (PSA) may be positive in prostate tumors and is used to screen for this cancer in men. Human chorionic gonadotropin (hCG) is normally produced by the placenta. Elevated hCG levels are most commonly associated with pregnancy, but they can also be caused by choriocarcinoma of the uterus, embryonal carcinomas, and other germ cell cancers.

Case A 36-year-old woman presents with vaginal discharge. She has a history of itching and white discharge. She is sexually active with her husband only. She takes oral contraceptive pills for contraception, and she has never been diagnosed with an STD (sexually transmitted disease). On examination, the vagina is hyperemic and covered with white cottage cheese-appearing discharge. Question What is the most likely diagnosis?

Correct answer: Candida vaginitis Explanation Candida vaginitis is the colonization of vagina and vulva by the fungus Candida albicans. It is favored by the acidic media and usually flourishes in patients on broad-spectrum antibiotics, patients who are diabetic, patients taking corticosteroid medications, those who are pregnant, and those who are immunocompromised. Another factor is increased wetness of the area, especially in combination with certain occlusive clothing. It usually presents with severe pruritus, erythema, and a curd-like discharge. Microscopic examination with KOH preparation directly or after culture on Nickerson's medium can be done. Under microscope, either from the direct swab or the culture, there is an appearance of a characteristic shape (hyphae and spores). Treatment with miconazole is usually effective. Some physicians add cortisone cream to relieve itching. Herpes is caused by infection with the herpes simplex virus. Patients present with severe pain and vesicles. Treatment is with acyclovir. Bacterial vaginosis, formerly known as non-specific vaginitis, is caused by a mixture of microorganisms, of which Haemophilus or Gardnerella are the most common and well known. It causes malodorous (fishy) discharge and is treated with metronidazole or clindamycin. Wet mount preparation with saline shows clue cells, which are epithelial cells covered with bacteria. Culture is not effective and not used. Gonorrhea is a sexually transmitted disease caused by Gonococci (gram-negative diplococci). It is characterized by discharge, fever, and sometimes joint pain and conjunctivitis. Treatment of choice is either ceftriaxone 125 mg IM or cefpodoxime 400 mg orally as a single dose. Trichomonas is caused by Trichomonas vaginalis, a parasite with flagella that causes copious vaginal discharge. Treatment with metronidazole is recommended. It can be seen under microscope (using wet mount preparation) as a motile parasite with flagella.

Case While doing routine newborn exams in the nursery on full-term infants, you note a 4-5 cm soft, raised, round, nontender, bruised swelling overriding the left occipitoparietal suture on a 30-hour-old male infant. He was born by spontaneous vaginal delivery with rupture of membranes at home. He has been feeding well. On the rest of his exam, his red reflex is present bilaterally, lungs, heart and abdominal exams seem normal. Rest of his skin is pink, warm and dry, without cyanosis. Question What is the most likely diagnosis?

Correct answer: Caput succedaneum Explanation Caput succedaneum represents an accumulation of serosanguineous subcutaneous fluid. This fluid is thus not bound by suture lines, but it may have poorly defined margins and may extend across midline and suture lines. It is caused by mechanical trauma of the presenting portion of the scalp pushing through a narrowed cervix. It is more likely to be seen after a prolonged or difficult delivery, especially after prolonged rupture of membranes. This fluid collection will resorb within a few days, so no treatment is necessary. Jaundice might result with very large caputs. Cephalohematoma is due to subperiosteal hemorrhage involving the outer table of a cranial bone. Unlike the caput, the swelling does not extend across a suture line. This may result from an instrumented delivery causing rupture of blood vessels between the skull and periosteum. Healing by resorption can take several weeks to months with some peripheral calcification. Large cephalohematomas can contribute to hyperbilirubinemia. Hydrocephalus refers to the presence of excess cerebrospinal fluid (CSF) within the ventricles and is caused by either excessive production of CSF or blockage of CSF circulation. Clinically, this presents with an increasing head circumference, full fontanelles, irritability, poor feeding, and vomiting. There may be sunset eye sign with the eyes deviated downward revealing the upper sclera. Plagiocephaly is an asymmetry of the cranial vault caused by constant pressure from positioning on the dependent side. It disappears as the newborn becomes more active and spends less time in one position. Craniosynostosis is the premature closure of one or more sutures altering the shape of the head. This can be differentiated into primary craniosynostosis, which has no underlying brain abnormality or metabolic defect, but may be related to intrauterine constraints and secondary craniosynostosis that may be due to a structural cerebral abnormality or related to other conditions such as hyperthyroidism, severe anemia, or a metabolic condition. Some genetic abnormalities can also cause craniosynostosis.

Case A 24-year-old woman presents with lower abdominal pain, nausea, and vomiting that has persisted for about 12 hours. She rates the pain at a 4 last night, which is when it initially began, but she currently puts it at a 9 on a numerical pain scale; she states the pain seems to be worsening with each passing hour. She states she had her menses 1 week prior and noted an irregular flow as well as excessive vaginal discharge since it ceased. She is single. She admits to not being monogamous, and she only occasionally uses barrier contraception during sexual encounters. She takes a daily oral contraceptive pill. Question Considering the most likely diagnosis at this time, what treatment regimen is most appropriate?

Correct answer: Ceftriaxone 250 mg IM and doxycycline 100 mg BID for 14 days Explanation The correct response is ceftriaxone 250 mg IM and doxycycline 100 mg BID for 14 days. It is highly likely that the patient described above has an active case of salpingitis/endometritis, which is commonly referred to as pelvic inflammatory disease (PID). PID is a several gynecological infection that is often polymicrobial; Neisseria gonorrhea and Chlamydia trachomatis are the most common sexually transmitted organisms that lead to PID. PID is frequently found in young, nulliparous, sexually active women with multiple partners. It is also considered the leading cause of infertility and ectopic pregnancy in this patient population group. PID presents with lower abdominal pain, chills, fever, menstrual disturbances, and purulent cervical discharge. Physical exam will reveal exquisite uterine, adnexal or cervical motion tenderness, as well as cervical discharge. Based on the information provided on the patient above, PID is the most likely diagnosis at this time. Mild to moderate cases of PID may be treated on an outpatient basis. Recommended regimens include: single dose of cefoxitin 2 gm IM plus probenecid 1 gm PO along with doxycycline 100 mg PO BID for 14 days. Another regimen considered may be ceftriaxone 250 mg IM and doxycycline, 100 mg BID for 14 days. Metronidazole 500 mg PO BID for 14 days may be added to either one of these regimens to help alleviate bacterial vaginosis, which commonly may be associated with PID cases.

Case A 30-year-old African-American primigravida at 37 weeks of gestation is hospitalized because of uterine contractions and mild vaginal bleeding that started several hours ago. Her pregnancy was uneventful; she had been feeling good, and for that reason, she refused prenatal care. Her temperature is 37 C; blood pressure is 85/55. Heart rate is 100, and respirations are 22/min. On examination, her uterus is firm, and there is a trace of blood in the introitus. Ultrasound shows abruptio placentae; there are no signs of placenta previa, and a viable fetus consistent with the gestational age. Her cervix is 1 cm dilated. Question What is your next step?

Correct answer: Cesarean delivery Explanation In an unstable patient with placental abruption and no signs that labor had started, a cesarean section is indicated to remove the placenta and prevent further complications for mother (e.g., shock, DIC) and fetal demise. Tocolysis in this unstable patient will put both mother and the fetus in a danger because placental abruption will probably continue. Vaginal delivery can be considered when delivery is in an advanced stage. In a case involving a cervix that is only dilated 1 cm, delivery has not started yet. Vasopressors are used if volume resuscitation did not restore blood pressure. Even then dobutamine is not the best choice: The American Heart Association recommendations based on blood pressure determinations are: dobutamine for systolic BP over 100 mmHg, dopamine for systolic BP 70 to 100 mmHg, and norepinephrine for systolic BP under 70 mmHg. Oxytocin is indicated to produce uterine contractions during the 3rd stage of labor and to control postpartum bleeding or hemorrhage. In this patient, labor did not start. Furthermore, oxytocin is contraindicated in obstetrical emergencies when surgical intervention is warranted and when vaginal delivery is not considered.

Question Your patient, a 48-year-old woman, presents with vaginal bleeding and states that she is "alarmed" because she is quite sure she is 2 months pregnant. History includes unremarkable live birth of a male child 7 years ago and a molar pregnancy a year ago. Examination reveals a uterus that is inappropriately large for gestational length and hCG levels are higher than expected. Fetal parts and heart sounds are not present. Your diagnosis is that of carcinoma, but you are able to reassure your patient that this neoplasm is of the type that is most sensitive to chemotherapy. What is your diagnosis?

Correct answer: Choriocarcinoma Explanation Gestational Trophoblastic Diseases are a group of related diseases forming a spectrum from Benign Hydatidiform Mole, to Invasive Mole, to Placental-Site Trophoblastoma, and finally to Choriocarcinoma. Treatment of women who have nonmetastatic gestational trophoblastic disease is almost 100% successful, and allows reproductive function to be preserved. The cure rate for metastatic disease is approximately 90 percent. Serous cystadenocarcinoma occurs in the ovary and is a cystic or semi-cystic neoplasm. It usually occurs bilaterally An ovarian dysgerminoma is a malignant ovarian neoplasm, hypothesized to be derived from primordial germ cells of the sexually undifferentiated embryonic gonad. Endometrioid carcinoma resembles the typical carcinoma of the endometrium, as its name suggests, but is an ovarian carcinoma. This neoplasm may be seen with endometrial carcinoma. Ovarian teratoma is composed of tissues that are derived from three germinal layers, the endoderm, mesoderm, and ectoderm. Many teratomas will contain hair or teeth. These true neoplasms will usually present with lower quadrant pain.

Question Ico-delete Highlights A hydatidiform mole is removed from the uterus of a 20-year-old woman. Subsequent to evacuation, her serum human chorionic gonadotropin (hCG) concentrations are monitored, which continue to rise. What is the most likely diagnosis?

Correct answer: Choriocarcinoma Explanation Hydatidiform mole is a form of gestational trophoblastic disease. It is a benign proliferation of trophoblastic tissue with cystic (hydropic) swelling of chorionic villi. It commonly occurs in women younger than 20 or over 45. Once a hydatidiform mole has been diagnosed, it must be removed. This can be done by either evacuating the uterus (an option preferred in younger women) or by hysterectomy. Since most trophoblastic cells secrete hCG, its levels are indicative of tumor mass, a persistently high or increasing level of hCG after removal of the mole indicates development of an invasive mole or choriocarcinoma. Hydatidiform moles are essentially due to a failed pregnancy; a second, ectopic pregnancy is extremely unlikely. Since only trophoblastic tissue produces hCG, none of the other mentioned conditions would cause an increase in its level.

Question The placenta has 2 portions that are identified with either the fetus or the mother. Each portion has its origin from different cells or tissues. Which of the following is the fetal portion of the placenta?

Correct answer: Chorion Explanation The fetal portion of the placenta is derived from the chorion which contains the chorionic plate and chorionic villi. The maternal portion is formed by the decidua basalis. The yolk sac develops in the first trimester to aid in the transfer of nutrients. It also has a role in blood development, is the primitive gut, and the walls contain the primordial germ cells. Usually the yolk sac detaches by the 6th week, but can persist in the adult as Meckel's diverticulum. The decidua is the endometrium found in the pregnant uterus and is shed at partition. There are several forms of decidua that occur during pregnancy and a decidual cast that can occur independent of pregnancy. The amnion can be found surrounding the fetus and amniotic fluid. It is derived from trophoblast and can be seen in the first trimester on a sonogram. Allantois is found in most mammals and serves a different function in each species. In humans its blood vessels give rise to those found in the umbilical cord. In the term fetus the allantois becomes the median umbilical ligament.

Case A 30-year-old G1P1 Caucasian woman presents with a desire to become pregnant. She has been having unprotected intercourse for 12 months without pregnancy. She typically has 3 - 4 menstrual cycles a year. She reports that she had similar problems becoming pregnant with her 1st child. Her prior doctor did a complete workup for her infertility and amenorrhea. Records have been sent for your review. She successfully conceived on a medication 3 years ago, but she cannot recall the name of the medications. She has never used any method of contraception. This patient would like you to treat her infertility. Question Ico-delete Highlights What is the drug of choice for this patient?

Correct answer: Clomiphene Explanation The drug of choice for this patient is clomiphene (Clomid and others); it has estrogen agonistic and antagonistic effects, which increase gonadotropin release and follicular maturation. This medication also improves the luteinizing hormone release and estradiol secretion. Spironolactone is the medication that inhibits aldosterone and dihydrotestosterone, functioning as an androgen receptor blocker. Its primary use in gynecology is for acne, hirsutism, and polycystic ovarian syndrome. Leuprolide is a medication that causes inhibition of gonadotropin release, which reduces ovarian steroidogenesis. It is used in women for the treatment of uterine fibroids and endometriosis; it treats prostate cancer in men. It produces the opposite desired effect when given to a patient with infertility. Mifepristone is a medication with progesterone and glucocorticoid antagonistic effects, which reduce endometrial development. It is used for early pregnancy termination; off-label, it is used for endometriosis, uterine fibroids, and postcoital contraception. This medication should not be given to this patient. Danazol is an older medication that suppresses follicle stimulating hormone (FSH) and luteinizing hormone (LH). It reduces estrogen production and is weakly androgenic. This medication would worsen the patient's chances of conception.

Question A 17-year-old primigravida presents for prenatal care and diagnosis. She states that she eats everything except meat. She also admits that she is not really fond of eggs or milk; as a result, she consumes them very rarely. Assuming her statement is true, what is most likely to be deficient in this young pregnant woman?

Correct answer: Cobalamin deficiency Explanation Cobalamin (or vitamin B12) is a nutrient required in small quantities, but it is present mostly in foods of animal origin, such as milk and milk products, meat, poultry, and fish. The recommended daily intake in a pregnant teenager, such as this client, is 2.6 mcg\day. Deficiency is rare and is associated with strict vegetarians, vegans, patients with pernicious anemia, and in cases of malabsorption, such as sprue. Deficiency may cause a variety of symptoms, such as anemia, soreness of tongue, and neurological symptoms like numbness, tingling of feet, and impaired cognitive function. Infants born to mothers with a deficiency have limited stores of the vitamin and may develop deficiency within months after birth. Delayed development, megaloblastic anemia, and failure to thrive are the potential dangers. In case this client insists on following a strict vegetarian diet, foods fortified with B12 or vitamin supplements can be prescribed. Iron, folic acid, and riboflavin can be found in green leafy vegetables. Dried beans and peas are also good sources of folic acid. Fortified cereal and beans are well-known sources of pyridoxine and iron; therefore, deficiency of these should not be a matter of concern in a healthy, vegetarian client.

Question A 34-year-old multiparous woman presents for a routine PAP smear after being "too busy" to have annual exams for the past 7 years. 3 Pap smears in her 20's have all been normal. She has had 1 episode of venereal warts in her late teens; there was no recurrence. She had 2 vaginal deliveries. She does not smoke. Remainder of her history is negative. Her Pap smear is reported as "atypical squamous cells of undetermined significance". The HPV test is positive. What is the most appropriate next step in regards to evaluating the patient?

Correct answer: Colposcopy Explanation Under the Bethesda system of reporting Pap smear results, the term atypia is reserved for abnormalities which are neither clearly reactive in nature nor meet the criteria for squamous intraepithelial lesions. Follow-up testing is required in order to make a definitive diagnosis. Colposcopy with directed biopsies of any identified abnormality is the appropriate next step in this patient, because her PAP smear shows atypia and she is HPV positive. In a patient with a negative history for HPV disease and annual PAP smears, and who can be counted on for compliance, a follow-up PAP smear at a shorter interval (6 months) would be sufficient surveillance after a first atypical PAP smear. Cone biopsy and LEEP are not appropriate until a more specific tissue diagnosis is available. Hysterectomy is not indicated for an as yet undetermined cervical lesion.

Case A 32-year-old nulligravida woman presents with gradual onset of excessive hair growth on the face, chest, abdomen, back, and upper parts of her limbs, together with the irregular menstruation. She denies changes in weight, body shape, and voice. Her menarche started when she was 8. Her family history is non-contributing. Physical examination reveals short stature, excessive male-pattern hair growth, and slight baldness. Her BMI is 18.5. The rest of the examination, including pelvic ultrasound, is normal. Question What is the most likely diagnosis?

Correct answer: Congenital adrenal hyperplasia Explanation This patient has male-pattern growth of body hair. It usually occurs in androgen-stimulated parts of the body (the face, chest, areolae). In women, androgens are produced in ovaries, adrenal glands, and the hair follicle. Precocious puberty, gradual onset of the hirsutism without virilization in second or third decade, and irregular menstruation in a woman with short stature will lead you to consider late onset congenital adrenal hyperplasia. Congenital adrenal hyperplasia (CAH) is one of the most common autosomal recessive disorders. It is proven by the presence of elevated first morning 17-hydrohyprogesterone because of the deficiency of the enzyme 21- hydroxylase. Absolute or partial deficiency of the enzyme can manifest early or late in life. There are early forms in female newborn with masculinization and ambiguous genitalia (the shunting of 17-hydroxyprogesterone to the production of androgens) or in some salt-wasting cases (the deficiency in production of cortisol and aldosterone). The combination is also possible. Late onset cases are most commonly mild. They usually manifest as some type of androgen excess later in life and without the aldosterone deficiency. An adrenal tumor will present with rapid onset of virilization, which is not the case in this patient. Elevated dehydroepiandrosten sulfate (DHEAS) produced by adrenal glands will confirm the diagnosis. Ovarian tumor will also present with rapid onset of virilization, probably with palpable adnexal mass. Elevated testosterone will confirm your diagnosis. Polycystic ovarian syndrome (PCOS) is the most common endocrine disorder of reproductive-age women. It typically presents with irregular vaginal bleeding, obesity, infertility, and hirsutism, and is characterized by ovarian dysfunction, disordered gonadotropin secretion, and hyperandrogenemia. Anovulatory cycles, infertility, amenorrhea, hirsutism, acne, and acanthosis nigrans are sometimes the result. Ultrasound may reveal hyperplastic endometrium because of the absence of ovulation and progesterone cyclic shedding (this may lead to the endometrial cancer) and characteristic bilateraly enlarged ovaries with thick capsules, multiple subcapsullar follicles, and hyperplasia of ovarial stroma. The levels of testosterone might be mildly elevated, but DHEAS and 17-hydrixyprogesterone are normal. Confirmation test consists of LH:FSH ratio and decreased level of sex-hormone binding protein. BMI that puts a patient in the underweight group and short stature are not parts of the clinical picture. In most cases, hirsutism is a benign, primarily cosmetic condition. When accompanied by masculinizing signs or symptoms, particularly when these arise after puberty, hirsutism may be a manifestation of a more serious underlying disorder such as an ovarian or adrenal neoplasm. Therefore, patients with clinical evidence of hyperandrogenemia, including hirsutism and acne, should have an evaluation of adrenal androgens. Before you declare the diagnosis of idiopathic hirsutism, you should exclude those disorders.

Question Which of the following, is a classical hydatidiform mole?

Correct answer: Diploid karyotype, absence of an embryo, swelling of all villi Explanation A classical mole, or complete mole, contains a diploid karyotype, absence of an embryo, and hydatidiform swelling of all villi. Viable fetuses may be seen with a true mole, in the case of a twin pregnancy. A triploid fetus may be present with trophoblastic hyperplasia.

Case A 53-year-old woman is seen by her gynecologist. She had 3 children, and she is tubectomized. She attained menopause at 48. Over the last few weeks, she has noted some vaginal bleeding; it occurs unpredictably. She has had regular pap smears that have always been normal. The last one was 11 months prior to presentation. Speculum examination and bi-manual palpation of the genitalia reveal no abnormalities. Question What is the next best step in management of the patient?

Correct answer: Endometrial biopsy Explanation Endometrial biopsy is the correct answer. Vaginal bleeding in a post-menopausal woman mandates endometrial sampling, with a view to ruling out endometrial cancer. Histopathological evaluation of material is essential. The most common causes of post-menopausal bleeding are vaginal atrophy (60 - 80%), exogenous estrogens (15 - 25%), endometrial cancer (10%), endometrial hypertrophy (5 - 10%), as well as cervical and endometrial polyps (2 - 12%). Repeat pap smear is incorrect. The patient had one done less than a year ago, which was normal. In light of her previously normal pap smears, the next one is not due for another 3 - 5 years. Pap smears may show the occasional dysplastic cell of endometrial origin, but it is by no means an accurate means of diagnosing endometrial pathology. Chlamydia serology is incorrect. It is done for the diagnosis of pelvic discharge in sexually active women; it is not used in cases of post-menopausal bleeding Laparoscopy is incorrect. It is not useful in the workup of post menopausal bleeding. Hormonal profile is incorrect. While exogenous estrogen is a possible cause of post menopausal bleeding, it is a diagnosis made by taking a careful history.

Case A 25-year-old Caucasian woman and her husband have been trying to have a baby for the last 2 years. The patient's medical history included cyclical pelvic pain, dysmenorrhea, and dyspareunia. The physical exam reveals the following: diffuse abdominal or pelvic pain of variable location; nodular thickening and tenderness along the uterosacral ligaments, on the posterior surface of the uterus, and in the posterior cul-de-sac; scarring and narrowing of the posterior vaginal fornix; and adnexal enlargement and tenderness. Question What is the most likely diagnosis?

Correct answer: Endometriosis Explanation Endometriosis is the growth of tissue outside the uterus. The ectopic endometrial tissue can proliferate, infiltrate, and spread to remote sites elsewhere in the body. Sites that endometriosis is most often found are ovarian, pelvic peritoneum, anterior and posterior cul-de-sac, uterosacral, round and broad ligaments, and fallopian tubes. The triad of infertility, dysmenorrhea, and dyspareunia is highly suggestive of this diagnosis. An ectopic pregnancy is one that develops at any site other than the endometrium. Symptoms of an ectopic pregnancy are abdominal pain and possibly a missed period; abnormal bleeding at the time of presentation is not uncommon. Physical characteristics of an adnexal mass that may give a presumptive diagnosis are consistency (cystic, solid or both), size (in centimeters), morphology and surface contour (smooth, nodular, vague or sharp borders), location around the uterus, mobility (free or fixed to adjacent structures), and tenderness (unilateral or bilateral). The structures supporting the urethra (urethrocele), bladder (cystocele), uterus, posterior wall of the vagina (enterocele), and rectum (rectocele) weaken. Often symptoms of this pelvic relaxation occur at or after menopause because of the lack of hormonal effect, causing atrophy of these tissues. Each of these structures has different presenting symptoms. A leiomyoma is a benign uterine tumor. Other names for leiomyoma include myoma, fibroid, and fibromyoma. Symptoms associated with leiomyoma are bleeding, pressure, and pain.

Case A 26-year-old nulligravid woman presents for a refill on her oral contraceptive pill. She states she is not sexually active and has no current need for contraception, but that she needs the pill for her 'female condition'. She cannot remember the name of her suspected disease, but states that a previous physician assistant prescribed the pills. She has taken combination oral birth control pills most of the time for the past 8 years; the patient reports that without them she has severe menstrual cramps that limit her activities. Along with pelvic cramping, she sometimes experiences nausea and low back pain. She experienced dyspareunia when she had been sexually active, despite testing negative for sexually transmitted diseases. Taking the pills regularly has reduced her symptoms to a tolerable level. When she asked about alternate contraceptives, she was told methods such as the copper intrauterine device would not help her condition. Her mother, grandmother, and an older sister all experienced similar problems. The mother and grandmother eventually had hysterectomies and the older sister's diagnosis was confirmed by surgery. Her last menstrual period was 2 weeks ago. She denies acne, hirsutism, and weight gain. She is compliant with taking pills and denies breakthrough bleeding. Her past medical history is unremarkable except for the above-described issue. She has had no surgeries, pregnancies, or other medical conditions. She takes no other medications and denies use of tobacco, alcohol, and drugs. The patient's vitals are normal. A physical exam reveals minimal tenderness throughout the pelvis and a somewhat 'fixed' uterus on bimanual exam. The rest of her physical exam is normal. Urine hCG is negative. Question What is the most likely diagnosis for this patient?

Correct answer: Endometriosis Explanation This patient's most likely diagnosis is endometriosis, a condition in which the endometrium (lining of the uterus) proliferates outside of the uterus and leads to decreased pelvic organ mobility and scar tissue development in the pelvis. Severe dysmenorrhea and dyspareunia are common symptoms. Imaging and common laboratory tests are inadequate to diagnose endometriosis, and the definitive diagnosis is made by visual observation of endometrial implants during surgery (open or laparoscopic) or by pathology from surgical specimens. It is common for women with endometriosis to report a family history of the disease. The condition is usually progressive from puberty until menopause, but the severity and course vary from patient to patient. 1 of the first-line treatments has been combination oral contraceptives, with progression to injectable Depot Leuprolide and/or laparoscopic surgery for more severe cases. In this case, even without a need for current contraception, it is reasonable to continue prescribing oral contraceptives for this patient's probable endometriosis, especially since her symptoms have been responsive to them in the past. Endometrial hyperplasia is the abnormal proliferation of endometrium within the uterus. Endometrial hyperplasia is a risk from endogenous or iatrogenic excess estrogen stimulation without adequate progestin. This progressive thickening of the endometrium is usually asymptomatic, but can eventually manifest as abnormal bleeding (e.g., breakthrough bleeding, post-coital bleeding, and post-menopausal bleeding). Hyperplasia would not cause the dysmenorrhea and dyspareunia. It is most likely to occur in older women. Pelvic inflammatory disease (PID) can be an acute or chronic condition in which inflammation occurs throughout the female genital tract (often from sexually transmitted organisms such as Neisseria gonorrhoeae and Chlamydia trachomatis). Symptoms can range from subtle discomfort or vaginal discharge to an acute abdomen in a septic patient. PID is diagnosed clinically and the diagnostic criteria are quite broad, requiring 1 or more of the following: 1) cervical motion tenderness, 2) uterine tenderness, or 3) adnexal tenderness. Unrecognized, untreated PID can lead to chronic inflammation, scarring, and infertility. Chronic PID could present very similarly to endometriosis. In this patient, since she has tested negative for sexually transmitted diseases several times, endometriosis (with her progressive symptoms and family history) is more likely. The symptoms of PID do not respond to oral contraceptives. Polycystic ovarian syndrome (PCOS) is a complex and common syndrome. In order to be diagnosed with PCOS, a patient must have 2 or more of the following: 1) menstrual history of infertility and/or oligomenorrhea, 2) polycystic ovaries demonstrated on ultrasound, 3) clinical or laboratory evidence of elevated androgens (e.g., acne, hirsutism, elevated testosterone). Some women with ovarian cysts can present with pelvic pain, but pain is not a hallmark of PCOS. Somatic symptom (somatization) disorder is a psychiatric disorder in which varied symptoms, most often pain but also including weakness, paralysis, unusual movements, and cognition problems, are present and often unexplainable by any tests or other diagnoses. Psychological stress and anxiety are often part of this disorder. While this patient does not have definitive proof of her diagnosis (e.g., a biopsy-confirmed endometriosis), all of her symptoms are readily explained by endometriosis. The presentation does not suggest a high degree of psychological stress and/or anxiety.

Case Ico-delete Highlights A 20-year-old Lebanese woman presents to a family practice office because she wants to start birth control. She has never been sexually active, and she is engaged to be married in 2 months. She feels well and has no complaints. She thinks she wants "the pill". Her fiancé is also a virgin, and they are not interested in condoms or other barrier methods of contraception. She wants to delay childbearing for at least 2 years. A summary of her past medical history includes: Medications: occasional over-the-counter ibuprofen for menstrual cramps and headaches Allergies: Penicillin Surgical history: Tonsillectomy Medical history: No known conditions. OB/GYN history: Menarche age 12. Regular monthly menses, with mild-moderate dysmenorrhea. Family history: Patient's older sister had a blood clot in her lung that followed delivery of a child. Her paternal grandfather has diabetes and hypertension. Her maternal grandmother had a stroke. Her mother had a deep venous thrombosis in a leg, and her maternal grandfather had prostate cancer. Social history: The patient works as a waitress part-time and is attending college. She currently lives in a dorm. She denies the use of tobacco, alcohol, and recreational drugs. Vitals are obtained from your medical assistant prior to the physical exam.

Correct answer: Etonogestrel subdermal implant (Nexplanon) Explanation This patient has a strong family history of hypercoagulability, with 2 first-degree relatives (a sister with a pulmonary embolism and mother with a deep venous thrombosis, or DVT), as well as a maternal grandmother with a stroke. The astute clinician needs to recognize this type of risk in an otherwise healthy young patient and avoid use of estrogens, which can increase risk of thromboembolism, until further evaluation can be done. Even if the patient presents with a desire for pills, the provider can educate her on all options. The only appropriate choice listed here is etonogestrel subdermal implant (Nexplanon), a progestin-only implant. It is placed subdermally in the upper arm, and can provide 3 years of highly-effective birth control. (Other estrogen-free contraceptive options for this patient could include progesterone-only pills, all 3 types of intrauterine devices, and the depot-medroxyprogesterone acetate injections.) Etonogestrel/ethinyl estradiol vaginal (NuvaRing) is an appropriate hormonal contraceptive option for patients in which estrogens are safe to use. It provides a monthly cycle and avoids daily compliance issues. It is inserted vaginally for 3 weeks and removed for 1 week; the patient is then instructed to begin the cycle again with a new ring. The levonorgestrel/ethinyl estradiol oral, extended cycle (Seasonique) is a oral combination hormonal contraceptive. Its main benefit is reducing the number of menstrual cycles to 4 per year. While this would benefit this particular patient in reducing her dysmenorrhea, the estrogen presents too high of a risk for a thromboembolism. Norelgestromin/ethinyl estradiol transdermal (OrthoEvra) is the contraceptive patch; it is applied weekly for 3 weeks, and then the user remains patch-free for 1 week before beginning the cycle over again. Similar to the NuvaRing, it provides a benefit in compliance, but it is otherwise similar in side effect provide and benefits of the combination oral contraceptive pills. OrthoEvra has been under study, and it currently contains an additional black box warning related to even higher risk of thromboembolism over other estrogen-containing methods. It should especially be avoided in this patient. Norgestimate/ethinyl estradiol oral (Sprintec) is a commonly used mid-dose combination hormonal contraceptive. Generally, it is well-tolerated and inexpensive. However, the estrogen component in this medication presents a risk for thromboembolism, as do the other choices listed above. Further investigation may reveal this patient does not carry a genetic tendency for thromboembolism. However, until then, the clinician should err on the side of caution and avoid estrogen-containing methods.

Case A 65-year-old woman is concerned about a non-tender lump she discovered in her right breast last month. When performing a breast self-exam this month, she noticed the lump persists, but it is unchanged. On examination, you feel a firm, unfixed, non-tender 2 cm mass in the right upper outer quadrant; there are no palpable axillary nodes. Mammography and a fine-needle aspirate (FNA) are inconclusive. Question What is the next most appropriate step?

Correct answer: Excisional biopsy Explanation Establishing a diagnosis under these circumstances requires excisional biopsy and pathological examination of the lesion. In this case, the mammogram and FNA do not explain the clinical findings despite the usual sensitivity of the FNA (>90%). Ultrasound studies may add some information but are typically not useful in establishing a definitive diagnosis. Initiating surgical or medical therapy is inappropriate, as would be delaying a diagnosis for 3 months.

Question A nulliparous 25-year-old woman has had an uneventful, spontaneous labor at term. She has an epidural anesthetic and has been in the second stage of labor for 2 hours. The fetal heart tones show some signs of deterioration. You decide to expedite the delivery with the use of forceps. What criterion defines the application of low forceps?

Correct answer: Fetal skull is at station +2, but not on the pelvic floor Explanation Low forceps is defined by the skull being at station +2 or greater, but not on the pelvic floor. Outlet forceps includes situations where the scalp is visible, the fetal skull has reached the pelvic floor, or the head is on the perineum. In addition, rotation must not exceed 45 degrees. Midforceps is the head engaged but above a station of +2.

Question What is a part of the clinical criteria for diagnosing bacterial vaginosis (BV)?

Correct answer: Fishy odor of vaginal discharge before or after the addition of 10% KOH Explanation BV can be diagnosed by the presence of 3 of the following signs or symptoms: homogenous thin white discharge that smoothly coats the vaginal wall clue cells under microscopy vaginal fluid pH of >4.5 fishy odor before or after adding 10% KOH Thick cottage cheese-like vaginal discharge and budding hyphae on microscopy are diagnostic considerations for vulvovaginal candidiasis. Frothy malodorous yellow-green discharge is associated with trichomoniasis infection.

Case A recently married 22-year-old woman presents with a 7-day history of severe vulval and vaginal pruritus and purulent vaginal discharge. She was having dysuria and dyspareunia. Examination showed frothy yellowish mucopurulent vaginal discharge with an offensive odor. Vaginal mucosa appeared inflamed, and cervical erosion was also observed. Vaginal secretion was collected, and a microscopy of wet film done immediately. Oval pear-shaped organisms, which were about the size of white blood cells and had wobbling rotatory motility, were observed among inflammatory cells. Gram stain was negative for Candida and Clue cells. Based on the diagnosis, the patient and her husband were treated with metronidazole. Question What is a characteristic of the microbial agent causing symptoms in the above patient?

Correct answer: Flagellated protozoan Explanation The microorganism causing vaginitis in the patient is Trichomonas vaginalis, a flagellate protozoan parasite. For T.vaginalis, no cyst form is known. The parasite has 4 anterior flagella and a 5th flagellum along the undulating membrane. Flagella are the organs of motility. Cilia are absent. The undulating membrane of T.vaginalis is short and reaches up to the middle of the body. This is a differentiating feature from other trichomonads of humans in which the undulating membrane extends full length of the parasite. The organism grows best under anaerobic conditions at 35-37°C with an optimal pH 5.5 to 6. Normal acidic pH of 3.8- 4 is detrimental to the growth of T.vaginalis. The trophozoite, a protozoan in the metabolically active growth stage, cannot survive outside the body for long, so transmission has to be from person to person by close contact. Humans are the only natural hosts of the parasite. T.vaginalis lives mainly in the vagina and cervix in women and in the anterior urethra of men. Sexual transmission is the common mode of infection. The parasite divides by longitudinal binary fission. Trichomoniasis is the most common non-viral sexually transmitted disease of worldwide distribution. Incidence of Trichomoniasis in the United States is high. An estimated 8 million new cases occur each year. Infection is most common in sexually active women of reproductive age. The incubation period ranges from 4 days to 4 weeks. Infection may be asymptomatic or cause acute inflammatory disease of the vagina and cervix. Trichomoniasis is characterized by an increased pH of vagina, which is brought about by decrease or elimination of Lactobacillus species that constitute normal vaginal flora. Hormones also may play a role in the susceptibility to infection as evidenced by the worsening of symptoms during menstruation. Iron from the menstrual blood is thought to help Trichomanas vaginalisto attach to the squamous epithelium of the genital tract. Complications of the infection include cervical erosion and infertility. In pregnant women infected with the organism, premature labor and low birth weight infants have been observed. Non-sexually transmitted form of the disease may occur in neonates (neonatal trichomoniasis) born to infected mothers. Symptomatic as well as asymptomatic cases have been reported. By causing acute local inflammation of the genital mucosa, T.vaginalis may act as a potential catalyst in the acquisition or transmission of other STDs like HIV and Human Papilloma Virus (HPV), the virus associated with pathogenesis of carcinoma of cervix (Ref:6). Infection in men is usually mild or asymptomatic. Clinical manifestation of urethritis may simulate non-gonococcal urethritis. Prostatitis, epididymitis, and infertility can occur as complications. Laboratory diagnostic methods include microscopy, culture, antigen detection tests by enzyme-linked immunosorbent assay (ELISA), and molecular tests based on polymerase chain reaction (PCR). Microscopy for motile trichomonas in wet preparation of vaginal discharge should be performed within 10-20 minutes of collection to prevent the organisms from losing their viability. Smears stained by Giemsa stain or Acridine orange helps to demonstrate structural details of the parasite (video clips showing motility of T.vaginalisand microscopic appearance of the parasite in Giemsa-stained preparation are accessible in reference 7). Nucleus, flagella, undulating membrane, and axostyle are prominent structures in stained smears. A culture is the gold standard, and special media like Diamond's medium are used. Growth result is obtained in 2-5 days. Rapid ELISA assay with good sensitivity and specificity for antigen detection is of value in settings without facilities for microscopy. PCR-based tests for detection of T.vaginalis in clinical samples using different primers are under development and validation. Self-obtained specimens of vaginal swabs can also be used for the test. PCR test is reported to be of use when shipping of specimens to a reference laboratory is required. Urine and urethral swabs are good clinical samples for diagnosing T.vaginalisinfection in men. Centers for Disease Control and Prevention (CDC) recommends metronidazole 2 grams orally as a single dose for treatment of T.vaginalis infection. Treatment of sexual partners is also recommended. Metronidazole resistant infections have been reported and Tinidazole, a 5-nitroimidazole, is useful for treatment of such cases.

Case An 18-year-old woman presents for advice regarding emergency contraception. She had sexual intercourse the day prior to presentation; she did not use contraception, and she wants to know if it is possible that she is pregnant. Her last menstrual period (LMP) began 5 days prior to presentation. She usually gets her menses once in 28 days and the period usually lasts for 5 days. The physical exam is normal. Question What hormone is responsible for stimulating follicular development during this phase in the menstrual period?

Correct answer: Follicle-stimulating hormone (FSH) Explanation The fifth day of the menstrual cycle corresponds to the follicular phase of the ovarian cycle that starts from the first day of bleeding and is characterized by a rise in FSH. Increased FSH stimulates a cohort of primary follicles to begin rapid gonadotropin-dependent growth of granulosa cells. Granulosa cells are the only ovarian cells with FSH receptors and are responsible for estrogen synthesis. Thus, FSH stimulates growth of granulosa cells and estrogen synthesis which, in turn, acts to further support the trophic effects of FSH on granulosa cells. Estrogen is produced by the granulosa cells of the follicle; they are very important in the follicular phase. Their level is very low at the beginning of this phase and reaches its maximal point at the middle of the cycle, just before the LH peak. The cells are the feedback for the LH production, and in part, for the FSH production. Estrogens are responsible for changes in the endothelial lining of the uterus that occur in pregnancy. They also promote sexual development. Inhibins are also hormones produced by granulosa cells of the follicles; they are part of the negative feedback for the FSH. Inhibin B levels rise during the luteal-follicular transition, are highest during the mid-follicular phase, go up again during the LH peak, and finally decrease in the late follicular phase. Inhibin A levels decrease during the late luteal phase. Ovulation occurs approximately at the middle of the menstrual cycle (day 14). At the end of the follicular phase, a rise in the level of estrogens takes place preceding the LH peak. Ovulation will occur an average of 30 hours after this peak. The second half of the menstrual cycle is the luteal phase, which starts from the day of ovulation to the first day of menstruation. In this phase, the ovarian corpus luteum begins to produce progesterone as well as estrogen. Progesterone helps prepare the uterine lining for potential implantation. If there is fertilization of the ovum, the production of progesterone continues for 4 to 5 weeks until the placenta can produce a sufficient quantity of the hormone to maintain the pregnancy. If there is no fertilization, progesterone levels decrease and menstruation occurs.

Case A 22-year-old nulligravid Caucasian woman presents to your OB/GYN clinic for evaluation of menstrual changes. She reports having normal menses since menarche at age 13, but then noticed she has only had 3 menses over the last year, with the last one about 3 months ago. She denies dysmenorrhea and menorrhagia, any major life event changes, and any major stressors. Review of systems is positive for a recent increase in severe headaches. Past medical and surgical histories are unremarkable, with no known medical conditions or surgeries. She takes no chronic medications but has been using over-the-counter analgesics for her headaches. She has no allergies. Social history reveals the patient has never been sexually active and lives at home with her parents and siblings. She does not use alcohol, tobacco, or drugs. Vitals and a urine specimen for hCG are obtained from your medical assistant prior to physical exam. Weight 152 lb Height 67" Pulse 70 Blood pressure 112/66 Temperature 97.2°F Urine hCG Negative Question What physical exam finding would be seen in a condition that explains both the headaches and amenorrhea in this patient?

Correct answer: Galactorrhea Explanation Galactorrhea would suggest elevated levels of serum prolactin and a prolactinoma in the pituitary as the underlying cause. A prolactinoma is associated with menstrual changes (often oligomenorrhea and amenorrhea), galactorrhea, headaches, and visual changes. Acanthosis nigricans is a hyperpigmentation of the skin, especially in the neck folds, and is associated with elevated insulin levels, diabetes, and hyperandrogenism. These conditions are commonly associated with polycystic ovarian syndrome and amenorrhea or oligomenorrhea, but there is no direct association with headaches. Hirsutism is a sign of hyperandrogenism and is often seen in women with polycystic ovarian syndrome, although it can occur due to other causes. The serum testosterone levels may or may not be elevated. Hyperandrogenism is linked with menstrual changes but not directly with headaches. Imperforate hymen is a suspected cause in primary amenorrhea, in which the woman has never had a menstrual cycle. This woman presents with secondary amenorrhea, so her hymen would not be expected to be intact. This condition also has no influence on headaches. Thyroid nodules and overall thyroid dysfunction can play a role in menstrual regularity, but a nodule would not directly cause headaches.

Case A 24-year-old African American G2P1 patient at 37 weeks gestation presents to her obstetrician without any complaints. The patient has no significant past medical history, takes no medications, and denies any allergies. The mother's prenatal laboratory testing was unremarkable. The patient's family medical history is also unremarkable. She feels the baby moving frequently. The patient's vital signs are as follows: temperature is 98.2°F; pulse is 79 beats/minute; respirations are 11; and blood pressure is 104/59. Urinalysis reveals no abnormal findings. Laboratory values are as follows: Hemoglobin: 12.2 g/dL (Normal: 13.0-16.0 g/dl) Hematocrit: 37% (Normal: 37-47) Platelets: 88,000/mm3 (Normal Value: 130 - 400 thous/mm3) White Blood Cells: 9.7 thous/mcl (Normal Value: 3.8 - 10.8 thous/mcl) AST: 35 IU/l (Normal: 7-40 IU/l) ALT: 35 IU/l (Normal: 0-40 IU/l) Alkaline Phosphatase: 195 U/l (Normal Values: 70-230 U/l) Question Which of the following is the patient's most likely diagnosis?

Correct answer: Gestational Thrombocytopenia Explanation Thrombocytopenia is seen in around 10% of patients who are pregnant and is most commonly caused by gestational thrombocytopenia, although it can be seen in more serious conditions, such as DIC, HELLP, and TTP. Other conditions that can cause low platelet counts during pregnancy are medications, connective tissue disorders (systemic lupus erythematosis, etc.), hemolytic uremic syndrome, and HIV infection. Gestational thrombocytopenia is characterized by: (1) a mild decline in platelet count (usually above 70,000/mm3); (2) no history of bleeding difficulties or thrombocytopenia prior to the pregnancy; and (3) a return to a normal platelet count 2-12 weeks after delivery. Gestational thrombocytopenia is relatively benign condition, and the fetus and mother require no additional therapy. Disseminated intravascular coagulation (DIC) can occur as a consequence of HELLP syndrome, placental abruption, sepsis, amniotic fluid embolism, or intrauterine fetal demise. Patients with DIC have clinical manifestations of epistaxis, hematuria, purpura, and bleeding from puncture sites or incision sites. Fibrinolysis is an important component of DIC, and laboratory values show elevated levels of D-dimer and fibrin degradation products (FDPs). Platelet counts are usually low; clotting times (aPTT and PT) are elevated; and antithrombin levels, as well as individual clotting factors (V and VII), may also be diminished. Given the lack of associated risk factors, DIC would be an unusual cause for thrombocytopenia in this patient. HELLP Syndrome is associated with third trimester pre-eclampsia, HTN, and proteinuria, and the HELLP acronym stands for Hemolysis, Elevated Liver Tests, and Low Platelets. While most patients with pre-eclampsia can be managed expectantly, patients with HELLP need to given IV magnesium sulfate to prevent seizure activity. Induction and delivery (based on fetal maturity) is the best course of action to treat HELLP, as long as the fetus is mature. Idiopathic thrombocytopenic purpura (ITP) is a diagnosis of exclusion and reflects a disorder of increased peripheral platelet destruction, as most affected patients have auto-antibodies to specific platelet membrane glycoproteins. Patients usually have physical findings of thrombocytopenia, including purpura, increased bruising tendencies, and epistaxis. In the patient with known ITP, high-dose IV glucocorticoids and IV immunoglobulin (IVIG), with or without platelet transfusions, are appropriate treatments. Splenectomy is usually reserved for patients in whom medical therapy fails. Thrombotic thrombocytopenic purpura (TTP) is the multi-system condition characterized by microangiopathic hemolysis and platelet aggregation with hyaline thrombi. These platelet microthrombi predominate, forming in systemic arterioles and capillaries, leading to partial occlusion of vessels. Organ ischemia, thrombocytopenia, and erythrocyte fragmentation then occur. Additionally, most patients have altered mental status, including confusion, generalized headaches, focal neurological deficits, seizures, visual disturbances, or coma. Renal insufficiency and gross hematuria may also be seen.

Question Which one of the following bacteria is carried by about 20% of women of child-bearing age and is a cause of sepsis and meningitis of the newborn?

Correct answer: Group B streptococci Explanation Group B streptococci are carried by about 20% of women of childbearing age in their vagina and are a cause of sepsis and meningitis of the newborn and are acquired during passage through the birth canal. Lactobacilli, Bacteroides, Bifidobacterium and Clostridium are normally found in the colon and do not cause sepsis and meningitis of the newborn.

Case A 21-year-old woman has been seen on an annual basis for the past 5 years. She is 66 inches tall, weighs 205 lbs, has a blood pressure of 130/70 mm Hg, and has a temperature of 98.5°F. Her physical exam is normal and lungs are clear to auscultation. The results of her first Pap smear came back negative, with no endocervical component. Question What should be done next in the management of this patient?

Correct answer: Have her come back in 3 years unless problems arise Explanation In this case, you should have her return in three years because there is no need to repeat the smear unless this Pap smear from this patient is abnormal. Endocervical/transformation zone cells are NOT required for a cervical cytology test to be classified as satisfactory. Women should have a Pap smear every 3 years starting at the age of 21. From ages 30-65 years, screening should be either every 5 years with HPV screening or every 3 years without HPV screening. After the age of 65, a woman may stop having Pap smears, as long as her pap smears were normal previously or no history of CIN2+ within the last 20 years.

Case A 31-year-old multigravida known to have blood group A and Rh-negative red blood cells is pregnant with her 3rd child. Her husband is also type A, but he is Rh-positive. She has an indirect Coombs titer at 1:16 dilution of her serum at 28 weeks gestation. Her past medical history includes 2 pregnancies; her 1st child, a boy, was healthy, while the 2nd child, a girl, was born at 36 weeks gestation after the mother was noted to have an indirect Coombs titer at 1:16. Amniotic fluid obtained at 26, 28, 30, 32, and 34 weeks of gestation was analyzed by determining the optical density (OD) for bilirubin and indicated a progressive increase in the bilirubin level. Question What is the most likely explanation for the increase in bilirubin?

Correct answer: Hemolysis of red blood cells in the fetus Explanation The correct answer is hemolysis of red blood cells in the fetus. An increased level of bilirubin is most likely the result of breakdown of heme, indicating that the fetus's red blood cells were hemolyzed. The fetus most likely suffers from HDN (hemolytic disease of the newborn). The answer high red blood cells count is incorrect. An increased production of red blood cells is associated with HDN. If suffering from this disease, the child's body tries to compensate from the red cell hemolysis by producing more of them quickly. Most of the newly produced cells are immature and cannot function properly. As a result, the child can be severely anemic; however, the hematocrit is simply measured by centrifugation of unclotted blood and cell pellet volume assessment as a percentage of total blood volume, not by spectrophotometry. The answers increased maternal IgG and Rh factor is present in the blood are incorrect. In HDN, increased hemolysis of the Rh-positive fetal red blood cells is caused by the maternal IgG antibodies that cross the placenta in increasing amounts during the 2nd trimester of pregnancy; however, IgG and Rh are both proteins, and they are usually detected by interactions with specific antibodies, not by changes in the optical density of the amniotic fluid. Swelling of the fetus's body is not correct. In HDN, if the fetus cannot cope with severe anemia, its heart can fail while fluid accumulates in the organs causing swelling; however, such anomalies of the fetus are usually detected by ultrasonic visualization, not by optical density changes of amniotic fluid.

Case A 24-year-old African American G2P1 presents to her obstetrician at 34 weeks gestation with weight gain, fatigue, diffuse "swelling," and headache. On physical examination, you note periorbital edema and 3+ pitting edema in the lower extremities peripherally. Physical examination also demonstrates tenderness to palpation in the right upper quadrant. The patient has increased reflexes bilaterally. Vital signs are as follows: Temperature is 99.2°F. Pulse is 89 beats/minute. Respirations are 18. Blood pressure is 174/99 mm Hg. Urinalysis reveals no erythrocytes per high power field and no casts, but there is a large amount of protein in the urine. Laboratory values are as follows: Test Result Normal Range Hemoglobin 9.2 g/dL 13.0-16.0 g/dL Hematocrit 27% 37-47% Platelets 38 K/mm3 130-400 K/mm3 White blood cells 12.7 K/μL 3.8-10.8 K/μL AST 595 IU/L 7-40 IU/L ALT 585 IU/L 0-40 IU/L Alkaline phosphatase 295 U/L 70-230 U/L Question What is the most likely diagnosis?

Correct answer: Hemolysis, elevated liver tests, and low platelets syndrome (HELLP) Explanation These laboratory findings are not normal, and the clinical history and low platelets are most consistent with HELLP syndrome. HELLP Syndrome is most often associated with third-trimester pre-eclampsia, HTN, and proteinuria. The acronym HELLP stands for Hemolysis, Elevated Liver Tests, and Low Platelets. While most patients with pre-eclampsia can be managed expectantly, patients with HELLP need to be induced for delivery (based on the maturity of the fetus), given the high risks associated with this condition. Patients with HELLP syndrome also need to be given magnesium sulfate to prevent seizure activity. Disseminated intravascular coagulation (DIC) can occur as a consequence of HELLP syndrome, placental abruption, sepsis, amniotic fluid embolism, or intrauterine fetal demise. Patients with DIC have clinical manifestations, including epistaxis, hematuria, purpura, and bleeding from puncture sites or incision sites. Fibrinolysis is an important component of DIC, and laboratory values show elevated levels of D-dimer and fibrin degradation products (FDPs). Platelet counts are usually low; clotting times (aPTT and PT) are elevated; antithrombin levels and individual clotting factors (V and VII) may be diminished. Given the lack of associated risk factors, DIC would be an unusual diagnosis in this patient. Idiopathic thrombocytopenic purpura (ITP) is diagnosis of exclusion and reflects a disease of increased peripheral platelet destruction, as most patients have antibodies to specific platelet membrane glycoproteins. Patients usually have physical findings of thrombocytopenia, including purpura, increased bruising tendencies, and epistaxis. In this patient with known ITP, high-dose parenteral glucocorticoids and IV immunoglobulin (IVIG), with or without platelet transfusions, are appropriate treatment. Splenectomy is usually reserved for patients when medical therapy fails. Thrombotic thrombocytopenic purpura (TTP) is the multi-system condition characterized by microangiopathic hemolysis and platelet aggregation with hyaline thrombi that forms unrelated to coagulation system activity. In TTP, platelet microthrombi predominate, forming in systemic arterioles and capillaries, leading to partial occlusion of these small vessels. Organ ischemia, thrombocytopenia, and erythrocyte fragmentation ensue. Additionally, most patients have altered mental status, including confusion, generalized headaches, focal deficits, seizures, visual disturbances, and/or coma. Renal insufficiency and gross hematuria may also be seen.

Case A 46-year-old woman presents because she has not had her period for almost a year, has hot flashes, and feels fatigued most of the time. She complains of insomnia, and states that she keeps smoking despite trying many times to quit. Her libido is low and she does not use any hormone-based birth control. Her blood analysis indicates a TSH (thyroid-stimulating hormone) level of 3.1 mcU/mL and a FSH (follicle-stimulating hormone) level of 55.3 mIU/mL. Question What is the most accurate conclusion regarding the menstrual status of this patient?

Correct answer: Her FSH level is consistent with menopause Explanation The correct response is that her FSH level is consistent with menopause. Based on the symptoms and blood analysis of the patient, the most likely diagnosis for this patient is menopause; she has not had a period for almost 12 months and her elevated FSH level is consistent with menopause. There is no known correlation between the TSH level and menopause or the menopause transition. Primary ovarian insufficiency is defined as a failure of ovarian function in women younger than 40. This leads to menopause-like symptoms, irregular or absent periods, and infertility; however, the FSH level is not elevated (higher than 40 mIU/mL), as is the case in menopause. The patient's age is not consistent with menopause is an incorrect response. Even though the average age for entering menopause is 51, women can enter menopause earlier.

Case A 52-year-old woman presents because her menopausal symptoms have been extremely distressing. Over the past 4 months, she has experienced severe mood swings, hot flashes, night sweats, breast tenderness, and changes in her appetite. She has never smoked; she has an occasional drink. She had an IUD that was removed at 35 years of age. There is no family history of cancer. After a prolonged discussion, a decision to start hormone replacement therapy is made. Question What is a legitimate concern with the use of an estrogen-only supplement?

Correct answer: Higher risk of endometrial cancer Explanation A higher risk of endometrial cancer is the right answer. When estrogen is administered as the sole therapy without cyclical progesterone, it causes unopposed proliferation of the endometrium. Over a period of time, this increases the chance of dysplasia, leading to endometrial cancer. To prevent this, estrogen is usually administered with cyclical progesterone. Vaginal dryness and itching is incorrect. Estrogen helps to maintain the epithelial lining of the vagina and increases lubrication, thereby decreasing the symptoms of postmenopausal atrophic vaginitis. Worsening of mood swings is incorrect. Hormone replacement therapy during menopause is beneficial in mitigating mood symptoms. When used, they are prescribed in the lowest dose possible for the shortest period of time. Lowering of serum HDL level is incorrect. Estrogen does exactly the opposite; it raises HDL and lowers LDL levels. It acts as a cardioprotective agent in that respect. Higher risk of vertebral fractures is incorrect. Estrogen influences bone metabolism and prevents osteoporosis.

Case A 27-year-old woman presents with cramping abdominal pain and vaginal bleeding. Further history reveals amenorrhea for the past 2 menstrual cycles. On examination, she is found to have left lower abdominal tenderness and an adnexal mass. Lab values reveal an elevated Beta HCG level. Question A further history would possibly reveal what associated risk factor?

Correct answer: History of PID Explanation A history of pelvic inflammatory disease is the correct answer. The vignette describes a potential ectopic pregnancy, evidenced by the triad of amenorrhea, adnexal mass, and an elevated B-HCG. Inflammation of the fallopian tubes secondary to bacterial infection resolves with fibrosis and adhesion formation. The fertilized egg gets trapped on its transit through the fallopian tube, resulting in an ectopic pregnancy. Being thin walled, implantation in the fallopian tube invariably results in a rupture. Diaphragm use is incorrect. Diaphragms sit atop the cervix, preventing sperm entering the uterine cavity. They are used as a temporary measure and are not associated with ectopic implantation. Condom use is incorrect. It is used as a temporary measure, and condom use is not associated with ectopic implantation. Oral contraceptive use is incorrect. Regular OCP use is associated with an extremely low failure rate. In the event of contraceptive failure, there is no increased risk of ectopy with OCP use. Vaginal spermicide use appears to decrease a woman's risk of acquiring bacterial STDs, particularly C. trachomatis and N. gonorrhoeae cervical infections, the most common organisms in cases of PID. In addition, most women use a spermicide with a diaphragm which may further decrease the risk for PID.

Question A 35-year-old female sees a gynecologist for the first time in many years. She has multiple sexual partners and wishes to discuss a different form of birth control and disease prevention. Diagnostic results indicate cervical cancer that has spread beyond the cervix and involves some of her vagina. What virus is associated with the development of cervical cancer?

Correct answer: Human papilloma virus Explanation Human papilloma virus is a papovavirus. Human papilloma virus is responsible for warts. Human papilloma virus is associated with the development of cervical cancer. Epstein Barr virus is a herpesvirus. Epstein Barr virus is the cause of infectious mononucleosis. Epstein-Barr virus is associated with the development of the African form of Burkitt's lymphoma. Epstein -Barr virus is also associated with nasopharyngeal carcinoma. Hepatitis B virus is a DNA virus. It is also referred to as the Dane particle. There is an association of hepatitis B infection, and the development of hepatocellular carcinoma. Varicella-zoster virus is a herpesvirus. Varicella zoster virus is responsible for chicken pox and shingles, among other things. Varicella-zoster virus is not associated with the development of cancer in humans. HTLV is a retrovirus. HTLV-I is associated with T-cell leukemia/lymphoma.

Case You are performing an annual physical examination of a 14-year-old girl. Over the last 3 years, she has been treated for the depression related to her parents' divorce process. She complains of frequent constipation followed by loose stools and is treated for dermatitis herpetiformis. She participates in swimming competitions and spends about 18-20 hours per week training. However, her mother is worried because she has not grown enough and still has not gotten her period (her mother had her period when she was 12 years old). The rest of her personal and family history is not contributing. Your patient's BMI is 15 (percentile 3%); she is in Tanner stage 2 (the same as last year); and her bone age is 12.5 years. The rest of the physical examination is normal, and complete blood count results are within normal limits. Question What is the most likely diagnosis?

Correct answer: Hypogonadotropic hypogonadism Explanation Your patient has a delay in puberty, in weight gain, in linear growth, and has gastrointestinal symptoms as well. That should prompt you to think about the systemic disease with chronic undernutrition (in this case, most probably celiac disease). Her hypothalamic-pituitary-gonadal axis also may be suppressed because of the stress and the excessive exercise. Therefore, she most probably has hypogonadotropic hypogonadism. Hypogonadotropic hypogonadism may present prior to or after the completion of puberty. It involves slowed gonadotropin-releasing hormone release, leading to low levels of FSH and LH. Your patient was under stress, but she also has bone maturation and growth and weight gain delay, which should be investigated. Constitutional pubertal delay is a diagnosis of exclusion. Children with constitutional delay typically present with delayed growth, adrenarche, and sexual development, associated with a decline in growth velocity and delayed skeletal maturation. But before making the diagnosis of the constitutional delay, you must exclude other causes. In this case, you have signs and symptoms that make constitutional delay unlikely. Hypergonadotropic hypogonadism is due to the impaired response of the gonads to the gonadotropins. In most cases, it is congenital (chromosomal aberrations, enzyme defects), is part of different syndromes, and usually is not accompanied by a delay in growth, weight, and anemia.

Case A 24-year-old nulliparous woman presents because she has been trying to become pregnant for 3 years, but she has not been successful. Her husband has been tested for infertility, and she tells you that his sperm count is normal and that he has changed his undergarment from briefs to boxers. When inquiring about her medical past, she states that when she was 15, she had an inflamed appendix that led to abscess formation. Question What diagnostic study would be most helpful at this point?

Correct answer: Hysterosalpingogram Explanation The clinical picture is suggestive of infertility due to scarring of the fallopian tubes from adhesions secondary to the abscess formation from the inflamed appendicitis. A hysterosalpingogram is indicated if structural abnormalities are expected. The Pap smear is indicated when investigating cervical cancer, which is not seen in this patient. Laparoscopy is indicated if the hysterosalpingogram is abnormal. TSH level would be indicated if she had a previous pregnancy loss. A pelvic ultrasound would reveal nothing diagnostically about the tubes.

Case A 27-year-old para 2 develops a fever on postnatal day 5. A sustained elevated temperature over 24 hours prompts a thorough examination by her physician. Her breasts are engorged and non-tender with no erythema. Her uterus feels firm and well contracted, but exquisitely tender to palpation. Speculum examination reveals a normal appearing cervix with no discharge. A presumptive diagnosis of puerperal sepsis is made and broad spectrum antibiotics (Ampicillin/Gentamycin/Metronidazole) are initiated. There is no improvement after 48 hours. Regular temperature spikes are noted, and the abdominal pain has increased in severity. The pelvic sonogram shows no retained products of conception or pelvic fluid collection suggestive of an abscess. Question What is the next best step in management?

Correct answer: IV Heparin administration Explanation IV Heparin administration is the correct answer. The failure of antibiotics in the setting of puerperal sepsis must prompt the consideration of pelvic thrombophlebitis as a diagnosis. 48 hours of antibiotic therapy have resulted in no improvement. Sonogram reveals no pelvic or abdominal abscess formation, a differential diagnosis to the lack of response to antibiotics. IV Heparin is administered for the presumptive diagnosis of pelvic thrombophlebitis pending confirmation. Laparotomy is incorrect. Pelvic thrombophlebitis is managed conservatively. There is no role for operative management. IV Ceftriaxone administration is incorrect. With a combination of broad-spectrum antibiotics being ineffective, it is unlikely that the addition of Ceftriaxone would help. Furthermore, the presence of fever and abdominal pain in post-partum women, with no discernible signs of pelvic infection warrants treatment for thrombophlebitis. Plain CT abdomen is incorrect. A CT scan without contrast would not be of use. Addition of antifungal to regimen is incorrect. Fungal infection is far more unlikely that thrombophlebitis in this scenario.

A 48-year-old Caucasian woman presents due to feeling like she is losing her mind. She wants some tests done. Upon further questioning, she reports she is having multiple episodes daily in which she suddenly becomes very hot, flushed, and diaphoretic. These episodes last about 1 minute, then resolve. She has not measured a fever. The patient reports that the episodes occur during the day and at night, causing her to wake up drenched in sweat. As a result, her sleep has been poor, and she feels fatigued and irritable at both work and home. She has noticed these symptoms for about 2 months, and they seem to be increasing in severity. The patient has not had a period for 3 months; she recently did a home pregnancy test, which was negative. Prior to that, she had regular menses. This patient denies weight changes, palpitations, cold intolerance, bowel changes, as well as changes in her nails, skin, and hair. Although she admits irritability, she denies anxiety, depressed mood, and suicidal ideation. Her family history is remarkable for diabetes in her maternal grandfather and hypertension in her father. She is a G4P3Ab1. She denies any major psychosocial stressors recently. She drinks alcohol rarely, and she denies use of other drugs. Vitals and a urine specimen for hCG were obtained prior to the physical exam. Weight 168 lb Height 65" Pulse 72 Blood pressure 120/82 Temperature 98.2°F Urine hCG Negative Complete screening physical exam is normal, with normal sexual development and absence of hirsutism and acne. Question What diagnostic study result is most consistent with the suspected diagnosis?

Correct answer: Increased follicle stimulating hormone (FSH) Explanation This patient would be expected to have increased follicle stimulating hormone (FSH) levels. Based on the history and physical, her diagnosis is perimenopause, with a classic history of vasomotor symptoms (hot flashes/flushes and night sweats). Perimenopause is defined as the time prior to menopause in which the woman's ovarian hormone output begins to decline, often beginning in the mid-40s. Menopause is defined at the point in time in which the patient has been amenorrheic for 12 months. Postmenopause occurs after that. The average age for menopause is just over 51 years of age. With her history, a negative pregnancy test, and normal physical exam at age 48, the diagnosis of perimenopause can actually be made without diagnostic studies. If tests are done, however, it is recommended to check the "upstream" hormones from the pituitary (follicle-stimulating hormone and/or luteinizing hormone) rather than the ovarian hormones themselves (estradiol and progesterone). FSH elevation is the first measurable hormone evidence of menopausal ovarian failure. With the physiologic feedback mechanism, both FSH and LH rise in response to a low ovarian output of inhibin, which would normally suppress FSH in a younger woman. A decreased free thyroxine (free T4) level would indicate hypothyroidism. Although thyroid function should be considered in women having menstrual changes and fatigue, this patient denies other symptoms of hypothyroidism (weight gain, dermatologic changes, cold intolerance, constipation, and depressed mood). Hypothyroidism is not associated with hot flashes and night sweats. Decreased luteinizing hormone (LH) would be expected in situations in which there is primary pituitary or hypothalamic dysfunction or suppression, such as with the administration of hormonal contraceptives. Eating disorders and malnutrition may also cause low LH levels. This patient presentation does not indicate any of these disorders. In perimenopause, and more dramatically in menopause, estradiol levels decrease. An increased estradiol level would not be expected in this patient. Increased testosterone levels are most often seen in women due to polycystic ovarian syndrome (PCOS); they can also be seen in women with primary tumors of the ovaries, in which androgen production is increased. PCOS is characterized by menstrual changes consistent with anovulation (amenorrhea, oligomenorrhea, and history of infertility), hyperandrogenism (e.g., hirsutism and acne), and characteristic polycystic ovaries are seen on ultrasound. PCOS may be associated with obesity and insulin resistance or diabetes. The ovarian tumors are rare. This patient does not have a presentation consistent with either PCOS or ovarian tumor. Increased thyroid stimulating hormone (TSH) would indicate hypothyroidism. TSH is released from the anterior pituitary in response to low thyroid hormones. In a situation with low thyroid hormone output, the TSH rises. As in the explanation for elevated free T4, this patient's symptoms are not fully explained by a diagnosis of hypothyroidism.

Case A 50-year-old woman presents for her annual pelvic examination. She states her last menstrual period was over 6 months ago; the last few occurrences of menses were extremely irregular. The patient also describes having the sensation of intense heat in her face and trunk; the sensation is accompanied by sweating. She further states that these "heat episodes" have been occurring 1 or 2 times a week for the last several months. She has no other complaints at this time. She has received her annual pap and pelvic examination yearly, as well as a clinical breast exam, without any issues. During the pelvic examination, you note obvious vaginal thinning and excessive dryness; there is also apparent vaginal wall atrophy. Question Laboratory findings in this patient would include a decreased serum estradiol as well as what?

Correct answer: Increased serum follicle stimulating hormone (FSH) Explanation This patient scenario above is most likely to be caused from menopause. In the most pure sense of explanation, menopause is a cessation of menstruation from either natural aging (usually amenorrhea for at least 6 months) or an external cause (surgical). There is usually a 1 - 3 year time period during which women will typically adjust physiologically to the diminished hormonal and menstrual actions as well as the effects this has on their body. These effects may include hot flashes, night sweats, vaginal dryness, and in the later stages, osteoporosis. The average age of menopause in western societies is around 51 years old. Laboratory findings that indicate natural menopause include elevated levels of FSH, LH, decreased levels of estradiol (estrogen), and normal levels of TSH and prolactin. Decreased levels of TSH would be seen in hyperthyroidism, which is also referred to as thyrotoxicosis. One of the many symptoms seen with hyperthyroidism includes menstrual irregularities; however, if TSH levels are corrected, the menstrual regularity will most likely return to its normal state. Serum prolactin levels will remain unchanged in naturally occurring menopause. Levels may be increased if amenorrhea is a consequence of a prolactin-secreting pituitary adenoma. Decreased levels of aldosterone in relation to amenorrhea may be found in patients who are experiencing toxemia of pregnancy, which is not likely in this patient.

Case A 25-year-old G1P1 Caucasian woman recently delivered a healthy boy; she now presents with breast pain and tenderness on the right. Associated symptoms include malaise and fever. The patient has been compliant with all of her prenatal and post-natal obstetrical appointments. No medical complications were incurred during or after the pregnancy. The patient is currently breastfeeding. Physical inspection of the right breast shows focal breast erythema and tenderness. The patient has no known comorbidities, and she denies significant family medical problems. The patient denies palpitations, shortness of breath, or syncope. Pulse is 78 bpm, and respirations are 16 per minute; blood pressure is 102/68 mm Hg, and temperature is 101.2°F. Question What is the most likely diagnosis?

Correct answer: Infection with Staphylococcus aureus Explanation Mastitis is a condition that reflects cellulitis of the periglandular tissue in breastfeeding mothers, typically due to infection with Staphylococcus aureus. Clinical symptoms of mastitis include breast pain, skin redness, fever, and flu-like symptoms. Physical findings include focal breast erythema, swelling, and tenderness, while fluctuance suggests a breast abscess. Diagnosis rests on breast milk cultures and complete blood count analysis. If fluctuance is present on physical examination, then a breast ultrasound can be used to exclude abscess formation. Treatment for mastitis involves continued breastfeeding and antibiotics such as dicloxacillin or erythromycin. The treatment of a breast abscess is incision and drainage. Inflammatory breast carcinoma can clinically resemble unilateral mastitis, but it would not resolve with antibiotic therapy. Additionally, inflammatory breast carcinoma would be extremely unusual in a 25-year-old woman. If the patient's breast erythema, swelling, and tenderness continues despite antibiotic therapy, then a mammogram with an associated punch skin biopsy should be considered to rule out inflammatory breast carcinoma. Congestive heart failure can lead to bilateral breast swelling with increased breast interstitial markings and trabeculations on mammography, but it would be unusual in this patient without a past medical history of congestive heart failure or complaints of palpitations, shortness of breath, or syncope.

Case A 59-year-old woman presents with a firm non-tender mass with a "rock-hard" consistency in her right breast when palpated by her gynecologist. On further examination, the right breast exhibits redness and dimpling of the skin, and her right nipple appears slightly retracted. In addition, she reports a single episode of a bloody nipple discharge. A diagnostic mammogram reveals dense fibroglandular tissue with possible microcalcifications in the area of the palpable lump. A sonographic exam reveals an immobile 10 mm hypoechoic mass. Question What is the most likely diagnosis?

Correct answer: Infiltrating duct carcinoma Explanation Infiltrating (invasive) duct carcinoma is the most common type of breast cancer. It typically does not respect anatomic borders and invades the normal breast at random. Invasion of the fat tissue can be seen on gross examination as stellate white tissue extending into the yellow fat tissue. Invasive ductal carcinomas cause a dense fibroblastic response in the host tissue that gives these tumors a rock-hard consistency. The desmoplastic reaction caused by infiltrating ductal carcinoma resembles an irregular scar that deforms and causes a dimpling of the skin and retraction of the nipple (peau d'orange appearance). This patient has many classic signs of infiltrating ductal carcinoma: firm breast mass, dimpling of the skin, retraction of the nipple (peau d'orange), and a bloody discharge. A biopsy should be performed to confirm the imaging diagnosis. Benign cystic changes are typical of nonproliferative fibrocystic changes. These changes are found in some breasts of women over 45 and are probably related to normal aging of the female breast. The breast mass described in this case does not have the characteristics of a benign cyst. Fibroadenoma is the most common benign breast tumor. It is composed of elongated ducts surrounded by loose fibrous stroma. These well-circumscribed tumors typically present as easily movable spherical masses in breasts of 20- to 35-year-old women. Galactorrhea is a milky breast discharge in non-nursing women due to a prolactinoma (a pituitary adenoma) that causes an abnormal proliferation of lactotrophs and the secretion of excess amounts of prolactin. Prolactin stimulates milk synthesis in lactating women. Galactorrhea does not occur in men because men do not have enough breast tissue. Gynecomastia is the excessive growth of male mammary glands. Physiologic causes include: in the newborn, due to elevated maternal estrogens during pregnancy; during puberty, due to an increased estrogen to androgen ratio; and in old age, due to combined effect of decreasing testosterone and increased estrogen due to peripheral aromatization of androgen to estrogens in adipose tissue.

Case A 20-year-old primigravida presents at 30 weeks of gestation with a 2-day history of a headache, decreased urine output, and facial puffiness. On examination, vitals are as follows: pulse - 90/min, blood pressure - 164/116mmHg, and 166/114mmHg (taken 6 hours apart) and RR- 20/min. There is generalized edema and exaggerated deep tendon reflexes with presence of clonus. Abdominal examination reveals a fundal height corresponding to 30 weeks of gestation and the presence of good fetal heart sounds (FHS). Urine dipstick for protein reveals 3+proteinuria (300mg/dL) on 2 occasions. Question What is the next best step in management of this patient?

Correct answer: Injection of betamethasone Explanation Pre-eclampsia, a pregnancy-induced multi-system disorder occurs in 3-4% of pregnancies in the U.S. and is characterized by development of hypertension (>140/90) as well as proteinuria after 20 weeks of gestation. Severe pre-eclampsia is characterized by systolic pressure of 160 mm Hg and above and a diastolic pressure of 110 mm Hg or higher measured on 2 occasions at least 6 hours apart. Proteinuria often occurs and is characterized by excretion of >5 g/day or dipstick urine testing showing 3+ or higher proteinuria on 2 occasions. Other signs and symptoms include: headache, visual disturbances, abdominal pain, oliguria, pulmonary edema, signs of clonus, impaired liver function, thrombocytopenia and intrauterine growth restriction. The only cure for pre-eclampsia is delivery of the baby and placenta. Babies delivered prematurely, however, are at risk for breathing problems because their lungs may not be fully developed. Women who require pre-term delivery (<34 weeks) as described in this case are usually given two injections of corticosteroid (e.g. betamethasone) 24 hours apart to speed lung development. Since the two injections are given 24 hours apart and full benefit occurs 48 hours after the first injection. Steroid treatment also decreases other potential complications of pre-term delivery such as intraventricular brain hemorrhage. Strict bed rest at home is incorrect. This patient with severe pre-eclampsia must be carefully assessed for signs of imminent eclampsia and continual monitoring in a hospital setting is required. Intravenous phenytoin is incorrect. For the prevention of seizures, magnesium sulfate is the preferred drug. Immediate induction of labor is incorrect. Induction of labor should not take place until after 48 hours of corticosteroid treatment to promote lung maturation for delivery. Women with pre-eclampsia can develop seizures (eclampsia) and are usually treated with magnesium sulfate during labor and 24 hours after delivery to prevent seizures. Immediate Cesarean section is incorrect. Immediate C-section, if required, should not take place until after completion of 48 hours of steroid treatment to enhance lung maturation prior to delivery in women at less than 34 weeks of gestation with pre-eclampsia.

Case A 31-year-old woman presents with her husband to discuss methods of temporary contraception. They explain that they want to have children in the future but would like to wait a few years. They are both in good health. The woman's routine pelvic and physical examinations show a healthy young woman; her past medical history is unremarkable. She is a non-smoker and there is no family history of breast, ovarian, or uterine cancer. Question What would be the most effective means of temporary contraception for this couple?

Correct answer: Intrauterine device Explanation Contraceptives have both a theoretical effectiveness, which is a lab-tested value, and an actual effectiveness, which is how well they perform when used by actual people. In terms of contraceptive methods, intrauterine devices have a 99% theoretical effectiveness, and about a 97% actual effectiveness, making it one of the most effective methods of birth control available. Other benefits include its temporary nature and minimal side effects. Other widely used methods for fertility control include barrier methods. These include diaphragms, sponges, condoms, and the cervical cap. Although the theoretical effectiveness of these methods is high (about 94-98%), their actual effectiveness is probably somewhere in the high 80s due to user error. The morning-after pill (Plan B) is used after unprotected intercourse, but it has been shown to not be as statistically effective as the oral contraceptive pill taken as directed. It is also not intended for repeated use.

Case A 36-year-old G2P1 woman presents for her annual gynecological examination. In addition to routine testing, she is seeking advice on contraception. She currently takes medication as needed for migraine with aura. She has no other known medical conditions. She smokes half a pack of cigarettes daily. She is currently sexually active with her spouse and they remain undecided about their desire for more children in the future. Question What is the best contraceptive recommendation for this patient at this time?

Correct answer: Intrauterine device Explanation Contraindications to estrogen-containing contraceptives include history of certain medical conditions, such as personal or family history of cardiovascular disease (deep vein thrombosis, pulmonary embolus, thrombotic events, myocardial infarction, cerebral vascular accidents), migraines with aura. Patients over 35 who smoke are also at increased the risk for embolic and thrombotic events. This patient should utilize either a progesterone-only or a non-hormonal contraceptive option. An intrauterine device is appropriate, as those currently available are either progesterone-only or non-hormonal. Both combination oral contraceptives and the contraceptive vaginal ring both contain estrogen as well as progesterone. Consequently, they would pose too great a risk for embolic and thrombotic events. Both spousal vasectomy and bilateral tubal ligation are permanent sterilization options, which would not be suitable for a patient undecided about future childbearing plans.

Case You are evaluating a 26-year-old man; he is suspected of being infertile. His past medical history is unremarkable. On examination, you note he is 6'4"; he has mild gynecomastia, sparse body hair, and small soft testes. Question What is the most likely diagnosis?

Correct answer: Klinefelter syndrome Explanation Klinefelter syndrome is a sex chromosome disorder due to an additional X chromosome on a male karyotype: 47, XXY. Patients present either as adolescent boys who fail to progress through puberty or as adult males suspected of having primary infertility. Low androgen levels lead to gynecoid features, including a female pattern of body hair and gynecomastia. The testes are typically small and soft, and libido also may be reduced. Cystic fibrosis can cause male infertility due to absent development of the vas deferens. Tall body habitus, gynecomastia, and small testes are not typical of cystic fibrosis. Recurrent pulmonary infection and intestinal malabsorption are also features of cystic fibrosis; these are lacking in the described case. Turner syndrome is noted by a 45, X karyotype, short stature, webbed neck, and primary amenorrhea in women. Fragile X syndrome is due to a trinucleotide repeat expansion in the fragile X gene in boys/men. It is also characterized by learning difficulties and, in more severe cases, overt cognitive disability. The testes are typically enlarged. Hypospadias is a developmental problem of the penis where the urethral meatus is proximally placed. Most cases are sporadic, and fertility can be affected, but gynecomastia, altered body hair distribution, and small testes are not typically associated with hypospadias.

Question A 25-year-old woman, G0P0, presents to your office with a 1-year history of oligomenorrhea. Her most obvious physical exam findings are hirsutism and obesity, having a calculated BMI of 30. What would you expect to find on her workup results?

Correct answer: LH:FSH ratio of 2:1 or greater Explanation The correct answer is LH:FSH ratio of 2:1 or greater as this is the most positive indicator of polycystic ovarian syndrome. TSH less than 0.4 mIU/mL would indicate the possibility of hyperthyroidism and therefore is incorrect. The only symptom this patient has that would correlate with hyperthyroidism is the oligomenorrhea. A fasting glucose less than 100 mg/dl would indicate insulin sensitivity and a patient suspected of having PCOS usually demonstrates insulin resistance. A free testosterone level less than 0.6ng/mL is incorrect as 25-50% of patients with PCOS have elevations in free testosterone. A prolactin level greater than 40ng/mL indicates hyperprolactinemia, which usually is not a finding in PCOS and is indicative of a separate disorder.

Case A 23-year-old woman presents with a history of chronic pelvic pain. She attained menarche at 13, and she has had regular periods. She has been experiencing severe pain during menses; the pain has been increasing in frequency. She has never had intermenstrual bleeding, and she regularly uses barrier contraception. However, she has refrained from intercourse for over 6 months due to the pain it causes her. Her vital signs and general physical examination appear to be normal. A vaginal exam reveals cervical motion tenderness with reduced mobility and nodularity of the utero-sacral ligaments. Question For a definitive diagnosis, what is the diagnostic test of choice?

Correct answer: Laparoscopy Explanation Laparoscopy is the correct answer. Laparoscopy is considered the gold standard in diagnosis of endometriosis. Laparoscopy allows visualization of the islands of abnormally implanted endometrial tissue. However, visual inspection alone has a high false positive rate. Confirmation requires biopsy of the abnormal areas, with histopathology demonstrating endometrial epithelium, glands, stroma, or hemosiderin-laden macrophages. Hysterosalpingography is incorrect. A visualization of the uterine cavity and tubes by this technique is useful in delineating anatomic defects, such as a bicornuate uterus, as well as mechanical obstruction by adhesion or stenosis as seen post pelvic inflammatory disease. It is not of any use in endometriosis. FSH and LH levels is an incorrect response. They may be used in the workup of infertility, but they provide no information on endometriosis. Ultrasonography is incorrect. In general, imaging techniques are not particularly helpful in diagnosis. It is useful in ruling out other pelvic pathology. Pap smear with endocervical curettage is incorrect. It is mostly a screening procedure for cervical and endocervical pathology. It can help in the identification of endometrial dysplasia and malignancy, but not endometriosis.

Case A 29-year-old woman has Wilson's disease, for which she is taking D-penicillamine. Today, her laboratory findings (including liver function tests) are within normal limits; there are no neurological signs. Her work requires frequent (3 - 5 days per week) travel, including international overnight travel all over the world. She wants contraception that is both effective and convenient in terms of application. Question What is your advice?

Correct answer: Levonorgestrel-releasing Intrauterine system Explanation Only spermicide, barrier contraceptives, and progesterone-only preparations can be considered in Wilson's disease. Levonorgestrel is a 2nd-generation progestin (synthetic progestogen). A levonorgestrel-releasing intrauterine system causes thickening of cervical mucus, thereby reducing sperm motility and penetration; over time it decreases proliferation of the endometrium. The local inflammatory response could be toxic to sperm, thus providing an additional contraceptive effect. Combination oral contraceptive pills contain both estrogen and progestins. Estrogen-containing contraceptives are not recommended since estrogen may interfere with copper excretion. The progestogen-only pill is efficient; however, it must be taken at or around the same time every day (within 3 - 12 hours). Therefore, the pill may be inconvenient for a woman with a job that requires international travel all over the world. Nonhormonal copper intrauterine devices should be avoided in a disease that has the main characteristic of copper accumulation in the body. Copper IUDs primarily work by disrupting sperm mobility and damaging sperm. Contraceptive patches also contain estrogen; therefore, they should be avoided in Wilson's disease.

Question What hormone is responsible for ovulation?

Correct answer: Luteinizing hormone Explanation Luteinizing hormone is the hormone responsible for initiation of ovulation. Follicle-stimulating hormone is responsible for follicle maturation. Estrogen increases the number of luteinizing hormone (LH) receptors, which also leads to deregulation of LH. Ultimately, a rapid rise in LH (LH surge) occurs approximately 24-36 hours prior to ovulation. Estrogen is produced by the conversion of testosterone to estradiol by aromatase in the granulosa cells.

Question Examination of a newborn infant revealed the presence of hairy patch present on the lower back. Neural tube defect is most commonly associated with which of the following?

Correct answer: Maternal folic acid deficiency in the first trimester Explanation Neural tube defects are most commonly associated with maternal folic acid deficiency in the first trimester. Hence, 400mg of folic acid is recommended when attempting to get pregnant and during early pregnancy to prevent neural tube defects. Neural tube defects (NTDs) result due to the failure of closure of neural tube during the 4th week of embryogenesis. NTDs include anencephaly, spina bifida, and encephalocele and may also cause severe disabling birth defects. Congenital malformations of the spinal cord are the result of improper closure of the caudal neuropore. Malformations of the neural tube involving the spinal cord and vertebral arches are termed as spina bifida. The non-fusion of the spine can occur in varying degrees of severity ranging from a lack of bone fusion to protrusion of the meninges with or without nerve involvement. Severe type of spina bifida (meningomyelocele) involves protrusion of the spinal cord along with the meninges through a defect in the vertebral arch. The clinical manifestation vary depending on the level of the lesion; nerve involvement results in paralysis of the legs, urinary and fecal incontinence, skin anesthesia, and abnormalities of the pelvis, knees, and feet. Display of a hairy patch, a dimple, or hyperpigmentation may also be present. Early prenatal diagnosis by ultrasound and alpha-fetoprotein level estimation is essential for proper management. Though maternal folic acid deficiency plays a pivotal role in NTDs, the etiology of neural tube defect remains multifactorial. Maternal infections, radiation, and teratogen exposure predominantly in the early pregnancy are associated with NTDs. Increased maternal age is associated with Down syndrome.

Case 6 hours after delivery, a 25-year-old primigravida presents with fullness and pain in the lower abdomen. Her pregnancy was normal; labor was induced with misoprostol and progressed normally during the first stage; episiotomy was performed 2 hours after second stage had started; and the third stage of labor was normal. Baby weighted 3,800 gr, and APGAR score was 9, 9, and 10 in 5 minute intervals. Postpartum visually estimated blood loss is about 550 ml (normally ≥500 ml in the first 24 hours after delivery). She did not void after the delivery. On examination, her vitals are stable and her bladder is overdistended. Question What is the best next step in the management of this patient in addition to catheterization?

Correct answer: Measurement of urine volume Explanation Your patient has an overt urinary retention because of the inability to void spontaneously within 6 hours of vaginal birth and the physical finding of overdistended bladder. It is unlikely that she will void now if she did not void within first 4-6 hours after delivery. Postpartum urinary retention occurs in 10-15 % of women and is probably multifactorial. Risk factors in this case are the primiparity, prolonged and induced labor, and episiotomy. She should be catheterized, and the urine volume measured. Urine volume will guide further management: if more than 200 mL is obtained, the catheter should be left for another 24 hours. If less than 200 mL, the catheter may be removed and the bladder checked again. Antibiotics are not indicated because she has no signs of infection. Oxytocin is not indicated in a patient with stable vital signs when there are no signs and symptoms of the atony of the uterus. Besides, oxytocin may cause rapid bladder filling, thus worsening the condition. Visual estimation of the blood loss is inaccurate. Uterine massage may be considered in the cases of postpartum hemorrhage. Visual estimation of the blood loss is inaccurate. Urodynamics studies evaluate the functional status of the lower urinary tract, usually in the cases of urinary incontinence. They are not indicated in this case.

Case A 54-year-old woman presents for her annual pelvic examination. She states her last menstrual period was over 12 months ago, adding that the last few occurrences of menses were extremely irregular. The patient also describes having multiple daily episodes of severe, intense heat in her face and trunk that is accompanied by sweating. She further states that these "heat episodes" have been occurring 4 - 6 times a day for the last 4 weeks and are interfering with her everyday activities in addition to her sleep. She notes no other symptoms at this time. She has received her annual pelvic examinations yearly, as well as clinical breast exams and mammograms, without any significant findings. Her past medical history is negative for cardiovascular disease, blood clots, or breast cancer. During the pelvic examination, you note obvious vaginal thinning and excessive dryness, as well as apparent vaginal wall atrophy. Question What pharmacologic intervention can be prescribed to assist in minimizing the patient's symptoms?

Correct answer: Menopausal hormone therapy Explanation The patient in the scenario above is most likely having signs and symptoms due to menopause. Menopause in the most pure sense of explanation is a cessation of menstruation from either natural aging (usually amenorrhea for at least 6 months) or an external cause (surgical). There is usually a 1-3 year time period that women will typically adjust physiologically to the diminished hormonal and menstrual actions and the effects this has on their body. These effects may include hot flashes, night sweats, vaginal dryness, and in the later stages, osteoporosis. The average age in Western societies to experience menopause is around 51 years old. The patient described in the scenario above is suffering from moderate-to-severe symptoms of the physiological menopausal transition. Given that this patient does not have any obvious risk factors, Menopausal hormone therapy (MHT) is the best pharmacological option at this time. MHT at one time was referred to as hormone replacement therapy. Benefits of the use of MHT are found to present with reducing hot flashes, night sweats, and other related issues like poor sleeping and irritability. MHT will also treat vaginal symptoms related to menopause such as vaginal dryness and discomfort. Another benefit to its use is slowing of bone loss and easing mood swings or depressive symptoms. It is extremely critical the healthcare provider stress to the patient that if MHT is started, its use is only recommended for a short period of time and at the lowest effective dose. Low-dose oral contraceptives or birth control pills may be an option for menopausal symptom relief only if the patient is perimenopausal. Since this patient has not had menses cycle in over 8 months, she is categorized as being menopausal, so this treatment would not be appropriate. Phytoestrogens are substances found in plants, such as soy and red clover extracts that bind to estrogen receptors. They do not appear to significantly improve menopausal hot flashes and would not be the best option for this patient. Progestin-releasing intrauterine devices (IUDs) would not be an appropriate treatment option as these are only used for patients using MHT to reduce the incidence of dysfunctional uterine bleeding and endometrial carcinoma. Testosterone replacement therapy in women is reserved for those patients suffering from mainly hypoactive sexual desire disorder (HSDD); other signs and symptoms that may benefit from testosterone replacement therapy include hot flashes, pubic atrophy, muscle atrophy, and osteoporosis; however, this is not the correct treatment option for this patient.

Case A 50-year-old woman presents for her annual pelvic examination; her previous examination took place about 18 months ago. She tells you that her last menstrual period was over 12 months ago; the last few occurrences of menses were extremely irregular. The patient also describes having multiple daily episodes of severe, intense heat in her face and trunk that is accompanied by sweating. She further states that these "heat episodes" have been occurring 4 - 6 times a day for the last 4 weeks. They have been interfering with her everyday activities, as well as her sleep. She has no other complaints at this time. She has received her annual pelvic examinations yearly as well as clinical breast exams and mammograms; there have been no significant findings. During the pelvic examination, you note obvious vaginal thinning and excessive dryness; there is also apparent vaginal wall atrophy. Question What pharmacologic intervention can be prescribed in order to assist in ameliorating the patient's symptoms?

Correct answer: Menopausal hormone therapy Explanation This patient scenario above is most likely the result of menopause. Menopause in the most pure sense of explanation is a cessation of menstruation from either natural aging (usually amenorrhea for at least 6 months) or an external cause (surgical). There is usually a 1 -3 year time period that women will typically adjust physiologically to the diminished hormonal and menstrual actions and the effects this has on their body. These effects may include hot flashes, night sweats, vaginal dryness, and in the later stages, osteoporosis. In western societies, the average age at which women achieve menopause is about 51 years old. The patient described in the scenario above is suffering from moderate-to-severe symptoms of the physiological menopausal transition. Given that this patient does not have any obvious risk factors, menopausal hormone therapy (MHT) is the best pharmacological option at this time. At one time, MHT was referred to hormone replacement therapy. Benefits of the use of MHT include reducing hot flashes, night sweats, and other related issues, such as poor sleeping and irritability. MHT will also treat vaginal symptoms related to menopause, such as vaginal dryness and discomfort. Another benefit to its use is slowing of bone loss and easing mood swings or depressive symptoms. An extremely critical component that must be remembered by the healthcare provider and the patient if MHT is started is that the use of MHT is only recommended for a short period of time and at the lowest effective dose. Low-dose oral contraceptives (or birth control pills) may be an option for menopausal symptom relief only if the patient is perimenopausal. Because this patient has not had her menstrual period in over 8 months, she is categorized as being menopausal, so this treatment would not be appropriate. Phytoestrogens are substances found in plants, such as soy and red clover extracts, that bind to estrogen receptors. They do not appear to significantly improve menopausal hot flashes, and they would not be the best option for this patient. Progestin-releasing intrauterine devices (IUDs) would not be an appropriate treatment option as this is only used for patients using MHT to reduce the incidence or dysfunctional uterine bleeding and endometrial carcinoma. Testosterone replacement therapy in women is reserved for those patients suffering from mainly hypoactive sexual desire disorder (HSDD); other signs and symptoms that may benefit from testosterone replacement therapy include hot flushes, pubic atrophy, muscle atrophy, and osteoporosis; however, this is not the correct treatment option for this patient.

Case A 53-year-old woman presents for her annual physical examination. She states her last menstrual period was over 12 months ago, and she adds that the last few occurrences of menses were extremely irregular. The patient also describes having sensations of intense heat in her face and trunk; the sensations are accompanied by sweating. She has also been extremely bothered by increased issues with memory, concentration, low energy, severe insomnia, weight gain, and decreased libido. She is not experiencing any other symptoms at this time. She has been receiving her annual pap and pelvic examination regularly, and she has also been receiving her clinical breast exams; there have not been any issues. Question What is the most likely diagnosis?

Correct answer: Menopause Explanation This patient scenario is most likely caused by menopause, specificallylater menopausal transition, according the 'The Stages of Reproductive Aging Workshop (STRAW) Guidelines'. Menopause is the cessation of menstruation from either natural aging or an external cause (surgical). Usually, several years are necessary for women to adjust physiologically to the diminished hormonal and menstrual actions as well as their effects on the body. The effects felt during this adjustment time can vary tremendously; they may include hot flashes, night sweats, heart palpitations, vaginal dryness, and in the later stages, osteoporosis. Signs and symptoms also may be seen that would make a healthcare provider consider the diagnosis of depression secondary to the menopausal phases: depressed mood, anxiety, poor memory and concentration, fatigue, headache, insomnia. During the perimenopausal time period women are 2 - 4 times more likely to experience depression even without a prior relevant history. The large fluctuation of hormonal levels is what leads to these severe changes in symptoms that tend to last from 5 - 7 years. The average age in Western societies to experience menopause is around 51 years old. Ovarian failure is referred to as premature menopause; it is the cessation of menses before the age of 40, which does not coincide with the patient case. Pregnancy is a possibility in this case, but it is not likely. Cervical cancer is not as likely a diagnosis; this patient has cooperatively obtained her yearly pelvic exams, there were no significant abnormalities noted as a result of the visits. Polycystic ovarian syndrome is not as likely to be the main diagnosis in the patient described. She has never had this diagnosis before; also, it is typically a syndrome found in women of childbearing years who may be having infertility issues due to the hormonal dysfunction.

Question The pregnant mother of a 2-year-old patient is concerned about exposure to environmental neurotoxins affecting her developing fetus. She asks you questions about whether the placenta would block any transfer of toxins to a growing fetus. Which environmental neurotoxin transfer to the fetal system is actually enhanced by the placenta?

Correct answer: Mercury Explanation Methyl mercury is absorbed almost completely and crosses the placenta easily and then becomes stored in the fetus. Fetal blood concentrations exceed maternal concentrations by 50-100%. High doses can result in intellectual disability, spastic paralysis, and death. Low doses are linked to deficits in neuromotor performance, cognition, memory, and language. The placenta effectively blocks the transfer of cadmium. Studies have shown cadmium concentrations in umbilical cord blood to be significantly lower than that in maternal blood and placental cadmium concentration to be highest. Polychlorinated biphenyls (PCP) have contaminated sport fish, particularly bottom-feeding species from water contaminated with PCPs. Also during the 1940s and 1950s the inside of concrete silos on many farms in the Midwest were coated with sealants containing PCPs that over time have peeled off and become mixed with silage to feed beef and dairy cattle. Incinerators and other PCP-disposal facilities or hazardous waste sites are other sources of PCP exposure. PCPs are synthetic hydrocarbons that are lipophilic, have a long half-life, and can cross the placenta easily. Fetuses and neonates are more sensitive to PCPs because the hepatic microsomal enzyme system that facilitates metabolism and excretion are not fully functional. Insecticides (or pesticides) appear to cross the placenta relatively easily, being lipophilic and having a low molecular weight. Residue levels of DDT and its metabolites were detected in maternal blood, placenta, and umbilical cord blood of mother/child pair studies. A correlation exists between pesticide concentration and age, dietary habits, and area of residence of pregnant women. Environmental tobacco smoke (ETS) exposure during fetal development is one of the most ubiquitous and hazardous of environmental exposures. Placental vascular resistance is often increased when women smoke during pregnancy and there are subsequent alterations of protein metabolism and enzyme activity in cord blood. Maternal uterine blood flow is lowered with reduced flow of oxygen from the uterus to the placenta. There are increased levels of carboxyhemoglobin in both maternal and fetal blood when the mother smokes, which can lead to fetal hypoxia with consequent chronic fetal hypoxic stress.

Case A 33-year-old G5P4 woman presents to her gynecologist. She has been using condoms and would like to discuss alternative birth control options. She has regular periods and is not currently trying to get pregnant. She is sexually active with one partner. Past medical history includes asthma, deep vein thrombosis during her first pregnancy, and a C-section for her fourth pregnancy. She has no known drug allergies. Question Of the following, which is the best birth control option for this patient?

Correct answer: Mirena (levonorgestrel) IUD Explanation Mirena (levonorgestrel) IUD is the most appropriate choice out of the above options. The patient has history of DVT, which means that estrogen-containing therapy is contraindicated due to its elevated risk of thromboembolism. Mirena IUD contains progesterone and therefore can be safely used by the patient. Both combination estrogen-progestin pill and Ortho Evra (ethinyl estradiol/norelgestromin) patch contain estrogen. Estrogen-containing products are contraindicated, given the patient's history of DVT. Tubal ligation is incorrect. While the patient is not currently trying to get pregnant, the history does not mention if the patient desires future pregnancy. Total abdominal hysterectomy is not a recommended form of birth control.

Case A lactating woman with a 5-month-old baby presents with increasing unilateral breast discomfort over the past few days. A home monitoring of her temperature revealed a reading of 100.8°F (38.2°C). She has been unable to feed her child from the affected breast for the past 24 hours. Examination reveals a breast with a fluctuant mass. Refer to the image. Question What is the next best step in the management of this patient?

Correct answer: Needle aspiration Explanation A lactational breast abscess is a relatively uncommon problem in feeding mothers, with an incidence of 0.1% (up to 3% in women with antibiotic-treated mastitis). Organisms ascend from the surface of the nipple and colonize the breast lobule. Milk serves as an excellent culture medium for the bacteria. Stagnation of milk due to cessation of feeding aids this process. What begins as a cellulitis/mastitis can thus evolve to form an abscess. The treatment of an abscess always involves drainage, regardless of its location; release of purulent material is key to resolution. With breast abscesses, needle aspiration under ultrasound guidance is the preferred initial intervention (and would then be followed by a course of oral dicloxacillin); however, if the overlying skin is compromised, or if needle aspiration and oral antibiotics are not sufficient for resolution, surgical incision and drainage may be recommended. Flucloxacillin is the antibiotic of choice for lactational abscess because it covers the Staphylococcus species, which are the usual culprits. However, antibiotics are only minimally effective without drainage of pus. At an early stage of development (when it is still considered mastitis), oral antibiotics alone may suffice; however, this would not be sufficient for this patient. Warm compresses with expression of milk is an adjunct to medical treatment, and it prevents the stagnation of milk in the breast.

Case A 22-year-old woman presents for her annual well-woman examination. She was seen last year and she had her first pap smear done, which showed no evidence of intraepithelial neoplasia. She is sexually active with 1 partner currently and a history of 2 partners total. She reports using condoms intermittently. Question According to current guidelines, what, if any, type of cervical cancer screening should be done on this patient at this visit?

Correct answer: No cervical cancer screening is recommended Explanation According to the 2012 ASCCP (American Society for Colposcopy and Cervical Pathology) guidelines, women ages 21-24 should be screened conservatively for cervical cancer. Following a normal cervical cancer screening result, this screening could consist of a pap smear alone or a pap smear with reflex high-risk HPV testing every 3 years. According to these guidelines, the patient in the scenario should have her next screening done at age 24.

Case A 45-year-old Caucasian woman presents with a 2-month history of a lump in her right breast. She has had no children. Her menstrual periods began at age 15 and are still regular. She is a successful businessperson who does not smoke cigarettes and drinks 6 cups of coffee per day. She gets 30 minutes of physical activity at least 5 times weekly. On examination, she has a nontender lump in the upper outer quadrant of her right breast. Several matted right axillary lymph nodes are also palpable. Question Given her history, what is her greatest risk factor for developing breast cancer?

Correct answer: Nulliparity Explanation The correct response is nulliparity. The clinical features are suggestive of breast cancer. Although it occurs mainly in women, a small percentage develops in men. Breast cancer has no single cause, but multiple risk factors have been identified. These include: Family history of premenopausal breast cancer in a first-degree relative (e.g., mother, daughter, sister) Having the first full-term pregnancy after age 30 Prior personal history of breast cancer Chest exposure to high dose of ionizing radiation Nulliparity Early menarche at or before age 12 Late menopause after age 50 Taking oral contraceptives (birth control pills) Lack of physical activity Current or recent use of oral contraceptive pills (for women who stopped using oral contraceptive pills 10+ years ago, there is no apparent increase in risk).

Case You are evaluating a 54-year-old postmenopausal woman. She initially presented 2 weeks prior due to vaginal spotting, for which you elected to perform an endometrial biopsy. The pathology indicated endometrial cancer. Review of the patient's history from the initial visit reveals the following information: She is obese and has a history of diabetes, for which she takes metformin. Her menses began at age 15 years, and she believes her LMP to have been around age 42. She has had 8 pregnancies, 6 live births, and 9 lifetime sexual partners. She has no history of sexually transmitted diseases. Previous methods of contraception include oral contraceptives briefly in her 20s and depot medroxyprogesterone acetate for 5 years in her 30s. She discontinued use of contraceptives in her mid-30s, and she has rarely been sexually active since then. She is a non-smoker and non-drinker; she denies illicit drug use. Question What risk for endometrial cancer did this patient's history reveal?

Correct answer: Obesity Explanation Obesity is associated with a 2-5 times increased risk for endometrial cancer. Obesity plus diabetes further increases the risk. This patient started menses at age 15, and the late range of normal menarche can be between 15 and 16 years of age. Early menarche (before the age of 12 years) is considered a risk factor for endometrial cancer, not delayed menarche. A grand multipara is a woman who has delivered 5 or more viable babies. Infertility and nulliparity are considered risk factors for endometrial cancer, so this patient's history of multiple pregnancies without infertility is not a risk factor. In fact, multiparity is considered a protective factor against endometrial cancer. The patient's 2-year history of depot medroxyprogesterone acetate injections for birth control is not considered a risk factor for endometrial cancer. Long-term use of estrogens without progestin is a risk factor for endometrial cancer. The unopposed estrogen effect can contribute to endometrial hyperplasia and eventually endometrial cancer. The number of sexual partners is not significantly associated with endometrial cancer risk. The brief use of oral contraceptives is not a risk factor. Use of oral contraceptives is considered protective against endometrial cancer.

Question A 65-year-old woman presents to your office complaining of bloating, early satiety, changes in her bowel habits and an unintentional 10 pound weight gain over 3 months. Upon further questioning you discover that she never had children and her sister passed away 20 years ago from some sort of cancer in her belly. On her physical exam an adnexal mass is felt. What is the most appropriate next step to order to further evaluate this patient?

Correct answer: Obtain a transvaginal ultrasound Explanation The correct answer is transvaginal ultrasound because this should be ordered next based on history and physical findings (1). It also would help to distinguish the characteristics of the adnexal mass, cystic formations, septations within the mass and the presence of ascites (2). It should be felt that the most likely diagnosis for this patient is ovarian cancer. CA125 is not correct. While this tumor marker is used in the setting of a patient with ovarian cancer, which should be suspected in this patient, it is not diagnostic. It is only a piece of the puzzle and is more warranted before and during the use of neoadjuvant or adjuvant chemotherapy. CA125 can be elevated in a number of disease processes to include colon, breast, hepatic, and uterine. It can also be within normal limits (<35 IU/mL) in early stages of ovarian cancer (1, 2). Colonoscopy is not correct as this patient is more likely to have ovarian cancer given that an adnexal mass was felt on exam. This patient could be thought to have a colonic malignancy or metastatic involvement of the colon in that her bowel habits have been noted to change and that her sister had some sort of cancer in her belly. Genetic testing is not correct. This is something that could be pursued after a proper diagnosis is made in this patient. Mammogram is not correct. Although this study can be obtained in the workup to reveal the possibility of a metastatic breast cancer or a simultaneous breast cancer it should not be the next step (1).

A 48-year-old Caucasian woman presents due to feeling like she is losing her mind. She wants some tests done. Upon further questioning, she reports she is having multiple episodes daily in which she suddenly becomes very hot, flushed, and diaphoretic. These episodes last about 1 minute before resolving. She has not measured a fever. The patient reports that these episodes occur during the day and at night, during which she awakes drenched with sweat. As a result, her sleep has been poor, and she feels fatigued and irritable at both work and home. She has noticed these symptoms for about the last 2 months, and they seem to be increasing in severity. The patient has not had a period for 3 months; she recently did a home pregnancy test, which was negative. Prior to that, she had regular menses. This patient denies weight changes, palpitations, cold intolerance, bowel changes, as well as changes in her nails, skin, and hair. Although she admits irritability, she denies anxiety, depressed mood, and suicidal ideation. Her family history is remarkable for diabetes in her maternal grandfather and hypertension in her father. She is a G4P3Ab1. She denies any major psychosocial stressors recently. She drinks alcohol rarely, and she denies use of other drugs. Vitals and a urine specimen for hCG were obtained prior to the physical exam. Weight 168 lb Height 65" Pulse 72 Blood pressure 120/82 mm Hg Temperature 98.2°F Urine hCG Negative Complete screening physical exam is normal, with normal sexual development and absence of hirsutism and acne. Question Based on this patient's history and physical, what is the most reasonable management for this patient?

Correct answer: Offer blood tests and provide education/prescription for hormone therapy. Explanation Based on the history and physical, her diagnosis is perimenopause, with a classic history of vasomotor symptoms (hot flashes/flushes and night sweats). Perimenopause is defined as the time prior to menopause in which the woman's ovarian hormone output begins to decline, often beginning in the mid-40s. Menopause is defined at the point in time in which the patient has been amenorrheic for 12 months. Postmenopause occurs after that. The average age for menopause is just over 51. With a negative pregnancy test and normal physical exam at age 48 with this history, the diagnosis of perimenopause can actually be made without diagnostic studies. If tests were done, an elevated follicle-stimulating hormone is the best indicator of declining ovarian function. The most effective treatment for vasomotor symptoms is hormone therapy, and the patient's history gives no contraindications for its use. Because this patient presents to the clinic desiring tests, the clinician should offer blood tests and provide education/a prescription for hormone therapy. Raloxifene (Evista) is a selective estrogen receptor modulator (SERM), which is used in preventing and treating osteoporosis and prevention of breast cancer. It is not useful for vasomotor symptoms, and increased hot flashes are a common side effect of this medication. This would not be an appropriate medication choice for this patient. As mentioned above, blood tests could be offered but are not necessary. The cancer antigen 125 (CA-125) is a marker primarily associated with ovarian cancer. It is most useful in monitoring patients with a history of ovarian cancer for recurrence and in the evaluation of suspicious solid ovarian masses. This patient's presentation does not support a solid reason for ordering a CA-125. A pelvic ultrasound is useful in evaluating many gynecologic anatomic abnormalities, but it will not demonstrate function. In this patient's case, she presents with declining ovarian function due to perimenopause, and this is undetectable on ultrasound. Her physical exam was normal. If she had presented with pelvic complaints, an abnormal pelvic exam, or irregular or heavy menstrual bleeding, an ultrasound could have been useful. Although thyroid disorders can cause menstrual cycle changes and fatigue, ordering a thyroid ultrasound is not indicated in this case. A more appropriate way to rule in or rule out thyroid disease is with appropriate history, physical exam, and a serum thyroid-stimulating hormone level. This patient denies other symptoms of hypothyroidism (weight gain, dermatologic changes, cold intolerance, constipation, and depressed mood). Hypothyroidism is not associated with hot flashes and night sweats.

Case A 45-year-old woman is status-post (s/p) total abdominal hysterectomy and bilateral salpingo-oophorectomy (TAH-BSO) secondary to stage I ovarian cancer. She has completed a course of cisplatin and cyclophosphamide, and she presents for follow-up. Serial CA-125 has returned to normal. Question What is the most common site of metastasis for this tumor?

Correct answer: Omentum Explanation The most common route for the spread of ovarian carcinomas is transcoelomic (along with the normal flow of peritoneal fluid) in which case common sites for metastasis include the pericolic gutters, the omentum, and small and large bowel. Only rarely do ovarian tumors spread via hematogenous dissemination to the lung, liver, or bone, making follow-up imaging of these areas unnecessary. The brain is not considered a site for ovarian metastasis.

Case A 45-year-old woman presents with a 3-day history of persistent nipple discharge. She is essentially healthy with an insignificant past medical history. She quit her job to stay home with her young children in the past year; she has noted a weight gain of 15 lb, which she attributes to being at home more, leading to increased snacking as well as less activity. She also says her periods have spaced out, occurring anytime from 28-53 days. They still last about 2-3 days, as they previously had. She denies hot flashes and the possibility of pregnancy; her husband had a vasectomy. Her review of systems is otherwise negative. She takes vitamin E every day, but she takes no other medications or herbs; she denies the use of illicit drugs. On physical exam, you note a healthy-looking woman. Her breast exam is negative for lumps, dimpling, and nipple retraction. You are able to express some clear fluid from the left nipple. It seems serous. Her axillary exam is negative for any enlarged lymph nodes; the rest of her exam is normal. Question What is the next step in the management of this patient?

Correct answer: Order a mammogram. Explanation A mammogram should be ordered. The patient is a 45-year-old woman with spontaneous serous discharge. Nipple discharge is a common breast complaint and is usually associated with benign etiology. Because most nipple discharge is a result of benign processes, less invasive, non-surgical diagnostic modalities have been explored to alleviate the need for surgical intervention. Most of the patients with nipple discharge have benign disease, such as intraductal papilloma, papillomatosis, duct ectasia, or fibrocystic condition. Studies have shown that approximately 11% of patients will have an underlying carcinoma. To be significant, nipple discharge should be spontaneous, persistent, and non-lactational. Persistent clear or bloody nipple discharge merits, at the very least, diagnostic breast imaging. All nipple discharges associated with a breast lump must be surgically evaluated. The incidence of associated cancer is much higher when the discharge is serosanguineous or sanguineous. Mammography has been utilized for evaluation of obvious abnormality to guide surgical intervention. The sensitivity of mammography for detecting malignant ductal pathology is 57% and the positive predictive value is 16.7%. It should be the initial test in patients with clinically significant nipple discharge and in women over 40. Cytological analysis of the discharge fluid has traditionally shown high false-negative rates, and it is not revealing in localizing the lesion. Hemoccult testing has been shown to be useful for determining the presence of blood, but its use in predicting histology is questionable. For the patient in this case, all the tests noted could be indicated. An FSH can be used to check for menopausal status, and a prolactin level could be done to check for abnormal levels. A TSH to check for hypothyroidism could also be done; decreased thyroid hormones are associated with galactorrhea. Hypothyroidism would also explain the menstrual irregularity. Mammography should be conducted initially; later, hormonal assay and a cytological examination can be carried out. In summary, given that this patient presented with spontaneous persistent discharge, the most appropriate initial approach would be to perform a mammography.

Case A 32-year-old healthy woman is asking for advice about contraception. She wants contraception that is both effective and convenient in terms of application. She thinks that oral contraceptive pills will work the best, and she is interested in additional benefits that the pill can provide (particularly cancer protection). Question You inform her that, in her case, oral contraceptives may reduce the risk of death from what condition?

Correct answer: Ovarian cancer Explanation Ovarian cancers are often diagnosed in the advanced stages; this is the reason why ovarian cancer accounts for more deaths than any other gynecological cancer. The risk of ovarian cancer is reduced by 30% with pill use for <4 years, by 60% with use for 5 - 11 years, and by 80% after 12 or more years of use. Therefore, protection against ovarian cancer reduces the risk of gynecological cancer death. The relationship between breast cancer and OCP is controversial. Some research finds no increased risk of breast cancer in current or former users of OCPs aged 35 - 64, in women with a family history of breast cancer, and in women who started oral contraceptive use at a young age. Other research has found a very low risk of breast cancer in women of childbearing age (relative risk of 1.1 - 1.2, depending on other variables). This small increase is not substantially affected by duration of use, dose, type of component, age at 1st use, or parity. Even 10 years after the discontinuation of oral contraceptive use, no difference in breast cancer incidence between past users and those who have never used oral contraceptives has been found. Breast cancers diagnosed in women who have used oral contraceptives are more likely to be localized to the breast; therefore, overall, there is no significant difference in the cumulative risk of breast cancer between those who have used oral contraceptives and those who have never used them. There also is no evidence that OCP use offers protection. There is no evidence that oral contraceptive pills have any effect on colon cancer. Combined OCP use does not decrease the risk of cervical cancer; it increases it by about 2-fold, but only in long-term users (>5 years) with persistent human papilloma virus infection. Combined OCP use does not decrease the risk of hepatic cancer. Rarely, it has been associated with the development of benign or malignant hepatic tumors (adenoma and hepatocellular carcinoma). The risk increases with higher dosage, longer duration of use, and older age. Current estimates indicate there is about 2 times the risk of liver cancer after 4 - 8 years of use; therefore, protection by OCPs against ovarian cancer reduces the risk of gynecological cancer death.

Case A 35-year-old woman presents due to abdominal fullness. She states that she began noticing her symptoms about 6 months ago. She denies pain, menstrual, bowel, or bladder changes. Her medical history is negative, and she has no other complaints. She noted no noticeable change in her symptoms with menses. A serum pregnancy test was performed and the results were negative. A CBC, WBC, and urinalysis were within normal limits. A pelvic exam is performed as well as an ultrasound. A firm movable mass was felt in the right adnexal region. The rest of the exam was normal. The ultrasound revealed cystic pelvic mass approximately 4 cm in diameter with regular borders. No other abnormalities were found. Question What is the most likely diagnosis?

Correct answer: Ovarian cyst Explanation The patient has an ovarian cyst. Ovarian cysts are usually asymptomatic. The other conditions are more likely to present with pain. This patient's only complaint is abdominal enlargement. There is also no sign of infection such as an elevated WBC. Ovarian cysts are fluid filled cavities within the ovary. Fluid collects each month forming a follicle around the ovary. Once ovulation occurs, the follicle becomes a corpus luteum and this produces progesterone. The cyst is formed either by the follicle or the corpus luteum. Many women have ovarian cysts that are harmless and asymptomatic; however, symptoms such as pelvic pain or pressure, menstrual irregularities, bowel or bladder changes, or enlargement of the abdomen may occur. Complications include rupture, dysmenorrhea, adnexal torsion, peritonitis, and malignancy. Ovarian cysts are normally visualized by ultrasound. An MRI is more sensitive but less specific than ultrasound. CT scans are less accurate than ultrasound. Regardless of the imaging method, a pelvic exam should be performed. Some causes of ovarian cysts include hypothyroidism, early menarche, and the use of Tamoxifen. Treatment of cystic ovary is: Monitoring with the ultrasound on a regular basis Waiting and watching if the cyst is less than 2 cm in diameter and is found to be filled with fluid Oral contraceptive pills may reduce the development of new cysts. Surgery may be necessary for large cysts which cause pain or other symptoms. An oophorectomy may be necessary. In some cases, a cystectomy may be performed to remove the cyst, leaving the ovary intact. Pain and other signs of inflammation such as fever, etc are the most common presenting symptoms of salpingitis and pelvic inflammatory disease as these are inflammatory processes, and these symptoms are not present in this case. Endometriosis presents as chronic pelvic pain unresponsive to treatment, and also many times with progressive dysmenorrhea, dyspareunia, and gastrointestinal and urologic complaints, which are not present here.

ase A previously healthy 30-year-old woman, G1 P1, presents with amenorrhea, weight loss, shortness of breath, and increasing abdominal circumference. Menstrual irregularity started about 1 year ago, and her last menstruation was 3 months ago. She has lost around 5 kilos over the last few months, but at the same time, her waist has enlarged. Shortness of breath started a week ago, and it is worse when she is lying down; she now sleeps using at least 2 pillows. An examination of the lungs shows dullness to percussion, decreased tactile fremitus, and inaudible breath sounds bilaterally. Physical examination of the abdomen shows bulging of the flanks in the reclining position, and there is a difference in percussion in the flanks that shifts when she is turned on the side. Pelvic examination shows a normal uterus and left adnexa; the right adnexum appears enlarged, smooth, and tender. Complete blood count and chemistry is normal, and a chest X-ray confirms the presence of pleural effusion on the right side. Fluid obtained from peritoneal cavity shows heterogeneously bloody content that clots; the leukocyte number is normal, and serum-ascites albumin gradient (SAAG) is 0.8 g/dL (low). The sample is negative of malignancy. Question What is the most likely cause of her symptoms?

Correct answer: Ovarian fibroma Explanation Ascites in a young woman is most often caused by liver, pancreatic or renal disease, pelvic or abdominal tumours, and infection. The SAAG less than 1.1 g/dL points to the non-portal hypertensive ascites. This case is Meigs syndrome; it is characterized by the triad of benign ovarian tumor (usually fibroma), ascites, and pleural effusion that resolves after resection of the tumor. Menstrual irregularity and amenorrhea are probably caused by a fibrous growth in this woman's ovary, which causes abnormal levels of sex hormone production. An extrauterine pregnancy will not present with the ascites and pleural effusion. The clinical presentation of ectopic pregnancy occurs at approximately 2 months after the last normal menstrual period; it typically causes vaginal bleeding and abdominal pain. Acute heart failure may present with shortness of breath, and ascites and ovarian tumor may be the accidental finding; however, SAAG >1.1 g/dL rules out non-portal hypertensive ascites. The SAAG is calculated by subtracting simultaneously obtained the ascitic fluid albumin value from the serum albumin value, and it correlates directly with portal pressure. Therefore, it is the best single test for classifying the ascites into the ascites caused by portal hypertension (SAAG >1.1 g/dL) and non-portal hypertension (SAAG < 1.1 g/dL). Its accuracy is about 97%. Acute liver failure is not likely when there is a normal CBC and chemistry panel with non-portal hypertensive SAAG. Visible bloody ascites requires the presence of more than 20,000 red blood cells/µL. It is usually a consequence of either malignancy or a traumatic tap. A traumatic tap results in a heterogeneously bloody fluid that will clot. Therefore, a traumatic tap is most probably the case of bloody peritoneal fluid.

Case Your patient in the labor and delivery unit is a 26-year-old primigravida; she is at 36 weeks of gestation. She is experiencing fever and abdominal tenderness. Her pregnancy has been normal; however, 2 days ago she noticed clear fluid vaginal leaking. There are no changes in fetal movements, and there has been no vaginal bleeding. Today her temperature is 38.5 C, blood pressure is 120/80 mm Hg, heart rate is 92/min, and respirations are 18/min. Fetal heart rate is 160/min (tachycardia), with good variability. Contractions are registered every 10 minutes. On examination her abdomen is diffusely tender, the cervix is closed, and there is turbid vaginal pooling that shows an elevated pH and a ferning pattern. Question After administering antibiotics, what will be your next step in the management of this patient?

Correct answer: Oxytocin Explanation The correct response is oxytocin. Your patient most probably has chorioamnionitis secondary to the prolonged rupture of membranes. Delivery is indicated for the benefit of both mother and fetus. Her cervix is closed; therefore, the induction of labor is the best next step in the management. A cesarean section is not indicated in this case; instead, you should try vaginal delivery. You should only consider a cesarean section if there is significant maternal and/or fetal distress and a vaginal delivery is remote. Magnesium sulfate is not indicated; your patient has chorioamnionitis, and you do not want tocolysis. Indomethacin is not indicated; you do not want to delay the labor. Steroids (e.g., betamethasone) are used to improve fetal lung function and are given up to 34 weeks of gestation (the greatest benefit is at less than 32 weeks of gestation). Your patient is at 36 weeks of gestation; therefore, steroids are not indicated.

Case A 49-year-old woman presents with a 1-month history of a rash on her right breast. She reports that it has not been responding to the creams that she has been applying. On examination, she has an erythematous and sharply demarcated crusting over her right areola. There is a nontender lump in the upper outer quadrant. Her left breast is normal. Question What is the most likely diagnosis?

Correct answer: Paget's disease Explanation The most probable diagnosis in this patient is Paget's disease. In Paget's disease of the breast, patients present with a unilateral, sharply marginated, red, scaly rash or erythematous crusting affecting the nipple and areola. There may also be an underlying breast cancer, such as a mammary duct carcinoma. Treatment depends on whether or not there is an underlying malignancy and includes mastectomy if a malignancy is present. There are several types of malignant melanomas, including: Superficial spreading melanomas—plaques with blue-black spots Nodular melanomas—rapidly growing dark papules Lentigo maligna melanomas—brown macules with scattered darker spots on sun-exposed areas, such as the face Acral lentiginous melanomas—most common form in African American patients; they develop on the palms, sole, subungual skin, and mucous membranes Patients with basal cell carcinomas (BCC) can present with a shiny pearly nodule (noduloulcerative BCC), or with erythematous scaling plaques (superficial BCC), or with a solitary flat yellowish plaque (morpheaform BCC). Patients with squamous cell carcinomas (SCC) may present with a red papule or crusted plaque on the lips, ears, neck or hands that may ulcerate, invade the underlying tissue, and metastasize. There are several forms of Kaposi's sarcoma (KS), including: Classic KS—purple plaques or nodules on the lower limbs of elderly men of Mediterranean origin without AIDS AIDS-related form—widely disseminated purple papules or plaques on the skin, mucous membranes, and viscera Lymphadenopathic form—involves lymph nodes and skin

Case A 75-year-old woman presents to your office complaining of intense pruritis of the vulva and occasional bleeding. She is unsure if the bleeding is caused by her scratching in attempts to alleviate the itching. She has tried some OTC preparations to alleviate the itching and has not had any relief from them. She denies any vaginal discharge or dysuria. On physical examination you notice excoriations and some scattered lesions that look like eczema on the vulva and they do not scrape off. Also noted was inguinal lymphadenopathy. You decide to do a punch biopsy. The pathology report reveals large eosinophilic cells. Question What is the most likely diagnosis?

Correct answer: Paget's disease Explanation Paget's diseaseis associated with intense pruritus of the vulva, along with lesions that resemble eczema. Pathology also characteristically shows large eosinophilic Paget's cells (1). Lichen sclerosis is not correct because the pathology would reveal changes associated with chronic inflammation. Vulvar carcinoma is incorrect as it generally shows squamous cell characteristics. (1). Melanoma is incorrect as the lesion will generally appear hyperpigmented (1). Candida albicans is incorrect because the lesions that appear on the vulva do not scrape off and she does not have any vaginal discharge. Candida albicans is also not associated with inguinal lymphadenopathy.

Case A 26-year-old woman, gravida 2 para 2, with a past medical history of Chlamydia cervicitis and recurrent vulvovaginitis has given birth to her third child via cesarean section. She is status-post delivery 24 hours, and the duration of labor was 20 hours. She presently reports that she feels "warm and feverish," has a malodorous vaginal discharge, and has a 6-hour history of achy lower abdominal pain. She denies dysuria, pyuria, urinary frequency, change in urinary odor or color, shortness of breath, chest pain, hemoptysis, nausea, vomiting, and diarrhea; she also denies any drug allergies. Her physical exam is remarkable for a fever of 100.9°F with the remainder of her vital signs normal. Additionally, she has a soft, tender lower abdomen that has noted swelling and frank pain in the lower abdomen/pelvis; a foul, white lochia is noted. The cesarean section incision site is well approximated and without induration, discharge, or erythema; there is no costovertebral tenderness, adnexal mass, or peripheral edema appreciated. Question What is the most appropriate first-line treatment for this patient at this time?

Correct answer: Parental clindamycin plus gentamicin Explanation This patient's presentation is most consistent with endometritis. Established risk factors include prolonged rupture of the membranes (>24 hours), chorioamnionitis, an excessive number of digital vaginal examinations, prolonged labor (>12 hours), toxemia, intrauterine pressure catheters (>8 hours), fetal scalp electrode monitoring, preexisting vaginitis or cervicitis, operative vaginal deliveries, cesarean section, intrapartum and postpartum anemia, poor nutrition, obesity, low socioeconomic status, and coitus near term. Most puerperal infections are due to anaerobic streptococci, gram-negative coliforms, Bacteroides spp., and aerobic streptococci. In up to 30% of culture isolates, group B streptococci are partly or wholly responsible, while gram-negative species (E. coli) comprise up to 36% of all isolates. Common manifestations include fever (early maternal post-operative fever suggests infection of the womb), fever, a soft and tender uterus, cervical motion tenderness, and foul lochia. High fever and hypotension shortly after delivery are classically reflective of group B streptococci. Upon laboratory assessment, a leukocytosis above the established baseline (leukocytosis is a normal finding during labor and the immediate puerperium) indicates infection. Urinalysis and culture may indicate pyuria and bacteria in a UTI. Additional diagnostic tests include a Gram stain and wet mount of vaginal discharge or endocervical culture or DNA probe for Chlamydia. Chlamydia is more frequently associated with mild late-onset endometritis, so cultures for this organism should be obtained from patients with endometritis diagnosed several days after delivery. A combination therapy with clindamycin and an aminoglycoside is considered the criterion standard by which most antibiotic clinical trials are judged. A combination regimen of ampicillin, gentamicin, and metronidazole provides coverage against most of the organisms that are encountered in serious pelvic infections. Doxycycline should be used if Chlamydia is the cause of the endometritis. The combination of clindamycin and gentamicin administered intravenously every 8 hours has been considered the criterion standard, first-line treatment. Parental cefoxitin, Timentin, imipenem, or Unasyn with doxycycline is an alternative. In teenagers, postabortion endometritis may be caused by organisms that cause PID; the initial treatment regimen in these patients is IV cefoxitin and doxycycline. Although urinary tract infections are often associated with a clinical picture similar to that of mild endometritis, the lack of urinary complaints reliably excludes UTI, making the use of Bactrim DS inappropriate. Staphylococcus aureus is not commonly seen in cultures from women with postpartum infections of the uterus, so dicloxacillin is not a suitable empiric choice. High-dose ampicillin is effective against Group D streptococci strains (e.g., Streptococcus faecalis); group D streptococci are, however, isolated from cultures in only 14%. It is added when the patient has a less than adequate response to the usual regimen, particularly if Enterococcus spp. are suspected.

Case A 24-year-old woman presents with a 12-hour history of lower abdominal pain, nausea, and vomiting. She rates the pain at a 4 last night when it initially began, but currently she puts it at a 9 on a numerical pain scale; she states the pain seems to be worsening with each passing hour. She states she had her menses 1 week prior; she noted an irregular flow as well as excessive vaginal discharge since it ceased. She is single and admits to not being monogamous; she only occasionally uses barrier contraception during sexual encounters. She takes a daily oral contraceptive pill. Question Given the details supplied above, what diagnosis is most likely at this time?

Correct answer: Pelvic inflammatory disease Explanation It is highly likely that the patient described above has an active case of salpingitis/endometritis, which is commonly referred to as pelvic inflammatory disease (PID). PID is a several gynecological infection that is often polymicrobial; Neisseria gonorrhea and Chlamydia trachomatis are the most common sexually transmitted organisms that lead to PID. PID is frequently found in young, nulliparous, sexually active women with multiple partners. It is also considered the leading cause of infertility and ectopic pregnancy in this patient population group. PID presents with lower abdominal pain, chills, fever, menstrual disturbances, and purulent cervical discharge. Physical exam will reveal exquisite uterine, adnexal or cervical motion tenderness, as well as cervical discharge. Based on the information provided on the patient above, PID is the most likely diagnosis at this time. Ectopic pregnancy is always a possibility, but it is lower on the overall differential due to the fact that the patient experienced menses just 1 week prior. An ectopic pregnancy is even less likely due to fact that the patient currently takes oral contraceptive pills. Regardless, a serum pregnancy test will be obtained as part of this patient's workup. Appendicitis is always a possibility and commonly will present with the following: early—periumbilical pain; later—right lower quadrant pain and tenderness; anorexia, nausea, vomiting, or low-grade fever are also symptoms. The fact that there is presence of an unusual vaginal discharge as well as the strong history of potentially being exposed to sexually transmitted diseases helps put this diagnosis lower in the differential. Bacterial vaginosis (BV) is also considered a polymicrobial disease however is not considered a sexually transmitted infection. BV will lead to a malodorous discharge but typically will not have any signs or symptoms or inflammation. Endometriosis is a very unpredictable disease state and is variable in its presentation as well as course. Patients most often will complain of varying degrees of dysmenorrhea, chronic pelvic pain, and dyspareunia. It is not an infection, not caused by an organism, and is not a sexually transmitted disease. The patient above has very minor details similar to endometriosis; however, it is not consistent with the general findings of this disease state.

Case A 19-year-old woman presents to the emergency room with lower abdominal pain. She states that the pain started 1 week ago and has gradually been worsening. She is concerned because she developed a fever of 102 F this morning. She also had an episode of vomiting. Her last menstrual period was 1 week ago, and she has noted a foul smelling yellow discharge since then. She is sexually active with 1 partner and does not use contraception. Question What is the likely diagnosis in this patient?

Correct answer: Pelvic inflammatory disease Explanation Pelvic inflammatory disease is correct. Pelvic inflammatory disease (PID) occurs when bacteria spreads from the vagina into the uterus, fallopian tubes, and/or ovaries. It is usually sexually transmitted, with chlamydia and gonorrhea being the most common causes. It may be asymptomatic until late in some individuals. The patient's symptoms of abdominal pain, fever, and vaginal discharge all occur in PID. Furthermore, the patient does not practice safe sex, placing her at higher risk for PID. Acute appendicitis is incorrect. Acute appendicitis does have gradual onset of abdominal pain, but this typically occurs over several hours instead of days. Acute appendicitis is not associated with vaginal discharge. Ruptured ectopic pregnancyis incorrect. While the patient is not using contraception, her history is not conducive with the presence of ectopic pregnancy. Her last menstrual period was 1 week ago. Even if the patient is pregnant, most ectopic pregnancies do not rupture until the patient is 6-8 weeks pregnant. Pregnancy should be ruled out with hCG testing, but PID is more likely in this patient. Ovarian cyst is incorrect. While ovarian cyst may cause abdominal pain, it is not typically associated with malodorous vaginal discharge. Ovarian torsion is incorrect. Ovarian torsion may cause lower abdominal pain, but the onset is typically more abrupt.

Case A 19-year-old woman presents with a 2-day history of worsening pelvic pain. She describes the pain as sharp and throughout her pelvis, and she is unresponsive to over-the-counter analgesics. The pain onset was gradual, and she denies trauma. She admits to some discolored vaginal discharge and mild fever. She denies vaginal pruritus or burning, dysuria, hematuria, nausea, vomiting, and bowel changes. The pain does not change with meals, urinating, or bowel movements. She experiences dyspareunia. Her past medical history is unremarkable, with no known medical conditions; she has no allergies, she takes no medications, and she has not had any prior surgeries. She denies the use of alcohol, tobacco, and drugs. Her last menstrual period was approximately 3 weeks ago. She uses no contraception, and she admits to "multiple" sexual partners. On physical exam, the patient has an oral temperature of 100.2F (37.9C); she appears to be in discomfort on the exam table, and her skin feels warm. She is diffusely tender with guarding on abdominal and bimanual exam throughout the lower abdomen/pelvis. On speculum exam, some yellowish cervical discharge is noted. Her cervix is extremely tender with movement. The remainder of her exam and vitals were normal. Initial tests are obtained, with results as shown. Complete blood count (CBC) Elevated white blood cell count (WBC) Urinalysis Normal Complete Metabolic Panel (CMP) Normal Serum hCG Negative Abdominal and pelvic ultrasounds Free pelvic fluid, which is otherwise normal Chlamydia trachomatis Pending Neisseria gonorrhoeae Pending Question What is the most likely diagnosis?

Correct answer: Pelvic inflammatory disease Explanation This patient is presenting with pelvic inflammatory disease (PID). PID can be an acute or chronic condition, in which inflammation occurs throughout the female genital tract (often from sexually transmitted organisms, such as Neisseria gonorrhoeae and Chlamydia trachomatis). Symptoms can range from subtle discomfort or vaginal discharge to an acute abdomen with a septic patient. PID is diagnosed clinically, and the diagnostic criteria are quite broad, only requiring one or more of the following: 1) cervical motion tenderness, 2) uterine tenderness, or 3) adnexal tenderness. Unrecognized, untreated PID can lead to chronic inflammation, scarring and infertility. This patient's sexual history and age are risk factors for PID. Another urgent diagnosis, ectopic pregnancy, can present with pelvic pain, such as this patient's. She is at high risk for pregnancy. However, her serum hCG (pregnancy) test is negative, so ectopic pregnancy can be effectively removed from the differential. Endometrial hyperplasia is a condition in which the lining of the uterus (endometrium) over-proliferates. It can lead to malignancy. Endometrial hyperplasia can occur with polycystic ovarian syndrome, obesity, and inappropriate unopposed administration of estrogens. Hyperplasia is usually asymptomatic and does not cause acute pelvic pain. This patient's ultrasound excludes endometrial hyperplasia. An ovarian malignancy would be very rare in a 19-year-old patient. Ovarian malignancy is most common in post-menopausal women, and it may present with insidious onset of abdominal/pelvic discomforts and bloating. This patient's ultrasound did not reveal any ovarian masses. Polycystic ovarian syndrome (PCOS) is a complex and common syndrome. In order to be diagnosed with PCOS, a patient must have two or more of the following: 1) Menstrual history of infertility and/or oligomenorrhea, 2) Polycystic ovaries, demonstrated on ultrasound, 3) Clinical or laboratory evidence of elevated androgens (acne, hirsutism, elevated testosterone). Some women with ovarian cysts can present with pelvic pain, but pain is not a hallmark of PCOS. This patient has no history, physical exams, or labs suggestive of PCOS as a diagnosis.

Case A 37-year-old gravida 4, para 1, whose last normal menstrual period was 6 months ago, presents with a 5-month history of irregular menses. She states that her menstrual period is now 10 days in length. She uses approximately 10 - 12 pads or tampons daily during menses. She also has unpredictable bleeding between menstrual periods; this bleeding may begin anywhere from 2 - 12 days after the end of her regular period and may last for between 2 - 15 days. She uses between 5 - 10 pads or tampons a day during this time. She denies syncope, but admits to headaches and occasional dizziness. She also complains of bilateral pelvic pain that is present constantly; it radiates down both thighs. The pain is crampy, and it is relieved to some degree by naproxen sodium. On exam, the patient's vital signs are within normal limits. Abdominal exam demonstrates normal bowel sounds. There is a globular and irregular pelvic mass that extends about 5 cm above the umbilicus; the mass is nontender. There is no rebound or guarding. The pelvic exam reveals scant blood in the vaginal vault, no vaginal or cervical lesions, and no cervical motion tenderness; it confirms the presence of the pelvic mass. Question What is the next best step in management for this patient?

Correct answer: Pelvic ultrasound Explanation The patient's age and history of menometrorrhagia, pelvic pain, and a globular, irregular pelvic mass are strongly suggestive of leiomyomata uteri. Fibroids are the most common solid pelvic tumors in women. The best study to evaluate the mass is pelvic ultrasound. Dilation and curettage is not indicated at this time. The patient should have an endometrial biopsy performed because she is over the age of 35 and is experiencing abnormal vaginal bleeding, though it is probably due to leiomyomata uteri. If an endometrial biopsy is unsuccessful, then hysteroscopy and dilation and curettage would be indicated. CT scan of the abdomen and pelvis, with and without contrast, is not indicated at this time. If the ultrasound exam is suspicious for malignancy, a CT scan of the abdomen and pelvis would be indicated to further evaluate the mass, as well as to determine the extent of any lymphadenopathy. Hormone studies (FSH, LH, prolactin, free and total testosterone) are not generally useful in the diagnosis of a pelvic mass. They may be of use in a patient with abnormal vaginal bleeding whose pelvic exam does not reveal any masses.

Case A 28-year-old woman in her 2nd trimester of pregnancy presents with a 3.5 cm painless mass on her left breast. She has no other associated symptoms. She has a positive family history of breast cancer. She is afebrile; pulse is 80/min, and BP is 110/72 mmHg. On examination of the breasts, you notice engorgement and hypertrophy. The mass appears solid on ultrasonography (USG). Question What is the next step in evaluation of this mass?

Correct answer: Percutaneous biopsy under local anesthesia Explanation The correct answer is percutaneous biopsy under local anesthesia. This is used to obtain a histological diagnosis, which can be done safely during pregnancy with a sensitivity of around 90%1. During pregnancy, non-ionizing examinations are preferred to those needing ionizing agents1. Breast ultrasonography is the first diagnostic instrument used by clinicians when a breast mass and the axillary area need to be assessed in a pregnant woman; it is non-ionizing and has high sensitivity and specificity1. MRI is incorrect; it should only be used when it will alter clinical decision making and when ultrasonography is inadequate1. No well-designed studies of the efficacy and safety of MRI of the breast during pregnancy have been reported, and results of some studies have shown that gadolinium-based MRI contrast agents pass through the placental barrier and enter fetal circulation1. Fine needle aspiration cytology (FNAC) should be done only in case of a cystic mass1. Gestational and puerperal hormones induce physiological hyperproliferative changes of the breast, which could lead to a false positive or false negative result with FNAC1. Therefore, this procedure is not recommended during pregnancy1. Staging with sentinel lymph node biopsy is done once the diagnosis of breast cancer is established. Clinicians can safely use sentinel lymph node staging during pregnancy using technetium 991. However, blue dye is associated with a risk of an anaphylactic maternal reaction, and it would probably distress the fetus; therefore, the use of blue dye should be avoided during pregnancy. Although sensitivity and specificity of sentinel lymph node biopsies during pregnancy have not been established, researchers have successfully used technetium-based identification in pregnant women1. Staging examinations and tumor biology assessment affect management decisions in pregnancy and should not be avoided.

Case A 58-year-old woman with no significant past medical history presents with a 6-month history of "heartburn", sometimes occurring after meals. There is associated fatigue, bloated abdominal sensation, early satiety, and alternating constipation and diarrhea. She denies fever, chills, changes in weight, chest pain, shortness of breath, abdominal pain, nausea, vomiting, melena, hematochezia, and vaginal discharge. Her last menstrual period was 4 years ago. She is unmarried, and she does not have any children. Her physical exam reveals normal vital signs and a normal cardiopulmonary exam. Her abdomen is protuberant; there is a shifting dullness and a fluid wave noted. The pelvic exam reveals a solid, irregular fixed lesion in the left lower abdomen. Question What is the most appropriate initial intervention for this patient at this time?

Correct answer: Perform a transvaginal ultrasound Explanation This patient's presentation suggests ovarian cancer. Doppler transvaginal ultrasonography is used in the initial evaluation, and it is the most useful initial investigational tool in the assessment of adnexal masses. Ultrasonography may define the morphology of the pelvic tumor. In addition, it can determine whether the suspected tumor has metastasized to other abdominal organs, such as the liver. Sonographic features suggestive of cancer include complexity with solid and cystic areas, extramural fluid, echogenicity, wall thickening, septa, and papillary projections. Proton pump inhibitors and irritable bowel therapeutics should not be initiated without adequate investigation into the presence of abdominopelvic malignancy. In patients with diffuse carcinomatosis and gastrointestinal symptoms, a GI tract workup and endoscopy may be indicated as an adjunctive modality to a transvaginal sonogram.

Case A 46-year-old woman presents for a consultation. She takes a multivitamin, iron, and calcium as advised. She has never smoked cigarettes and does not drink alcohol at all. After she gave birth to her fourth child about 7 years ago, she underwent bilateral tubal ligation because both she and her husband decided not to have more children. Afterward, she started gaining weight (now her BMI is 30). In the last several months, her periods have come every 2 weeks and last about 5-7 days. Before that, they were regular: every 28-30 days, with 3-5 days of bleeding. Her mother died from breast cancer at age 70. Physical examination reveals pallor and tachycardia. Pelvic examination is normal. Question What is the next best diagnostic step?

Correct answer: Perform endometrial biopsy Explanation The patient most likely has abnormal uterine bleeding, formerly dysfunctional uterine bleeding (DUB).This is irregular uterine bleeding that occurs in the absence of pathology or medical illness. It is a diagnosis of exclusion. Laboratory studies for patients with abnormal uterine bleeding include human chorionic gonadotropin (HCG), complete blood count (CBC), Pap smear, endometrial sampling, thyroid functions and prolactin, liver functions, coagulation studies/factors, and testing for the presence of uterine fibroids or polyps and hormone assays when indicated. Most important, however, is to exclude endometrial hyperplasia or carcinoma because your patient has several risk factors: she is older than 35 years, she is obese, and she has prolonged periods of unopposed estrogen stimulation and probably chronic anovulation. Endometrial biopsy is the most important and most commonly used diagnostic test for abnormal uterine bleeding in such a patient. In abnormal uterine bleeding, there is usually a hormonal imbalance (too much estrogen thickens endometrium, and progesterone causes excessive irregular bleeding). The effect of oral contraceptive pills on endometrium mimics that of an ovulatory cycle. However, before you start oral contraceptive pills, you should confirm the diagnosis of abnormal uterine bleeding, which means excluding the other reasons. Follicle-stimulating hormone (FSH) testing may be ordered when a woman's menstrual cycle has stopped or become irregular to determine if the woman has entered menopause. That is not the case in your patient. Mammography is indicated in this woman. The National Cancer Institute recommends that women age 40 or older should have screening mammograms every 1 to 2 years. However, it is not the most important test you should order at this point; instead, you should find a reason for bleeding because of all the consequences of losing blood and because of the risk of endometrial cancer. Checking luteinizing hormone (LH) will be indicated in women having difficulties getting pregnant, having irregular or heavy menstrual periods, or having symptoms of pituitary, hypothalamic, or ovarian disorders. Again, endometrial biopsy done in the office is probably more important.

Case A 28-year-old sexually active woman presents with a 1-week history of vaginal discharge and odor. She has been in a monogamous relationship and she denies any history of similar problems. She had an annual exam last month; chlamydia and gonorrhea tests were negative, and her Pap result was ASCUS with a reflex HPV, which was negative. The pelvic exam confirms increased watery discharge and no foreign body. Question What is the most appropriate next step in the management of this patient?

Correct answer: Perform microscopic examination and pH testing of discharge. Explanation The initial steps in the evaluation of a patient with symptomatic vaginitis are physical exam, microscopic evaluation (using saline and KOH), and testing pH of the discharge. These tests can all be performed quickly in the clinic. The patient most likely has a bacterial vaginosis by history. Expected objective findings include non-inflammatory homogenous discharge, increased pH, fishy odor of the discharge (a positive "whiff test"), and the presence of clue cells on the microscopic exam. If the patient's history and physical exam support the diagnosis, then treatment should be oral or vaginal clindamycin or metronidazole, or possibly oral tinidazole. A viral culture should not be obtained; the history does not suggest common viral infection, such as herpes or condyloma. Viral culture results would not be immediately available and delay appropriate treatment of the patient's vaginitis. While the procedure is non-invasive and would not harm the patient, it is not cost-effective care. The intravaginal imidazole and triazole antifungals, such as clotrimazole and terconazole, are appropriate treatments for vaginal candidiasis or "yeast infections." The patient's history is not consistent with typical vaginal candidiasis (complaints of discharge with pruritus, but not odor). A microscopic examination and pH testing, as recommended above, would help to distinguish a yeast infection from bacterial vaginitis. The diagnosis should be established prior to treatment. Given this patient's history, repeat testing for Chlamydia trachomatis and Neisseria gonorrhea is not the best evaluation. The patient stated that she monogamous and had very recent testing for these sexually transmitted infections. Both C. trachomatis and N. gonorrhea infections may present with vaginal discharge or leave the patient asymptomatic. Similar to the case with viral cultures, retesting for these organisms does not yield immediate results, so while it is non-invasive and non-harmful, it is not cost-effective care for the patient. Repeat Pap testing is not indicated for this patient. Her previous result from 1-month ago was ASCUS with a negative HPV test, and the recommended follow-up Pap test interval is 1 year.

Case A G3P3 37-year-old Hispanic woman presents because she only menstruates a few times per year. Her family is concerned, but she is not because her menses have demonstrated this pattern most of her life. Menses last about 10 days. Her last menstrual period occurred 4 months prior to presentation. She denies dysmenorrhea, pelvic pain, nausea, vomiting, breast tenderness, fatigue, and vasomotor symptoms. She is not breastfeeding, and she delivered her last child 3 years prior to presentation. She and her husband are considering another child. The review of systems and past medical history is unremarkable. She takes a prenatal vitamin daily, but no other medications. The patient is a homemaker; she is married and lives with her husband, 3 children, and mother-in-law. She denies tobacco, drug, and alcohol use. Urine specimen and vitals are collected. Results are shown in the table. Weight 226 lb Height 64" Pulse 90 Blood pressure 132/85 Urine hCG Negative Physical exam reveals a pleasant, obese woman. Her uterus and adnexa are not palpable on physical exam due to obesity. The remainder of the physical exam is normal.

Correct answer: Perform ultrasound and endometrial biopsy. Explanation This patient should have an ultrasound and endometrial biopsy; she is at risk for prolonged stimulation to the endometrium, resulting in endometrial hyperplasia. Other risks factors for endometrial hyperplasia include obesity and being over 35. The classic anovulatory menstrual pattern presents with a history of infrequent (but heavy and long) menses. An ultrasound can evaluate the endometrial thickness, and an endometrial biopsy can determine histologically if hyperplasia is present. Monitoring this patient's menstrual cycle would provide helpful historical data, but doing so is not enough. So far, she has denied any significant pain. Endometrial hyperplasia and the development of endometrial cancer are not typically associated with pain. It is likely that this patient may be lost to follow-up if no intervention is done at this time. A levonorgestrel intrauterine system (Mirena) would be appropriate management for a patient at risk for endometrial hyperplasia, which is the case with this patient. It releases low-dose progestin to the endometrium, resulting in a thinning of the endometrium. However, this patient does not desire contraception, and the workup has yet to be completed, so insertion of a Mirena is inappropriate at this time. An MRI to evaluate the pituitary would be appropriate in patients with amenorrhea suggestive of a brain tumor. An example might be a prolactinoma, in which the patient has menstrual changes, headaches, galactorrhea, and possibly vision changes. This patient has a chronic history of irregular menses and no other endocrine or nervous system signs or symptoms. Appropriate blood tests should evaluate endocrine function; if normal, they negate the need to image the brain. Endometrial ablation is a permanent procedure in which the lining of the uterus (endometrium) is destroyed, preventing future proliferation of the tissue. It would reduce the risk of endometrial hyperplasia, but it should not be performed in women who desire future childbearing. Additionally, a complete workup, including ultrasound and endometrial biopsy, should be performed prior to ablation procedures. In addition to addressing her menstrual issues, the provider should counsel this patient regarding her risks (advanced maternal age and obesity) regarding a future pregnancy.

Case A 51-year-old Caucasian woman presents to the clinic to follow up on multiple tests she had requested from another practitioner. She has had what she describes as "episodes," in which she feels overheated and diaphoretic. Her coworkers note that her face turns red. These episodes seem to occur multiple times during the day; they have been occurring for the last 2-3 months, and they last 1-2 minutes before they resolve. She is also very warm when sleeping at night. She denies weight changes, palpitations, headaches, galactorrhea, acne, bowel changes, hair loss, and any changes to her skin or nails. Overall, she reports some mild malaise and irritability, but she denies depression and fatigue. Her past medical history is unremarkable, with no known medical conditions, allergies, medications, or prior surgeries. She denies the use of alcohol, tobacco, and drugs. Her method of contraception is her husband's vasectomy. Her last menstrual period was about 4 months ago, but periods had been regular and monthly prior to that. Physical exam and vitals are normal. Test results are shown. Complete blood count (CBC) Normal Thyroid-stimulating hormone (TSH) Normal Urinalysis Normal Complete Metabolic Panel (CMP) Normal Follicle-stimulating hormone (FSH) Elevated Quantitative hCG Negative Prolactin Normal Free/total testosterone Normal Question What is the most likely diagnosis?

Correct answer: Perimenopause Explanation This patient is experiencing the climacteric stage of life, which is often referred to as menopause. Technically, menopause is defined as the point in time in which a woman has had no menstrual periods for 1 year, which is due to decreased ovarian function, and the average age at which menopause occurs is 51.5 years. This woman would be considered perimenopausal. The most common presenting symptoms of this normal transition of the reproductive phase of life are hot flushes (flashes) and night sweats. Other symptoms include irritability, insomnia, memory loss, and vaginal atrophy. Clinically, the ovarian hormones are not tested. Instead, the rise in the "upstream" follicle-stimulating hormone (from the pituitary) in response to very low ovarian output is easily measured. In most cases, with classic symptoms and a patient in their 40s-50s, testing for menopause is not necessary. If the patient desires, or additional symptoms, history, and/or physical exam suggest another disorder, testing can be done. A pituitary adenoma can lead to amenorrhea and various endocrine symptoms. If the pituitary gland was affected by a tumor, several of the pituitary hormones (such as the TSH, prolactin, and FSH) would all be expected to be abnormal. Pituitary adenomas are much rarer than perimenopause, and this patient does not demonstrate any symptoms, history, or physical exam findings to support an adenoma. Polycystic ovarian syndrome (PCOS) is a complex and common syndrome. In order to be diagnosed with PCOS, a patient must have 2 or more of the following: Menstrual history of infertility and/or oligomenorrhea Polycystic ovaries, demonstrated on ultrasound Clinical or laboratory evidence of elevated androgens (acne, hirsutism, elevated testosterone) Other than 4 months of amenorrhea, this patient has no history, physical exams, or labs to suggest PCOS as a diagnosis. Pregnancy is the most common cause of amenorrhea in patients, and should always be excluded before making alternate diagnoses. This patient has both a reliable form of contraceptive (vasectomy) and a negative hCG (pregnancy test). She also is not reporting other common symptoms of early pregnancy, such as weight gain, nausea, and breast tenderness.

Case A 23-year-old primigravida is referred to her obstetrician by a community nurse. The nurse noted 2 blood pressure readings of 150/90 and 154/90 taken 15 minutes apart. Physical examination reveals a uterus at roughly 24 weeks gestation and grade 2 pitting edema. Urine analysis is 1+ positive for albumin. A sonogram at eight weeks gestation showed a single live intra-uterine gestation. She has had regular antenatal check-ups, and has no past history of hypertension or diabetes. Both her parents and grandparents are hypertensive. She is unaware of whether her mother or grandmother faced similar problems during their pregnancies. Question What is the most likely initiating event for her condition?

Correct answer: Placental ischemia Explanation Placental ischemia is the correct answer. The vignette describes a woman with classic features of pregnancy induced hypertension, as evidenced by the blood pressure readings, edema, time of onset beyond 20 weeks and proteinuria. The initiating event appears to be abnormal cytotrophooblast invasion of the spiral arterioles, which leads to activation of the maternal vascular endothelium. The cascade that follows includes enhanced formation of endothelin and thromboxane, increased vascular sensitivity to angiotensin II, reduced nitric oxide, and prostacyclin (vasodilator) synthesis. Over activation of her renin-angiotensin system is incorrect. While it is an important component of the response, it does not initiate the process. Overproduction of B-HCG is incorrect. It may result in high blood pressure as seen in hydatiform mole and multiple gestations. However, there is no indication of either in this vignette. Renal artery stenosis is incorrect. It is a common cause of hypertension in young individuals. It would be present through the duration of pregnancy. Essential hypertension is incorrect. Essential hypertension is a pathology associated with aging, the pathogenesis of which is unclear. It is certainly unlikely in a 23-year-old. In addition, the patient would have a past history of hypertension.

Case A 24-year-old obese woman arrives at an infertility clinic accompanied by her husband. The couple has been married for 3 years, and they have been unsuccessful at conceiving a child. History and examination of the male partner is unremarkable. She attained menarche at 13 years of age and gives history of irregular cycles for the past 5 years. There is no history of pelvic pain or discomfort. The patient's last menstrual period was 2 months ago. On physical exam, the woman has hirsutism and acne, and her pelvic examination is unremarkable. Investigations reveal elevated serum levels of dehydroepiandrosterone sulfate (DHEAS) and androstenedione. Question What is the most likely cause of infertility in this patient?

Correct answer: Polycystic ovarian syndrome Explanation The clinical presentation and investigatory findings are suggestive of polycystic ovarian syndrome. Polycystic ovarian syndrome (PCOS) was previously called as Stein-Leventhal syndrome. Polycystic ovarian syndrome is characterized by infertility, obesity, hirsutism, and menstrual disturbances; irregular menses can sometimes be seen, rather than amenorrhea. In addition, the ovaries will have numerous cysts. The diagnosis of PCOS can be made based on clinical and biochemical grounds of hyperandrogenism and oligo-ovulation after exclusion of other disorders such as congenital adrenal hyperplasia and hyperprolactinemia. Other features observed are overt diabetes due to insulin resistance, acanthosis nigricans, hypertension, and dyslipidemia. Ultrasound examination reveals ovaries studded with multiple, tiny follicular cysts surrounding the ovarian stroma. The aim of treatment is suppression of androgen production by the use of oral contraceptives or medroxyprogesterone acetate. Clomiphene citrate or gonadotropins are used for those desiring conception. Pelvic inflammatory disease (PID) can also lead to infertility; however, there would not be an association of PID with obesity, amenorrhea, or with features of hyperandrogenism such as hirsutism, acne, and elevation of DHEAS and androstenedione. Dysmenorrhea can be seen with PID. In addition, a patient with PID would usually present with lower quadrant pain. On physical exam, one would expect some positive findings, which was not the case here. Endometriosis can present with dysmenorrhea and irregular menstrual cycles. However, there is no association of endometriosis with obesity, hirsutism, and other features of hyperandrogenism. Leiomyomas are usually asymptomatic. Obesity and hirsutism are not associated with leiomyomas. Menorrhagia, rather than amenorrhea, is a more frequent presentation with leiomyomas. Endometrial carcinoma is seen in post-menopausal women. Obesity and hirsutism are not associated with endometrial carcinoma.

Case A 25-year-old female presents to your office complaining of inability to conceive despite persistent effort for the last 10 months. She is otherwise healthy with no significant medical or surgical history. The only medication she has ever taken is oral contraception, which she discontinued 18 months ago in anticipation of her desire to get pregnant. She began menstruating at age 12 and states that her periods have been irregular and somewhat painful for as long as she remembers. In addition, she says she has to wear pads because her bleeding is too heavy to control with tampons. She has had two sexual relationships in her lifetime and denies a history of sexually transmitted disease. She adds, "I know it's not my husband because he already has a son from his first marriage." Initial exam reveals an obese female with mild facial acne and coarse facial hair. Examination of the breasts and external genitalia are unremarkable. Pelvic exam is significant for moderately enlarged ovaries bilaterally with no adnexal tenderness. Question Based on your assessment, what is this patient's diagnosis?

Correct answer: Polycystic ovarian syndrome Explanation Polycystic ovarian syndrome, also known as Stein-Leventhal syndrome, is an idiopathic disorder characterized by anovulatory menstrual cycles and infertility, multiple ovarian cysts, as well as hirsutism, acne, alopecia, and hypertension. In addition, patients are often mildly obese and demonstrate impaired insulin sensitivity. The disorder is marked by excessive ovarian androgen production. These androgens undergo aromatization into weak estrogens in peripheral fat, which in turn exert a positive feedback loop on LH secretion as well as suppression of FSH release. LH further stimulates the proliferation of ovarian cysts, leading to continued androgen production. This cycle of events explains the classic laboratory findings of the syndrome, notably an LH: FSH ratio in excess of two in addition to elevated levels of androstenedione and testosterone. When not desirous of fertility, patients often experience symptomatic relief with oral contraceptive therapy. If fertility is desired, clomiphene citrate is the most common therapy. This anti-estrogen, when given appropriately, alters GnRH and support gonadotropic release in order to stimulate induction of ovulation. Kallmann syndrome is an X-linked disorder marked by GnRH deficiency and olfactory defects leading to anosmia or hyposmia. The condition is thought to be the result of impaired migration of GnRH-producing cells to the hypothalamus during embryogenesis. In the absence of GnRH stimulation, LH and FSH levels are low, leading to hypogonadism. The development of secondary sexual characteristics is delayed because of low levels of peripheral estradiol in affected women and testosterone in affected males. If fertility is desired, affected females may be treated with pulsatile GnRH therapy and gonadotropins. Asherman syndrome is a condition marked by uterine adhesions - or synechiae - that form in response to excessive endometrial curettage or uterine surgery. However, less commonly, it can also occur because of severe pelvic infection. In response to trauma, the normal endometrial lining is replaced by scar tissue, leading to the development of amenorrhea and infertility. Treatment via hysteroscopic adhesiolysis has been moderately successful for improving fertility. Meigs' syndrome describes a triad of hydrothorax, peritoneal ascites, and the presence of a benign ovarian fibroma. Patients most commonly present with expanding abdominal girth caused by tumor growth and ascites formation. The subsequent development of pleural effusions leads to dyspnea and pleurisy. CA-125 levels are characteristically elevated, and the diagnosis is confirmed with exploratory laparotomy revealing non-malignant tumor. Unilateral or bilateral salpingo-oophorectomy should be performed, depending on the patient's desire for preservation of fertility. Upon resection, ascites and pleurisy disappear and CA-125 levels return to normal. Sheehan's syndrome is the result of ischemic injury to the anterior pituitary gland observed following severe postpartum hemorrhage. Patients commonly present with oligomenorrhea, impaired lactation, cold intolerance, coarse hair, and fatigue and weight loss. Symptoms and laboratory findings are consistent with panhypopituitarism, specifically depressed levels of LH, FSH, GH, TSH, ACTH, and prolactin. These abnormalities in turn lead to low T4, estradiol, and cortisol concentrations. Replacement hormone therapy is required to alleviate symptoms and offer patients the opportunity to conceive.

Case A 16-year-old otherwise healthy girl presents with cessation of her menstrual cycle. She does not take any medications or use tobacco or drugs. She has a boyfriend, but she denies intercourse. Her family history is negative for gynecologic or fertility problems, autoimmune diseases, and endocrinopathies; her mother's and female relatives' menarche presented in the ages from 12-14 years. There are no syndromic features on examination. Her height, weight, and BMI are within normal range, and vital signs are normal. Her skin is clean (no acne) and there are no signs of hirsutism. There is no thyromegaly. Her breasts are developed and pubic and axillary hair is present. Her abdomen is benign; her external genitalia maturated; there is no clitoromegaly; vaginal mucosa is moist and pink; and saline moistened applicator swab reveals normal vaginal length. Single finger examination demonstrates the presence of uterus. Neurological examination is nonfocal; sense of smell is preserved; visual fields are normal by confrontation; and fundoscopic examination shows no papilledema. Question What will you consider first in further evaluation of amenorrhea in this girl?

Correct answer: Pregnancy Explanation Pregnancy must be considered as a possible causative factor of amenorrhea regardless of history because denial of intercourse among adolescents is common or the patient may not be able to disclose sexual activity or abuse. Around 98% of girls in US achieve menarche by the age of 15. The presence of secondary sexual characteristics makes the evaluation of pubertal delay unnecessary. Chromosomal abnormalities may also present with amenorrhea (e.g., Turner syndrome, androgen insensitivity). General appearance may reveal some syndromes (e.g., Turner syndrome with webbed neck, shield chest, widely spaced nipples) and midline facial defects associated with hypothalamic-pituitary axis problems and renal and vertebral anomalies in Müllerian defects, etc. Physical examination does not suggest these diagnoses; therefore, you should consider the more probable and more affordable diagnostic approach first. Functional hypothalamic amenorrhea is a diagnosis of exclusion when pregnancy and other pathologic causative agents are not found. There are no signs of pituitary and/or ovarian failure or gonadal dysgenesis in your patient. As axillary and pubic hair is developed, her adrenal androgens are probably normal. The presence of developed breasts and maturation of the external genitalia, vagina, and uterus shows that estrogen levels are probably within normal limits.

Case A 16-year-old girl presents after missing her period for the past 3 months. Onset of menses was at age 13, with initial irregularity for 6 - 8 months, and then consistency. She has a serious boyfriend but repeatedly denies any sexual activity. She also denies any drug use. She has been healthy, without any stomach pains, vomiting, diarrhea, vaginal symptoms, or dysuria. She claims to eat quite well. Her weight is at the 75th percentile and height at the 25th percentile. On further questioning, she admits that she has actually been trying to lose weight and is exercising more to make the cheerleading team. However, she denies any binge eating, purging, or laxative use. There is a positive family history for diabetes. Question What is the most likely cause of secondary amenorrhea in the patient?

Correct answer: Pregnancy Explanation Secondary amenorrhea is the cessation of menstruation for 3+ consecutive cycles after at least 24 months of regular monthly cycles when a normal ovulatory pattern should have been established. Most commonly, this results from some disruption of the hypothalamic-pituitary-ovarian axis. Natural causes of secondary amenorrhea include breastfeeding, menopause, and pregnancy. Pregnancy is the most common cause of secondary amenorrhea in adolescents. It should be considered and tested for in all situations. Denial of sexual activity does not preclude the need to rule out possible pregnancy, and it should be explored in this patient. Stress-related conditions are a common form of disruption of the hypothalamic-pituitary-ovarian axis, contributing to functional hypothalamic amenorrhea. By definition, this disorder excludes pathologic disease and is described by a decrease in hypothalamic GnRH secretion. A hypothalamic state is induced which reduces secretion of gonadotropin-releasing hormone (GnRH) producing levels of luteinizing hormone (LH) and follicle-stimulating hormone (FSH) that are too low to stimulate ovulation. Therefore, estrogen is not produced by the follicle that would normally support the proliferative phase of the endometrium. The triad of eating disorder, amenorrhea, and osteoporosis (also known as the female athlete triad) is well recognized among young women competing in certain sports emphasizing leanness. Secondary amenorrhea can be an early signal of anorexia nervosa, occurring before dramatic weight loss is apparent. Failure of normal ovarian function resulting in amenorrhea can also result from autoimmune processes often associated with thyroid dysfunction and diabetes mellitus. Poorly controlled diabetes is associated with anovulation. Patients will have low levels of estradiol and elevated FSH levels from an absence of a normal negative feedback system. Nothing in this patient's history alludes to a possible autoimmune problem. Craniopharyngioma (or pituitary adenoma) are found in the region of the sella turcica and, like stress-related conditions, can also disrupt the hypothalamic-pituitary-ovarian axis. Headache, vomiting, and vision loss or changes are some clinical features to look for; they are not apparent in this patient. Illicit drug use can also cause secondary amenorrhea and should be ruled out. Phenothiazines and opiates have been shown to suppress the activity of hypothalamic prolactin-inhibiting factor. Although this patient denies illicit drug use, this should also be further explored.

Question Vaginal trichomoniasis has been associated with adverse pregnancy outcomes. What is an example of such an outcome?

Correct answer: Premature rupture of membranes Explanation Trichomoniasis infection is associated with premature rupture of membranes, preterm delivery, and low birth weight. There is no proven association between trichomoniasis and hyperemesis gravidarum, preeclampsia, eclampsia, or gestational diabetes.

Case A 30-year-old woman presents with a 2-week history of vaginal discharge. She denies vaginal burning or itching, as well as urinary frequency, hesitancy, and dysuria. The discharge is reported as light yellow in color. She is hesitant to provide much detail, so a physical exam is performed. On physical exam, the patient is in no distress, afebrile, has normal vitals, and a normal exam except for pelvic. On speculum examination, the cervix is inflamed, bright red, and slightly friable with yellow discharge at the os. Vaginal mucosa is pink, moist, and without inflammation. Uterus and ovaries palpate to normal size and are nontender on bimanual exam. Samples are collected for analysis and the results that are available are listed in the table. Test Result Urine hCG Negative Urinalysis (dipstick) Normal, except for 1+ WBC Wet mount/KOH slide Negative for trichomonads, hyphae, and clue cells, >10 WBC per high-powered field Complete blood count Normal Question What action is the most appropriate intervention for this patient given her likely diagnosis?

Correct answer: Prescribe empiric antibiotics to cover Chlamydia trachomatis and Neisseria gonorrhoeae Explanation This patient presents with cervicitis, an inflammation of the cervix characterized by mucopurulent cervical discharge and/or endocervical bleeding (induced by swabbing the os). Cervicitis, especially with the finding of increased white blood cells (WBCs) in the vaginal fluid, is associated with sexually transmitted diseases (e.g., Chlamydia trachomatis and Neisseria gonorrhoeae). The increased vaginal leucorrhea may also explain the (mildly) positive leukocyte esterase on the urinalysis dipstick. Cervicitis may occur on its own or as part of pelvic inflammatory disease (PID). Of the choices listed, the most appropriate intervention is to prescribe empiric antibiotics to cover Chlamydia trachomatis and Neisseria gonorrhoeae. Test results for these 2 infections are not readily available at the bedside; however, treatment is recommended when the clinical diagnosis is made, rather than delaying and possibly missing an opportunity to treat and prevent sequela. If the patient were menopausal, with vulvovaginal atrophy as the cause of her symptoms, the most appropriate treatment would be to initiate twice weekly vaginal estrogen. Common symptoms of atrophy include dryness, irritation, and dyspareunia. On physical exam, both the vulva and vagina are noted to be pale and dry. The tissue may also be friable, but the cervix would not be inflamed. If this patient's signs and symptoms were caused by a vaginal yeast infection (candidiasis), it would be reasonable to prescribe an antifungal (e.g., fluconazole). Common yeast infection symptoms include vaginal burning and pruritus; the exam reveals inflamed vaginal mucosa and thick white discharge. The wet mount slide with KOH (potassium hydroxide) would reveal the characteristic branching hyphae. If this patient's symptoms were caused by bacterial vaginosis (BV), it would be best to prescribe oral or intravaginal metronidazole. BV is characterized by a watery, thin vaginal discharge and fishy odor. On the wet mount of vaginal secretions, clue cells are seen. Currently, there are no gynecologic conditions for which experts recommend douching with a vinegar solution. Douches used to be quite popular and are still readily available over-the-counter, but have little utility.

Case A 25-year-old woman presents to discuss her options of available contraception. You review her history and note menses onset at 12 years old, duration of menses typically around 6 days in length, and it occurs every 30 days. She is G0P0, and she has no history of abnormal pap smears or diagnosed STIs. The patient is a non-smoker, single, and in a monogamous relationship with 1 partner for the past year. All other medical history is noncontributory with the exception of the patient having a deep venous thrombosis at age 19 and a pulmonary embolism at age 21. Other than anticoagulation therapy for the appropriate amount of time, no other hematological evaluation was pursued after these events. Question Given the most likely inherited diagnosis, what would be the safest form of contraception at this time?

Correct answer: Progestin-only oral contraception pill Explanation The correct response is progestin-only oral contraception pill. Our patient above is overall healthy 25-year-old female patient. However, her history of multiple hypercoagulable events without any substantial risk factors puts her in the likely category of possessing an inherited hypercoagulable state. The most common hypercoagulable state that is inherited currently is Factor V Leiden. Despite the fact that this patient is very highly likely to have Factor V Leiden, she can still be offered a type of contraceptive method to prevent unplanned pregnancy. The common pre-thrombotic contraceptive component is estrogen. For this reason, the patient above would be safest at beginning any method of contraception that only contains progestin. There are several types of contraception that are progestin-only: progestin-only implants, a monthly injection of only progestin, levonorgestrel based intrauterine device, and the progestin-only pills, which are also referred to as the "mini-pill." The estrogen/progestin combination oral contraception pill, the etonogestrel/ethinyl estradiol vaginal ring, the norelgestromin/ethinyl estradiol patch-transdermal, and the estradiol/medroxyprogesterone monthly injection all contain a form of estrogen, which use of this should be strictly avoided in patients such as the one described in the above scenario. References

Case A 14-year-old boy presents with bilaterally tender and swollen breasts. As of late he has become increasingly self-conscious about the condition, resulting in his avoiding physical education classes and swim team practices. He has always had good general health with no history of medical illness, hospitalizations, or medications. He does not smoke or drink alcohol, and there is no history of illicit drug use or use of any activity-enhancing products. Family history is positive for a sister and his mother being diagnosed with breast cancer. The remainder of the review of systems is non-contributory. On physical examination, he appears normal in growth and development; there are tender soft masses in the lower quadrants of both breasts, approximately 3 centimeters in diameter. The remainder of the physical examination is unremarkable. Question What is the most appropriate next step in managing this patient?

Correct answer: Reassure the patient that this is normal Explanation Based on the history and physical examination, the most likely diagnosis is pubertal gynecomastia. Breast development in boys commonly happens during adolescence and may have many possible causes. Gynecomastia is a common condition that affects between 40 and 60% of all boys/men. This breast growth can have a major emotional impact on young boys. The typical behavior is for them to hide the chest when in public due to embarrassment and/or fear of being ridiculed. Thus, they tend to avoid activities such as swimming and other activities where others can see their breasts. During growth, this mass is most frequently tender and usually bilateral. Male gynecomastia during puberty is usually secondary to the normal hormonal imbalances between testosterone and estrogen that commonly occurs during puberty, hence the name pubertal gynecomastia. It has been reported to affect up to 40% of adolescent boys, usually by the age of 14. In these cases, the breast tissue growth is usually less than 4 centimeters in diameter and will disappear without any therapeutic intervention within two years in 75% of these boys and within 3 years in 90%. Although usually normal, boys with gynecomastia should still have an evaluation by a physician, particularly if they are experiencing undue emotional distress because of it. The physician should reassure the teen that this is normal and in most cases should disappear within a few months or years without treatment. If gynecomastia is found in a boy who is less than 9 years old or has not yet started puberty, more ominous causes must be considered. Medications, drugs, and diseases can all cause gynecomastia in boys/men, and thus must be considered in your differential diagnosis. The most common medications are estrogen and anabolic steroids. Illicit drugs, such as marijuana, heroin, and alcohol, can also cause gynecomastia. Diseases that can cause gynecomastia include endocrine disorders, tumors, and genetic conditions such as Klinefelter's Syndrome. In these cases, early recognition and appropriate treatment is most important. This is not a typical finding that would be consistent with non-accidental injury; therefore, a referral to social services would be inappropriate. Referral to a breast surgeon is unjustified in this case. Needle aspiration is unjustified in this case and may even be deleterious to growing breast tissue. Short course steroid therapy is unwarranted and not indicated in cases of pubertal gynecomastia.

Question A 32-year-old woman in her 34th week of pregnancy comes to the labor floor after 2-3 hours of vaginal bleeding. Labor and vaginal delivery of her previous 3 pregnancies were normal. She is frightened but oriented. While taking her medical history, what points are relevant to cover?

Correct answer: Recent sexual intercourse Explanation You should ask about recent sexual intercourse. This patient exhibits third-trimester bleeding, which is an obstetric emergency. History, physical exams, and lab test should be performed to assess maternal and fetal well-being. The differential diagnosis consists of placenta previa, abruptio placenta, and local vaginal or cervical bleeding from trauma. In medical history, you should ask about bleeding problems and sexual intercourse, as they can cause cervical trauma. In physical examinations, Leopold's Maneuvers outline the fetal lie and indicate fetal malposition. Postural hypotension suggests major uncompensated blood loss. After the history and physical exams have been completed, measurement of blood loss and urine output should be done; a type and crossmatch is an essential precaution. At this time, amniocentesis is appropriate to look for blood in the amniotic fluid and to determine fetal lung maturity. Internal examinations must not be performed in women suspected of having placenta previa unless immediate delivery is required. The use of oxytocin challenge test in the face of third-trimester bleeding is contraindicated. The best management for this woman would be careful monitoring of vaginal bleeding. Blood should be set up in the blood bank. Cesarean section may be required later, but not at this point. The use of Ritodrine to inhibit early labor is not indicated. Magnesium sulfate is used to control preeclampsia, but it is thought that magnesium sulfate can worsen bleeding in a woman with placenta previa by preventing blood coagulation.

Case A 74-year-old multiparous woman with a history of breast cancer has been referred to a gynecologist due to experiencing a sensation of vaginal fullness and pressure, sacral back pain with standing, vaginal spotting, coital difficulty, lower abdominal discomfort, and urinary frequency and incontinence. She states that she feels "a bulge" in the lower frontal vaginal area. She denies any fever, chills, flank pain, history of infectious diseases, changes in weight, dysuria, or hematuria. The physical exam is remarkable for an obese body habitus and nontender bulge located anterior within the vaginal introitus which becomes more pronounced upon Valsalva and standing. Question What is the next most appropriate step in the management of this patient?

Correct answer: Recommend pelvic muscle exercises Explanation This patient's most likely diagnosis is most likely an anterior vaginal wall prolapse or cystocele. Nonsurgical (conservative) management of pelvic organ prolapse is recommended by both the Agency for Health Care Policy and Research and the ACOG Committee on Practice Bulletin and should be attempted before surgery is contemplated. Conservative management confers several advantages: it is safe and inexpensive, it is not usually associated with morbidity and mortality, it is minimally invasive, it can lead to a high patient satisfaction, and it may be used for patients awaiting surgery or patients who are not interested in surgical management. One such conservative measure, pelvic muscle exercises, can improve pelvic floor muscle tone and stress urinary incontinence. The purpose of anterior vaginal repair, or anterior colporrhaphy, is to plicate the vaginal muscularis fascia overlying the bladder (pubocervical fascia) to diminish the bladder and anterior vaginal protrusion. Anterior colporrhaphy is indicated especially for patients with a central vaginal defect. Vaginal support devices (pessaries) are manufactured from medical-grade silicone and are safe, cost-effective, and minimally invasive options for treating patients with pelvic organ prolapse. An important adjunct in this approach is the application of topical estrogen prior to pessary use, particularly if signs of hypoestrogenism (atrophic vaginitis) exist. Once the pessary is in place, continued vaginal estrogen cream application (i.e., 1-2 times per week) or application of a vaginal estrogen ring once every 3 months is indicated, unless estrogen is contraindicated (i.e., estrogen-dependent breast tumors). General measures also include weight loss in obese patients. Patients with defecatory dysfunction may benefit from defecating proctography and proctosigmoidoscopy. Colonic transit studies are indicated in patients with motility disorders. Dynamic MRI defecography can be used to provide defecatory and high-quality soft tissue imaging; however, this expensive test has not been shown to change clinical decision making.

Question What is the correct advice for a patient regarding folic acid supplementation in an uncomplicated pregnancy?

Correct answer: Replenished folic acid stores are associated with a reduction in neural tube defects in the fetus/infant. Explanation Folic acid (also called vitamin B9) replenishment is strongly associated with a reduction in fetal neural tube defects. Folic acid deficiency is a common vitamin deficiency; 75% of neural tube defects could be prevented by the use of folic acid supplements. Pregnancy, along with certain anemias, leukemia, and Hodgkin's disease, are conditions that are considered to increase the body's need for folic acid. The recommended daily dose of folic acid for all women of reproductive age is 0.4 mg. For high-risk women (as evidenced by history of a previous child with neural tube defect or family history of neural tube defects), 4 mg of folic acid supplementation is recommended daily. Folic acid is considered a water-soluble vitamin. Water-soluble vitamins are not stored in appreciable amounts, so supplementation and/or continued adequate folic acid intake is necessary. Folic acid supplementation is not associated with increased risk of Down syndrome. Some researchers have suggested that folic acid supplementation actually reduces the risk of Down syndrome. Though many foods are fortified with folic acid, folic acid deficiency remains common. During food preparation and processing, much of folic acid content is lost. The typical American diet is not sufficient for the folic acid levels necessary in pregnancy.

Case 4 days after delivery, a 33-year-old mother of 2 develops fever, malaise, and nausea. She is currently breast feeding her baby. There appears to be no trauma or infection to the birth canal; the uterus is involuting well. Her right breast shows a few reddish streaks radiating up from the nipple in the upper outer quadrant, and the skin appears edematous. The nipple-areola complex has some minor crusting with a few abrasions. The breast is engorged; she has stopped feeding her baby from the affected breast. The breast is tender to palpation, although it is not fluctuant. Question What is the next best step in management?

Correct answer: Restart feeding from affected breast Explanation Restarting feeding from the affected breast is the correct answer. Puerperal mastitis is common problem faced by newly breast-feeding mothers. Infection typically ascends from the nipple-areola complex through small breaches in the skin. Patients typically present with localized, unilateral breast tenderness and erythema, which are accompanied by fever, malaise, and fatigue. However, systemic signs may be absent. The key to treatment and the prevention of an abscess forming is to continue breast feeding. If feeding is stopped, the stagnant milk acts as a reservoir for multiplying bacteria, and abscess formation may ensue. Studies have shown that it does not affect the feeding baby; the mother and infant are usually colonized with the same organisms at the time mastitis develops. Incision and drainage of the abscess is an incorrect response. An abscess presents clinically with more obvious local swelling and fluctuation. In addition, systemic symptoms are the norm. Broad-spectrum IV antibiotic use is an incorrect response. Antibiotics are currently recommended for mastitis. However, narrow spectrum oral antibiotics that cover Staphylococcus, such as cloxacillin, are recommended therapy, not broad-spectrum IV drugs. Analgesia with gentle compression is an incorrect response. It may provide symptomatic relief, but compression alone does not resolve the problem. Administering bromocriptine is an incorrect response. As a dopamine agonist, bromocriptine stops lactation. It may be a possible intervention in women who refuse to breast feed and develop recurrent abscesses.

Case A 35-year-old woman presents with early pregnancy symptoms. She missed her period and experienced fatigue. She has also been experiencing morning episodes of nausea and vomiting; however, pregnancy tests performed at home and in the office are negative. Ultrasound imaging supports a partial molar pregnancy diagnosis. Additional physical examination reveals pallor, bleeding gums, conjunctive inflammation, and corkscrew hair. Question A blood test will most probably indicate low levels of what compound?

Correct answer: Retinol Explanation The answer retinol is correct. All the symptoms described can be associated with vitamin A deficiency. Vitamin A deficiency is also known to be associated with molar pregnancies. The answers folic acid and iron are incorrect. Even though skin pallor may indicate lack of either of these 2 elements, and bleeding gums can be seen in patients lacking folic acid, the other symptoms are not typically associated with iron or folic acid deficiencies. Furthermore, iron and folic acid deficiencies are not known to cause molar pregnancy. The answer protein is incorrect. Having too little protein does not cause a molar pregnancy. Symptoms induced by lack of protein (e.g., thin and easy to pull out hair, transverse lines on nails) were not observed. The answer ascorbic acid is incorrect; other symptoms in addition to bleeding gums and corkscrew hair, such as follicular hyperkeratosis, are usually associated with lack of ascorbic acid (vitamin C). Also, a lack of vitamin C is not a major factor in the onset of molar pregnancies.

Case A 28-year-old pregnant woman who is currently at 33 weeks gestation is being evaluated for contractions at labor and delivery in the hospital. Laboratory testing on vaginal secretions from this patient reveals the following: Fetal fibronectin Positive Dried secretions on glass slide Ferning present Nitrazine paper Color change of yellow to blue Question What do these tests indicate?

Correct answer: Rupture of membranes has taken place. Explanation While direct observation of fluid leaking from the cervix is the best support, the above 3 tests (positive fetal fibronectin, ferning present on microscope slide and increased pH on nitrazine paper) indicate presence of amniotic fluid and thus rupture of membranes. Fetal fibronectin is not present in normal vaginal secretions. Normal vaginal pH during pregnancy is 4.5 - 4.7.1 However, amniotic fluid is 7 - 7.5, so after rupture of membranes, an alkaline pH is noted that causes corresponding color changes in the nitrazine paper. The laboratory testing that would most aid in a diagnosis of severe chorioamnionitis includes Gram stain for organisms and a white blood cell count of the amniotic fluid. 1 Often, the diagnosis is confirmed after pathology examination of the placenta. Fetal lung maturity is best evaluated with a lecithin to sphingomyelin (L:S) ratio. Some laboratories use different values for a level corresponding to fetal lung maturity (>/= to 2.0 or >3.0). A very low L:S ratio indicates fetal lungs are immature. 1 Confirmation of estimated date of delivery (EDD) cannot be accomplished with laboratory testing. Very early in a pregnancy, human chorionic gonadotropin (hCG) levels rise in a predictive manner in normal singleton, intrauterine pregnancy. 1 However, the level of hCG can neither be used to predict nor confirm the EDD. The best methods of confirming an accurate EDD include calculation based upon known date of last menstrual period (in a woman with regular menstrual cycles) and ultrasound in the 1st trimester.2 Preeclampsia is a syndrome in pregnancy consisting of hypertension and proteinuria. The testing that indicates a diagnosis of preeclampsia includes a blood pressure measurement and evaluation of the urine for protein, preferably from a 24-hour urine collection.3 None of the above listed tests on vaginal secretions indicate preeclampsia; furthermore, they do not indicate severity level.

Case A 28-year-old woman presents with abdominal pain and vaginal bleeding. The pain began last night as a dull ache in the right lower quadrant, but this morning it became much more severe. She also complains of dizziness and nausea. She cannot recall when her last menstrual period was, but she says that she began bleeding yesterday and has a light menstrual flow. On examination, the patient is afebrile; pulse is 100/min, BP is 86/60 mm Hg, and RR is 20/min. Physical exam reveals moderate to severe tenderness in the right lower quadrant with rebound. Pelvic exam reveals a small amount of blood at the cervical os with cervical motion tenderness. You order a variety of laboratory tests on this patient. Question What is the most likely diagnosis?

Correct answer: Ruptured ectopic pregnancy Explanation Ruptured ectopic pregnancy is probably the most life-threatening condition in gynecology. Any practitioner that cares for reproductive-aged women must keep a high index of suspicion for this dangerous event in order to avoid a delay in diagnosis that may be fatal. Ectopic pregnancy is defined as extra-uterine implantation of a pregnancy; it occurs most commonly in the fallopian tube (96%). Most tubal pregnancies are found in the distal 2/3 of the tube. 2% of ectopic pregnancies are uterine ectopic pregnancies (interstitial); other sites include the cervix, ovary, or pelvic and abdominal cavities, although these implantations are rare. The major risk factors for ectopic pregnancy are a prior tubal infection or tubal surgery, a prior ectopic pregnancy (giving a risk of 10-25+% for recurrence), a history of diethylstilbestrol (DES) exposure, or presence of an IUD (intrauterine device). Ectopic pregnancy occurs in 0.5 - 1.0% of pregnancies; however, the incidence of ectopic pregnancy is increasing in recent years. This increase is probably a reflection of the increasing incidence of pelvic inflammatory disease, increased use of IUDs, and increasing incidence of tubal ligation for sterilization and other tubal surgeries. A tubal pregnancy will usually present with a history of vaginal spotting and crampy pain as early as 4 to 5 weeks after the last menstrual period (LMP). The tubal pregnancy will usually bleed slowly at first, but when the pregnancy expands to the point of tubal rupture, patients can present with rapid hemorrhage, experiencing shock and then death. On physical exam, the patient will exhibit tenderness and often peritoneal irritation, especially if blood has leaked out of the fallopian tube. The uterus will be enlarged, but it will be smaller than expected for the age of the pregnancy. There is often cervical motion tenderness, and an adnexal mass may be felt. Abdominal pain with vaginal bleeding accompanied by hypotension, marked abdominal guarding, and rebound tenderness is suggestive of ruptured ectopic pregnancy. A serum pregnancy test may reveal a smaller quantitative value than expected by LMP. If the diagnosis is in question and the patient stable, serial beta-hCG values may be estimated. In ectopic pregnancy, the serial beta-hCG does not rise appropriately with respect to the gestational age (doubling every 48 - 72 hours). However, transvaginal ultrasound is usually utilized to look for the absence of an intrauterine gestational sac and the presence of an adnexal mass, which confirms the diagnosis of ectopic pregnancy. If a patient has a beta-hCG level of 1,500 mIU per mL or greater, but the transvaginal ultrasonography does not show the presence of an intrauterine gestational sac, then ectopic pregnancy should be suspected. Culdocentesis, with the finding of old, non-clotting blood, used to be performed commonly in the assessment of tubal pregnancy, but it has largely been replaced by ultrasound because this imaging technique has increased sensitivity. An untreated ectopic pregnancy is usually fatal. Early diagnosis is the key to appropriate treatment, and the mortality rate for ectopic pregnancy in the US is currently 10-15%. The treatment for ectopic pregnancy is usually surgical, especially if there has been tubal rupture. However, some early ectopic pregnancies may be treated with methotrexate. These patients must be followed carefully with serial serum pregnancy tests and ultrasounds, and up to 1/3 of these patients will still require eventual surgical intervention. While appendicitis, salpingitis, ovarian cyst, and tubo-ovarian abscess may also present with abdominal pain, the absence of specific symptoms for each of these helps in ruling them out; symptoms of abdominal pain, vaginal bleeding, and amenorrhea are characteristic for ectopic pregnancy.

Case A 22-year-old couple presents for the evaluation of primary infertility. The woman has no symptoms; her family and gynecological history are not relevant. Her spouse regularly takes pancreatic enzymes, albuterol, and vitamins; he also frequently takes antibiotics because of the presence of recurrent respiratory tract infections, bronchiectasis, and bronchiolectasis; there is also exocrine pancreatic insufficiency and intestinal dysfunction. Question What is the next step in the evaluation of their inability to conceive?

Correct answer: Semen analysis Explanation The correct response is semen analysis. Infertility is due to the female partner 1/3 of the time, the male partner 1/3 of the time, and both partners or the unknown problems in the remaining 1/3 of cases. In this case, the husband has signs and symptoms that could cause problems conceiving. The most common causes of the male infertility are structural abnormalities, sperm production disorders, ejaculatory disturbances, and immunologic disorders. Symptoms, signs and the therapy this patient is is receiving should raise the possibility that he suffers the cystic fibrosis (CF). It is estimated that more than 95% of CF male patients are infertile. The most common findings is azoospermia due to the epididymal obstruction and/or the failure of the vas deferens to develop properly. A small minority of male CF patients are still fertile, and for that reason sperm analysis should be routinely offered in all CF patients. Sample semen is evaluated for volume, sperm count, motility and morphology. Patients with poor semen quality or numbers may be prepared for intrauterine insemination by washing and concentrating the ejaculate. Symptoms, signs, and the therapy that the husband is receiving should raise the possibility that he suffers the cystic fibrosis (CF). The diagnosis of CF rests on the combination of clinical criteria and abnormal CFTR (cystic fibrosis transmembrane conductance regulator), as documented by elevated sweat chlorde values, nasal potential difference responses, and the administration of the various solutions and by CFTR mutation analysis. Even if it is estimated that more than 95% of male CF patients are infertile, a minority with certain mutations is still fertile. For that reason, in all CF patients, sperm analysis should be routinely offered along with reproductive health education. Serial ovarian sonographic evaluations may help in demonstrating the existence of the ovulation: the development of a mature antral follicle and its subsequent collapse during ovulation. It is time consuming and is not always accurate; however, it may be helpful in supporting the diagnosis of polycystic ovary syndrome (PCOS). There is no reason to perform this test in a female patient with a non-contributiong personal and family history when the most probable cause is a husband's disease. The progesterone test is a test used to confirm ovulation; blood is drawn about 7 - 10 days after ovulation at a time when progesterone levels peak, and the level of progesterone helps in the confirmation of ovulation. There is no reason to perform this test in a female patient with a non-contributing personal and family history. Hysterosalpingogram (HSG) is a test of the patency of the Fallopian tubes and exploration of the atrial cavity. Dye is injected through the cervix and up into the uterus and Fallopian tube; its movement is observed on a video screen. There is no reason to perform this test in a female patient with a non-contributing personal and family history.

Case A 33-year-old man is evaluated for infertility. His physical examination reveals normal secondary sexual characters. There appears to be no gynecomastia. His testicles appear well developed and of normal volume. His past medical history is insignificant. He does not smoke or take any medication. Further investigation and lab tests reveal the following: Normal serum testosterone and LH levels Elevated FSH levels Semen analysis shows a low sperm count Question What is the most likely diagnosis?

Correct answer: Sertoli cell dysfunction Explanation Sertoli cell dysfunction is the correct answer. Sertoli cells are responsible for the production of androgen binding protein. Low levels of androgen binding protein result in low testosterone levels within the testes despite normal levels of production. This hampers sperm production.Tthe final consequence of which is infertility. Inhibin is the other major product of the Seroli cells. Inhibin exerts a negative feedback on the release of FSH. Due to a loss of this feedback, elevated FSH levels are seen. Klinefelters Syndrome is incorrect. A chromosomal trisomy with an XXY genotype is responsible. Clinical features include changes in body habitus, small testicles and infertility, with raised gonadotropin levels (both FSH and LH). Androgen Insensitivity Syndrome is incorrect. This results from an inability of the cells to respond to androgen stimulus, despite normal testes and circulating levels of testosterone. Due to intact production of Mullerian inhibiting factor by the testes, female internal organs (namely the ovaries, fallopian tubes, and the uterus)do not develop. However, they have female external genitalia with normal labia, clitoris, and vaginal introitus. Leydig cell dysfunction is incorrect. It would result in low testosterone levels, and high LH and FSH levels. Undescended testes is incorrect. The vignette describes the testes as being normal.

Question A 40-year-old white male patient presents to your office to ask for advice regarding the inability of his wife to conceive. She is younger than he is by eight years and is believed by him to be in good health. Additionally, she has two children from a previous marriage, and both pregnancies were without complications. He was also married once before but his first wife did not conceive children. He has not yet undergone any fertility tests. The most important factor concerning male fertility is

Correct answer: Spermatogenesis, varicocele, infections, drugs, and heat Explanation Factors involved in fertility: The male factor---Spermatogenesis, infections (prostatitis, epididymis, mumps) varicocele, heat (decrease sperm count and motility) drugs (marijuana, tobacco, alcohol). The ovarian factor---Ovulation The cervical factors---Mucus and sperm interaction The uterine factor---Endometrial integrity and cavity size and shape The tubal factor---oviductal potency and anatomic relationships to the ovary The coital factor---Insemination

Question A 22-year-old woman presents 5 weeks after having a baby. She began to nurse her baby 24 hours after the delivery. 2 days prior to presentation, she felt her left breast to be warm, hard, red, and tender. You suspect left breast mastitis. What is the most common cause of this condition?

Correct answer: Staphylococcus aureus Explanation Mastitis is parenchymatous inflammation of the mammary glands that can present at some point after lactation has begun. The most common offending organism is staphylococcus aureus from the infant's nose and throat, which enters the breast through the nipple at the site of fissure during the nursing process.

Case A 23-year-old Asian woman presents for follow-up after being seen in the emergency department. She was seen 2 months ago in the emergency department for a 3.2 cm simple ovarian cyst, which had resolved on repeat ultrasound in 6 weeks. She reports that she had severe pain with the cyst, but all of her symptoms resolved after cyst resolution. However, she had the same experience about 18 months prior to that due to another simple ovarian cyst of 3.5 cm, and her severe pain resolved after cyst resolution. She is not sexually active, and she does not use contraception. She has no chronic medical conditions, and she takes no medications. Medical records confirm her history. The physical exam is unremarkable, showing a patient of normal body habitus, without acne and hirsutism. She is very interested in preventing future recurrences of these painful ovarian cysts. Question What should be done for this patient who wishes to prevent future ovarian cysts?

Correct answer: Start her on a moderate-dose combined oral contraceptive pill Explanation This patient's described ovarian cysts are simple and related to follicular development. Moderate-dose combined oral contraceptives have been shown to reduce ovarian cysts in population studies. Although a mechanism of oral contraceptives is suppression of ovulation, birth control pills do not treatexisting ovarian cysts. Furthermore, the lower-dose regimens on the market currently do not appear to lower the overall incidence of ovarian cysts and achieve prevention of cysts. Starting this patient on a moderate-dose combined oral contraceptive pill is reasonable, especially if indicated for prevention of pregnancy or other reasons. Performing a monthly ultrasound may identify early cysts, but doing so will not prevent them. Ultrasounds often show incidental functional cysts in the ovaries of asymptomatic women. Ultrasound would be indicated in a symptomatic patient or to follow a concerning cyst to resolution. Once resolved, this imaging is neither necessary nor cost-efficient. Insertion of a levonorgestrel intrauterine system (Mirena) is indicated for prevention of pregnancy and menorrhagia. The primary effect of the progestin, levonorgestrel, is intrauterine; therefore, ovarian function is generally unaffected. A levonorgestrel intrauterine system would not reduce ovarian cyst recurrence. Folic acid supplementation has long been recommended for women with cervical dysplasia, but few studies have actually confirmed that supplementation reverses dysplasia. Furthermore, there are neither recommendations nor evidence for folic acid supplementation preventing ovarian cysts. Testing for CA-125 should be done in women with current ovarian cancer or an ovarian mass suggestive of an ovarian cancer. This patient denies current symptoms; she is young and described simple ovarian cysts, which resolved.

Case A 44-year-old woman G5P5 presents for her annual pelvic examination. Her menses are regular, and she is currently mid-cycle. She notes "leaking urine" when she coughs, sneezes, or strains. There is a bulge into the anterior vaginal wall; it is exacerbated when the patient is asked to "bear down." A urinalysis is unremarkable. There is no cervical motion tenderness or discharge noted on pelvic examination. The patient is afebrile and in no distress. No lesions are noted on the external genitalia and the pelvic examination is unremarkable except for the noted bulge. Vaginal cultures for gonococcus (GC) and chlamydia are pending. Question What is the most likely diagnosis?

Correct answer: Stress urinary incontinence Explanation The patient most likely has stress urinary incontinence as the result of a cystocele. A cystocele is a weakening and stretching of the wall between the bladder and vagina. This condition may cause leakage of urine (especially with coughing, sneezing, or exercising) or incomplete emptying of the bladder. Other names for this condition are "prolapsed bladder" or "fallen bladder." A cystocele is classified as grade 1 when the bladder is prolapsed partway into the vagina. It is grade 2 when the bladder reaches the opening of the vagina. It is grade 3 when the bladder bulges out through the vagina. A cystocele may be caused by childbirth or by excessive straining or lifting. A decreased amount of estrogen occurring during menopause may contribute to a weakening of the muscles that support the bladder and vagina. Treatment options may include avoidance of activities including heavy lifting, Kegel exercises, weight loss, a pessary (a ring-shaped device placed in the vagina that holds the bladder in place), or surgery to create support for the vagina by tightening the muscles and ligaments and repositioning the bladder. A bladder infection could cause symptoms of incontinence. This patient's urinalysis does not suggest infection, and she has not mentioned dysuria. This patient reports regular menses, so she does not meet the criteria for menopause. Menopausal women may have difficulty with incontinence as estrogen levels decrease. Estrogen helps to maintain the strength of muscles that control the bladder and keeps the lining of the vagina and urinary tract healthy. Incontinence and incomplete emptying of the bladder are both common symptoms in patients with multiple sclerosis because the disease involves a dysfunction of muscle coordination and strength. The patient does not give a history of previous symptoms suggestive of MS, but a workup may be warranted if symptoms worsen or persist without another obvious etiology. The presence of an anatomical defect suggests incontinence resulting from a cystocele in this patient. Pelvic inflammatory disease may also cause dysuria, often in association with cervical wall motion tenderness. GC and chlamydia cultures are pending; if there is a suspicion of PID, empiric treatment should be considered. There are no lesions consistent with herpes or chancroid, and the pelvic examination does not suggest the presence of an infection.

ase A 36-year-old woman, gravida 1, para 1 came to the emergency department with severe abdominal pain and 2 episodes of profuse vaginal bleeding, chills, and light-headedness 10 days after cesarean delivery. Because of her age, she was closely monitored during the pregnancy and several ultrasound examinations were performed. Today, physical examination reveals tender and firm abdomen with bloody vaginal discharge; her pulse is 100/min; blood pressure of 100/60 mmHg; and temperature 36.0 C. Laboratory reveals leukocytosis with a white blood cell count of 17,000/μL and hemoglobin level of 11 g/DL. Coagulation tests and urine beta hCG are within normal limits. Ultrasound reveals low-resistance vessel in the inner third of the myometrium. Question What is the most probable cause of her bleeding?

Correct answer: Subinvolution of the placental site Explanation Your patient has a late postpartum hemorrhage, and ultrasound shows increased myometrial vascularity; these findings are most commonly associated with retained products of conception, uterine arteriovenous malformations (options not offered in the answer choices), or subinvolution of the placental bed. Normal beta hCG excludes gestational trophoblastic disease. Endometritis is unlikely in an afebrile patient without other signs of infection. You should think about the disseminated intravascular coagulation in a patient with severe generalized hemorrhaging and laboratory findings of elevated prothrombin, PT, APTT, reduced fibrinogen and platelets, and high levels of fibrin degradation products. Her coagulation tests are normal. Leiomyomas are benign smooth muscle neoplasm. Your patient had several ultrasound examinations and a Cesarean section, and none of them revealed the presence of leiomyoma. Therefore, low-resistance vessel in the inner third of the myometrium is most probably a post-delivery event.

Case A 42-year-old gravida 1 para 1 presents with a 4-month history of menorrhagia. She is having shortened menstrual cycles that are sometimes only 15 days in length, with menstrual bleeding for 5-6 days. She is using approximately 12-14 pads or tampons per day. She admits to fatigue, headaches, and occasional dizziness, but denies syncope. There is no dysmenorrhea. A thyroid-stimulating test last month was within normal limits. Abdominal and pelvic exams are normal. Question What statement is true regarding dysfunctional uterine bleeding?

Correct answer: The condition is most common after age 40. Explanation Dysfunctional uterine bleeding (DUB) is a condition of irregular uterine bleeding in a patient who does not have an anatomic uterine lesion. It is most common above the age of 40 years (50% of the cases), but it is also seen in adolescents (20%), in whom it is associated with anovulatory cycles. Anovulatory cycles are characterized by abnormal levels of estrogen and may be due to estrogen withdrawal or breakthrough. A deteriorating ovarian follicular function is responsible for anovulatory bleeding during the climacteric. Other etiologies, such as polycystic ovarian disease, fibroids, and thyroid disease need to be ruled out before making the diagnosis. An endometrial biopsy or dilation and curettage (if the patient cannot undergo an endometrial biopsy in the office) can be diagnostic, but it is not curative or even therapeutic in a patient with DUB. Medical therapy, including estrogens, progestational agents, progesterone-impregnated intrauterine devices, and combination oral contraceptives are used to treat the condition.

ase A 21-year-old G2P1 woman presents at 10 weeks gestation with vaginal bleeding and lower abdominal cramping. She is concerned about losing her baby. Examination demonstrates bright red blood coming through a closed cervical os. Question What is the most likely diagnosis?

Correct answer: Threatened abortion Explanation The clinical picture is suggestive of a threatened abortion. About 75% of abortions occur before the 16th week of gestation. In a threatened abortion, bleeding and cramping occur, but the cervix is not dilated. The pregnancy continues. In a complete abortion, the placenta and fetus are completely expelled. In an inevitable abortion, the cervix is dilated and bleeding is present. In an incomplete abortion, mild cramps are reported, but some of the products of conception—usually the placenta—remain in the uterus. The cervical os is opened. Habitual (recurrent) abortion is defined as a loss of 3 or more pre-viable pregnancies in succession.

Case A 28-year-old woman at 24 weeks gestation presents with insomnia and fatigue. Her symptoms have worsened as the pregnancy has progressed. She has noticed dyspnea on exertion that is associated with palpitations when grocery shopping or going to the laundromat. Vital signs are blood pressure of 128/88 mm Hg, pulse is 102/minute, respirations are 16/minute, and temperature is afebrile. Weight is 138 lb, which is stable from her last visit 4 weeks ago. On physical exam, the patient is a gravid woman in no acute distress. Clinical findings include mild periorbital edema without ankle edema and brisk ankle reflexes. Examination is otherwise unremarkable. Question What will confirm the most likely diagnosis?

Correct answer: Thyroid function studies Explanation The correct answer is thyroid function studies, including a TSH and Free T4 to confirm gestational thyrotoxicosis, which is commonly due to Graves' disease. Gravid patients frequently have fatigue during the first trimester, but if the symptoms were solely due to the pregnancy, they would have most likely subsided by the second trimester. Patients with thyrotoxicosis have hyperactivity that results in insomnia; they are easily fatigued, especially with physical exertion. Typically, thyrotoxicosis presents with weight loss, but in the case of a gravid woman, a weight gain of <0.9 kg/month is inadequate at this point in the pregnancy. Sinus tachycardia is the most common arrhythmia associated with hyperthyroidism. Periorbital edema is frequently seen in Graves' ophthalmopathy with or without proptosis, and hyperreflexia is a common neurological manifestation. Urinalysis would confirm the presence of proteinuria as seen in preeclampsia. Preeclampsia may be asymptomatic or present with weight gain or edema, wherein facial edema is more specific than ankle edema. Other signs include hyperreflexia due to neurologic involvement that can progress to seizures and dyspnea; pulmonary edema can be seen in cases of severe preeclampsia. Preeclampsia is diagnosed when a gravid woman presents with a new onset of hypertension (140/90 mm Hg) in the presence of proteinuria. This diagnosis would be less likely than thyrotoxicosis because the patient's blood pressure is not elevated to this level; it would not explain the patient's sinus tachycardia. A CBC with iron studies would identify an iron deficiency anemia, which could present with fatigue, dyspnea, tachycardia, and lack of weight gain if she had poor oral intake, but it would not explain the insomnia or hyperreflexia. Palpitations, shortness of breath, rapid heart rate, and shakiness could be seen if the patient was experiencing panic attacks. A psychiatric evaluation could establish this diagnosis, but it would not explain the patient's resting tachycardia, lack of weight gain in pregnancy, periorbital edema, or hyperreflexia. A cardiac stress test would identify a cardiac abnormality, such as coronary artery disease or an arrhythmia. These could explain the dyspnea on exertion and palpitations, but would not likely be associated with periorbital edema or resting tachycardia; they would not explain the lack of weight gain in pregnancy or hyperreflexia.

Case A 16-year-old girl is very short for her age. She comes to your office for a physical examination. You notice that there is a reduced carrying angle at her elbow. You also notice that she has sparse body hair and a webbed neck. Some tests are run, and it is determined that she has the genotype 45X. Question What is the most likely diagnosis?

Correct answer: Turner syndrome Explanation Turner syndrome is a genotype 45X. Turner syndrome will therefore affect females. Characteristics of Turner syndrome include a short stature, a reduced carrying angle at the elbow, sparse body hair,and a webbed neck. Fragile X syndrome is also called the Martin-Bell syndrome. This results in intellectual disabilities. It is X-linked. People affected with Klinefelter's syndrome are males. They will have two or more X-chromosomes. Some of the characteristics of Klinefelter's syndrome include tallness, gynecomastia, sterility, and small testicles. Cri du chat is due to a deletion of the short arm of chromosome 5. Cri du chat means "cry of the cat" and refers to the distinctive cry of affected babies. In addition, there is a particular facies associated with this syndrome. Down syndrome is due to trisomy 21. Epicanthal eye folds and intellectual disability are two of the features associated with Down syndrome.

Case A 32-year-old G2P2 woman presents 1 month post-intrauterine contraceptive device placement for follow-up position check. The patient states that she can no longer feel the strings from the device. She was told to check occasionally to make sure it stayed positioned properly. Upon vaginal exam, you confirm that the strings are no longer visible in the cervical os. Question What is the most appropriate procedure to evaluate this patient?

Correct answer: Ultrasound Explanation The fastest and best method to determine the location of an IUD is an ultrasound of the uterus. This can locate the IUD even if it has eroded through the lining of the uterus. This method is inexpensive, typically available in clinic, and does not expose the patient to radiation. If the IUD has perforated the uterus, then a laparoscopy may be necessary to remove the device. Other tests are not usually performed in this situation.

Case A 45-year-old woman presents with abnormal menstrual bleeding. She states that for the past 2-3 years, she has had to deal with prolonged periods; they last around 10 days, and there is extremely heavy bleeding. As a result, she has anemia, and she becomes fatigued easily. She denies any other symptoms, and she has no other significant past medical history. Question What is the most likely diagnosis?

Correct answer: Uterine polyps Explanation Uterine polyps grow from the inner surface of the uterus and may protrude into the center. They are benign growths that are caused by cell overgrowth from the inner wall of the uterus (endometrium). The size can be anywhere from a couple of millimeters to a mass large enough to fill the entire uterine cavity. They are usually attached by a stalk and are found most often in women in their 40s and 50s. The majority of uterine polyps produce no symptoms. When symptoms do occur, they can include irregular menstrual bleeding, bleeding between periods, infertility, and bleeding after menopause. The cause is unknown, but it is believed that they are affected by hormones and they have been found to be estrogen-sensitive. Risk factors include obesity, the presence of cervical polyps, hypertension, and tamoxifen therapy for the treatment of breast cancer. They are diagnosed by ultrasound, hysteroscopy, and curettage. Treatments include watchful waiting (since they can resolve on their own), medications such as progestins and gonadotropin-releasing hormone agonists, surgical excision, and hysterectomy if they are determined to contain cancerous cells. Pelvic inflammatory disease usually presents with pain and fever. This patient has neither. Irritable bowel syndrome patients usually have gastrointestinal symptoms such as abdominal pain, bloating, flatulence, diarrhea, or constipation. Ovarian cysts, when symptoms are present, usually present with pain. They do not cause bleeding unless there is a rupture. This would be acute, as opposed to this patient's chronic symptoms. Ectopic pregnancy can present with pain on one side of the pelvis. Usually, patients report a missed period prior to any other symptoms. This patient denied any similar symptoms.

Case A 64-year-old woman presents with urine leakage; it has been progressively worsening for the past 6-12 months. She also admits to having vaginal dryness, poor lubrication with intercourse, and subsequent dyspareunia. Urinary leakage primarily seems to occur with coughing, sneezing, or high-impact exercise. She denies dysuria, hematuria, urgency, and vasomotor symptoms. She tries to empty her bladder every 2-3 hours and has cut out all alcoholic and caffeinated beverages; however, symptoms still persist. The patient is a G6P6. She has no chronic medical conditions and no past surgeries. She wants to do everything she can to avoid surgery. She is in a monogamous relationship. She was last seen 1 year ago, with a normal well-woman exam and normal Pap smear. On physical exam, vitals are normal. The pelvic exam reveals dry, pale vaginal mucosa with a bulge on the anterior vaginal wall; it increases in size with a Valsalva maneuver. Urinalysis is normal. Question Although the provider can counsel the patient that surgery is the only definitive treatment for her condition, what intervention is the most likely to improve her urinary symptoms?

Correct answer: Vaginal pessary Explanation This patient's urinary symptoms are secondary to a cystocele, which is a bulging of the bladder into the vagina. This patient's pelvic floor musculature has been weakened by delivery of 6 children and her symptoms are further aggravated by post-menopausal vaginal atrophy. Stress incontinence is one of the most common symptoms associated with having a cystocele. If she is a good candidate for surgery, the most effective treatment for this patient's cystocele is surgical repair. However, most short- and medium-term studies note improved symptoms in women treated with a pessary. After 2-4 months of pessary use, observational studies have reported resolution of prolapse symptoms (bulge, pressure, splinting) in 70-90% of women, and resolution of associated urinary symptoms (stress and urgency incontinence and voiding difficulty) in 40-50% of women. Topical estrogen cream may also help with the vaginal dryness and might provide some mild relief of urinary symptoms, but it does not address the root cause of the problem. Oral nitrofurantoin (Macrobid) is an antibiotic, which would be appropriate for treatment (or prophylaxis) of urinary tract infection (UTI). This patient does not present with typical UTI symptoms of dysuria, frequency, and urgency; in addition, she does not have an abnormal urinalysis. Oral pentosan polysulfate sodium (Elmiron) is a medication used for interstitial cystitis (IC), which is a chronic inflammatory disorder of the bladder. Classically, IC presents with nocturia, urgency, frequency, pain with a full bladder, and sometimes dyspareunia. IC is a diagnosis of exclusion. This patient has a physical exam finding of a cystocele and vaginal atrophy, which explains her symptoms. Pentosan polysulfate sodium would not be appropriate. Oral phenazopyridine (Pyridium or Azo) is a medication frequently used for relief of dysuria associated with UTI. This patient denies dysuria, so this medication would not be indicated. Oral tolterodine (Detrol) is an anticholinergic medication used for urge incontinence and overactive bladder (OAB) syndrome. This patient denies urge incontinence and OAB symptoms.

Case A 62-year-old diabetic hypertensive woman is evaluated for incontinence. She is found to have a cystocele and grade 3 uterine prolapse. Her diabetes is poorly controlled; so is her hypertension. On examination, the uterus can be repositioned with firm digital pressure. In addition to being declared unfit for surgery, she is not keen on undergoing any procedure. She has no history of any urinary infections or pelvic inflammatory disease. Question What is the next best step in management?

Correct answer: Vaginal pessary placement Explanation Vaginal pessary placement is the correct answer. Vaginal pessaries are the mainstay of conservative management of uterine prolapse. Pessaries are manually placed around the cervix, which prevents the descent of the uterus. Acute pelvic inflammatory disease and recurrent vaginitis are the only real contraindications to pessary placement. Anticholinergic agents is incorrect. Anticholinergic agents are used in the management of overactive bladders and act by reducing the bladder tone. Cholinergic nerve supply is responsible for detrusor contraction; thus, anticholinergics are useful in reducing bladder tone. Laparoscopic ventral suspension of the uterus is incorrect. While the treatment of choice for symptomatic prolapse is surgery, it is contraindicated in patients that are unfit for surgery, as is this patient. Topical estrogen creams is incorrect. It does not help uterine prolapse. Estrogen creams are useful in treating vaginal dryness and atrophic vaginitis.

A 56-year-old Asian woman presents to establish care for a wellness exam. She recently relocated from out of state; she brings a copy of her medical notes and labs from her prior medical provider. The patient tells you that sexual intercourse is very uncomfortable for her. She admits insertional dyspareunia and poor lubrication, but she denies deep dyspareunia. Outside of intercourse, she denies pelvic complaints. She denies vaginal bleeding, pruritus, or discharge. She also denies urinary complaints, such as dysuria, frequency, and incontinence. She has occasional hot flashes, but she feels they are tolerable. Her relationship is otherwise healthy, and she does not feel pressured into intercourse against her will. Her past medical history is notable for LMP 5 years ago; she is now postmenopausal. She is a G2P1. Surgical history includes open fracture reduction of the left ankle at age 22 as well as a dilatation and curettage (D&C) at age 24 for a spontaneous abortion (SAB). She is divorced, and she has been in her current monogamous relationship for 1.5 years. She is a non-drinker and non-smoker; she denies drug use. Family history is unremarkable. The table indicates laboratory tests that were done approximately 1 year ago. CBC Normal TSH Normal FSH Elevated LH Elevated Estradiol Low hCG Negative Chlamydia trachomatis Negative Neisseria gonorrhea Negative HIV Negative RPR/VDRL Negative Hepatitis panel Negative Question Based on this patient's history and lab results, what physical exam finding do you expect?

Correct answer: Vulvar and vaginal atrophy Explanation This patient would be expected to have vulvar and vaginal atrophy on her physical exam. Vulvar and vaginal atrophy occurs after menopause, when the estrogen receptors in the epithelium are no longer stimulated as a result of declining ovarian output of sex steroids. With vulvar and vaginal atrophy, the clinician will observe thinning of the labia, less rugation of the vaginal walls, as well as pale coloration with dry (and possibly friable) vaginal mucosa. This condition occurs gradually after menopause, and patients with this condition may be reluctant to discuss it. Common complaints are vaginal dryness, irritation, dyspareunia, and inadequate lubrication with sexual activity. While routine testing for menopause is not recommended, this patient's laboratory results show an elevated follicle-stimulating hormone (FSH) and luteinizing hormone (LH), with low estradiol, which is consistent with her status as postmenopausal. A thick white discharge adhering to the vaginal mucosa would be the physical exam finding expected in a woman with a vaginal yeast infection (candidiasis). Patients might complain of vaginal irritation, pruritus, and increased discharge. She may also complain of insertional dyspareunia. This patient denied the other symptoms and is clearly postmenopausal, putting her at risk for atrophy. Uterine enlargement can be seen in women with uterine fibroids (leiomyomas) or simply after having multiple pregnancies. Uterine fibroids tend to diminish in size after menopause and this patient only experienced 2 pregnancies. She would not be expected to have appreciable uterine enlargement based on her history and labs. Uterine prolapse may occur in younger women, but it is most common in postmenopausal women. The pelvic floor support and uterine ligaments are inadequate to keep the uterus in place and it descends into the vagina. Prolapse can vary in severity. When it is present, the patient may describe a feeling of vaginal fullness or that "something is falling out." Prolapse is not associated with insertional dyspareunia or poor vaginal lubrication. Yellow-green vaginal discharge indicates vaginal infection. This patient has been tested for sexually transmitted diseases with her current partner, and she currently denies vaginal discharge and pruritus. With her condition of atrophy, the physical exam will reveal less-than-normal vaginal discharge.

Case A 62-year-old obese woman presents due to urine leakage. She has had some symptoms for about 2 years, but they are getting worse. She leaks urine when she coughs or sneezes. She is not very active, but if she jumps, she also leaks urine. She wears a pad daily, as she has leakage daily. The amount varies from a few drops to a gush. The patient denies hematuria, dysuria, and pelvic pain. She sometimes feels vaginal pressure and fullness. She is considering quitting her job because she is embarrassed to be in public when she has urine leakage. Her past medical history reveals she is menopausal; she has had 4 vaginal deliveries. She has no other known medical conditions; she has not had any surgeries; she takes no medications and has no allergies. She is married and works part-time at a call center; she denies the use of tobacco, alcohol, and drugs. On physical exam, she is obese, with an atrophic vulva/vagina. The pelvic examination reveals downward and forward rotation of the vaginal wall, with an anterior bulging when the patient is asked to strain. The remainder of her exam is normal. A dipstick urinalysis is normal. Question This patient desires definitive treatment. After completing treatment of her condition, what is the most appropriate way of preventing recurrence or worsening of the condition?

Correct answer: Weight loss Explanation The correct response is weight loss. This patient is presenting with a cystocele, a herniation of the bladder wall into the vagina. Common symptoms may include stress urinary incontinence (SUI) and a feeling of vaginal fullness; other symptoms include incomplete voiding and dyspareunia. Possible approaches to cystocele treatment include surgical repair, pelvic floor physical therapy and use of a pessary. Several risk factors (such as obesity, chronic cough, multiple vaginal births, especially to large infants) have been identified with cystoceles. In this obese patient, weight loss would be recommended in order to prevent recurrence of her cystocele after treatment. Frequent straining, which increases intra-abdominal pressures, may contribute to cystoceles (and recurrence). If this patient was performing a job with frequent heavy lifting, her employment may need to be address. However, there is no reason to counsel her to avoid prolonged sitting at her job in order to prevent cystocele recurrence. (Regular physical activity should be, of course, recommended to her for general health and weight loss, as noted above.) Daily cranberry supplements are often used for urinary tract health. However, limited evidence supports cranberry's benefits related to UTI prevention. There is no evidence that cranberry juice or supplements is helpful for cystoceles and stress urinary incontinence. Daily prophylactic antibiotics can be helpful in managing patients prone to frequent pyelonephritis and/or cystitis, or urinary tract infection (UTI). Common signs and symptoms of a UTI include urinary frequency and/or hesitancy, dysuria, and (possibly) gross hematuria; urinalysis shows white blood cells, red blood cells and nitrites. Pyelonephritis may have the above signs/symptoms, along with general malaise, fever/chills, nausea, flank pain, vomiting, and costovertebral angle tenderness. The antibiotics have no role in treating cystocele; they are not useful in preventing recurrence. Oral or transdermal oxybutynin, an anticholinergic, is used for detrusor instability (overactive bladder) and resulting urge urinary incontinence. No medications have been proven effective for preventing initial or recurrence of cystocele. Without symptoms of urgency, bladder spasm, and/or urge incontinence, this medication should not be recommended to this patient.

Question You are the resident on call in the ER when a multiparous patient presents in the first stage of labor, with contractions less than one minute apart. The chief resident calls you to assist in the delivery of the child and states that "the fetal head is engaged." In reference to this phrase, which of the following is true?

Correct answer: the widest portion of the fetal head has successfully passed through the pelvic inlet Explanation When the fetal head is engaged, the widest portion of the fetal head has successfully passed through the pelvic inlet. This phrase signals that the delivery of the infant is proceeding normally, that the presentation is head first, and is not a breech presentation. After this stage, with the complete dilation and effacement of the cervix, the child will descend down the birth canal. Only at that time would the fetal head be noted to be crowning, e.g., presenting as a bulge at the perineum.

Question You are moonlighting in a clinic that provides family planning services for low-income women. The following choices represent the only pertinent historical findings for individual patients. Who would be an appropriate candidate for oral contraceptives?

orrect answer: 43-year-old African American woman taking valproic acid for seizure control Explanation The correct response is a 43-year-old African American woman taking valproic acid for seizure control. Some anticonvulsants (e.g., phenytoin and carbamazepine) decrease steroid levels in women taking birth control pills by inducing hepatic enzymes. It is not clear that this results in ovulation, but caution is advised. This effect is not seen with valproic acid. Rifampin is associated with a drastic decrease in oral contraceptive steroids, and women should not depend on oral contraceptives for birth control while taking it. Most women with controlled dyslipidemia can safely take oral contraceptives, but women with an LDL of >160 or with additional risk factors for coronary artery disease should avoid oral contraceptives. There is a statistically significant increase in the risk of ischemic stroke (two-to-threefold) in women with classic migraines who are oral contraceptive users. The use of oral contraceptives in healthy, non-smoking women over 35 is considered safe until menopause has been documented. Cigarette smokers over 35 are at increased risk of vascular events to the extent that it exceeds the risks associated with potential pregnancy.

Case A 19-year-old woman presents with vaginal pruritus, dysuria, and dyspareunia. On exam, a profuse frothy yellowish-green vaginal discharge with reddened vaginal mucosa is noted. Question What is the next step in formulating a diagnosis?

orrect answer: Evaluate vaginal discharge under microscopy. Explanation A sample of the vaginal discharge should be evaluated under microscopy, particularly for the presence of trichomonads. Yellowish-green or gray frothy vaginal discharge is a classic finding in trichomoniasis, along with the accompanying signs and symptoms of pruritus, dysuria, and dyspareunia. Although trichomonads are sometimes found on a routine Pap smear, it should not be used in diagnosing trichomoniasis. Patients should be discouraged from douching; doing so changes the vaginal flora. Penicillin will not be effective against Trichomonas vaginalis, a protozoan parasite. The dyspareunia that the patient is experiencing is likely due to the inflammation of the vaginal mucosa resulting from an acute infection, so treatment with estrogen cream is not appropriate. Once the infection is cleared, if dyspareunia is still present, further evaluation should be performed.

Case A 32-year-old woman is 2-hours status post cesarean delivery of a twin gestation at 36 weeks. Her pregnancy was uncomplicated. She presented in early labor, which became prolonged despite oxytocin infusion. A cesarean section was performed when her labor became non-progressive. In the recovery area, she begins to complain of nausea and lightheadedness. On exam, her heart rate is 133 beats per minute and blood pressure is 76/42 mm Hg. Significant vaginal bleeding is noted, and abdominal palpation reveals a soft uterus. Question What is the most likely diagnosis?

orrect answer: Uterine atony Explanation The most likely cause of bleeding in this patient is uterine atony. Uterine atony is when the uterine myometrium fails to contract following delivery. Contractions of the uterine wall after delivery normally tamponade bleeding by tightening around uterine arterioles. Absence of this response causes continued bleeding, which usually becomes evident early after delivery. Bimanual uterine palpation revealing a soft, "boggy" uterus is what typically confirms the diagnosis. Risk factors include uterine distension caused by multiple gestations or polyhydramnios, as well as prolonged oxytocin use prior to delivery, placental previa, and chorioamnionitis. Treatment involves uterine massage and the use of medications that promote uterine contractions, including oxytocin, ergot alkaloids, and prostaglandins (e.g., Hemabate). Uterine atony can also occur as a result of retained products of conception; these products inhibit uterine contraction. In this case, manual or surgical extraction is necessary. Lower genitourinary tract or rectal tears may cause bleeding after delivery, the site of which is often visible on pelvic exam and amenable to suture ligation. Neither of these conditions impairs uterine contractions.

Case A 54-year-old woman with diabetes mellitus presents with a 3-week history of vaginal spotting. An endometrial biopsy is performed and the pathology indicates endometrial cancer. The patient has a history of 12 years of amenorrhea and considers herself postmenopausal. She denies pelvic pain and cramping. She cannot identify any health changes that relate to the new vaginal bleeding. She has not seen a medical provider for preventive services for over 5 years due to a lack of health insurance. She is currently on metformin 500 mg TID; the patient admits poor compliance with the second and third doses each day. She has no known allergies, and her past medical history is significant for a cholecystectomy and tonsillectomy. The patient does not smoke or drink alcoholic beverages and denies illicit drug use. Menarche was at age 15; her menses were generally regular in her teens and 20s. She was amenorrheic while using depot medroxyprogesterone acetate, then menses returned but were irregular in frequency in her 30s. She believes her LMP to have been around age 42. She denies significant dysmenorrhea, menorrhagia, or premenstrual syndrome throughout her menstrual history. She is a G8P6Ab2, with her first child delivered at age 19; she had 6 spontaneous vaginal births with two first-trimester spontaneous abortions. She has had 9 sexual partners and has no history of sexually transmitted diseases. Previous methods of contraception included oral contraceptives (briefly) in her 20s and depot medroxyprogesterone acetate (DMPA) for 5 years in her early 30s. She discontinued the use of contraceptives in her mid-30s and has rarely been sexually active since. Question What intervention would have had the best effect in preventing this patient's development of endometrial cancer?

Correct answer: A progestin during times of amenorrhea if the patient was not already on contraceptives Explanation The correct response is administration of a progestin during times of amenorrhea if the patient was not already on contraceptives. This patient's long period of amenorrhea followed by postmenopausal bleeding is suggestive of progression from endometrial hyperplasia to endometrial cancer. Various types of hyperplasia (e.g., simple, complex, atypical simple, and atypical complex) can be treated with progestin, which inhibits and reverses endometrial hyperplasia. Depot medroxyprogesterone acetate is considered a type of progestin that will treat endometrial hyperplasia and prevent progression to cancer. Total avoidance of DMPA throughout the patient's life would not have protected her uterus. Oral contraceptives contain a progestin component that helps counter the endometrial overgrowth that leads to hyperplasia and cancer development. In fact, oral contraceptive use is considered a protective factor for the development of endometrial cancer. The avoidance of oral contraceptives in this patient may have hastened the development of her cancer. The patient's diabetes is a risk factor for the development of endometrial cancer, especially if combined with obesity, but the type of treatment for diabetes (oral medication or insulin injections) has not been clearly associated with better or worse outcomes in endometrial cancer. An improved degree of glucose control indicates a decreased risk of endometrial cancer compared to poor glucose control in a diabetic. Insulin tends to provide better glucose control if the patient is compliant with the treatment. In this patient who already admits poor compliance with oral diabetes medication (metformin), it is anticipated that her compliance would be similar or worse with insulin injections. No firm conclusion can be drawn that the type of diabetes treatment would have prevented endometrial cancer. Yearly Pap tests may or may not have detected abnormalities in pathology; the Pap is not considered a reliable screening test for detecting endometrial cancer. If atypical glandular cells of undetermined significance are detected on a Pap, then definitive evaluation (e.g., a dilatation and curettage or endometrial biopsy) should be pursued. Having regular Pap tests would not have prevented the development of endometrial cancer, but they potentially could have detected an abnormality at an earlier time.

Case A 21-year-old woman presents for a Pap smear. On speculum exam, the cervix is in a normal position. During a bimanual examination of the pelvis, the body of the uterus is unable to be palpated by the abdominal or pelvic hand. When a rectovaginal exam is performed, the body of the uterus is palpable. Question What position is this patient's uterus in?

Correct answer: A retroflexed uterus Explanation When the uterus cannot be palpated by a bimanual exam but can be felt rectovaginally, it is tilted backward. A retrodisplaced uterus wherein the cervix is normally positioned, as in this case, is in a retroflexed position. A retrodisplaced uterus with a forward-facing cervix is in a retroverted position. An anteverted uterus is a forward-facing uterus, the most common position found. In a second-degree prolapsed uterus, the cervix is in the vaginal introitus with the body of the uterus descending from its normal place in the pelvis. In a third-degree prolapsed uterus, the cervix and uterus are outside the vaginal introitus.

Case A 35-year-old woman presents for her annual Pap smear. She tells you that she and her husband are now ready to start a family, but they are worried about the hypertension the patient has been suffering from for a year. When she tells you which medication she is on to control her blood pressure, you inform her that she needs to switch to another medication if she wants to get pregnant. Question What antihypertensive drug is she likely on?

Correct answer: ACE inhibitor Explanation ACE inhibitors prevent angiotensin I from converting into angiotensin II by blocking the converting enzyme. It thereby decreases the secretion of aldosterone, which itself increases the sodium and water retention in the body. ACE-inhibitors can improve survival in patients who have suffered heart attacks and in diabetics, but they have also shown to be teratogenic. Alpha blockers inhibit the stimulating effect of noradrenalin on the smooth muscle. Side effects are tachycardia, arrhythmia, angina, and intestinal problems, like abdominal pain, vomiting, and diarrhea. Beta blockers lower the blood pressure by being sympatholytic. The administration is contraindicated in patients with COPD due to bronchospasm, in patients with diabetes due to alteration in insulin/glucose homeostasis and blockage of autonomic response to hypoglycemia, and in patients with hyperkalemia due to the risk of increasing K+ serum levels. Loop diuretics inhibit Na-K-Cl-Cotransport and thereby eliminate excessive fluid from the body. It is contraindicated in patients with gout, due to causation of hyperuricemia. Ca channel blocker decreases the tonus of smooth muscle, by inhibiting Ca entry to the cells. It is mostly used in patients with angina and hypertension and should not be given to patients with sick-sinus-syndrome, lung congestion, or after suffering a heart attack.

Case A 28-year-old African American woman presents with a 4-day history of acute abdominal pain. She tells you that the pain has been achy, sore, and sometimes has a "stabbing" nature; the pain is primarily located in her lower left abdomen/pelvis. She rates the pain at a 5/10 on a pain scale. Heat and ibuprofen have been helpful in decreasing the pain. The pain is worse with intercourse and physical activity but not affected by meals. The patient reports somewhat similar pain when she had an ovarian cyst in the past. She denies fever and chills. She also denies dysuria, urinary frequency, hematuria, nausea, vomiting, bowel changes, and breast tenderness. Last menstrual period (LMP) was 4.5 weeks ago. The patient uses condoms for contraception. She has been married and monogamous for 8 years, with no prior partners. She is nulliparous. Menses have typically been regular and without severe dysmenorrhea. Her past medical history is unremarkable, with no chronic diseases and no medications.

Correct answer: Adnexal fullness and tenderness Explanation This patient most likely has a follicular ovarian cyst, so the expected physical exam finding is adnexal fullness and tenderness, which would most likely be seen on the patient's left side. Follicular ovarian cysts are extremely common in women of reproductive age. An ovarian cyst will often present with acute abdominal or pelvic pain, sometimes with dyspareunia. If the cyst ruptures, peritoneal signs may be present. On bimanual exam, tenderness is classically present with ovarian cysts. Ovarian cysts vary in size, typically ranging from 2-5+ cm; they may or may not be appreciated on physical exam, depending on the patient's body habitus. This patient is thin, and an experienced clinician should be able to palpate a slightly enlarged left ovary as well as elicit tenderness on palpation. Cervical motion tenderness is the classic exam finding associated with pelvic inflammatory disease (PID). Patients with PID may present acutely or chronically with pelvic pain, fever, and a history of risk for sexually transmitted diseases. Patients with PID may also present with nausea, vomiting, change in vaginal discharge, dyspareunia, and dysuria. This patient is monogamous, uses condoms, and is afebrile, so PID is an unlikely diagnosis. Chadwick's sign is seen in pregnant patients and is a physical exam finding in which the vaginal mucosa and cervix are seen as blue in color; the color is due to hyperemia and pelvic congestion from increased blood flow. It is not associated with pelvic pain, and it would not be expected in a patient with a negative hCG. Frothy vaginal discharge is a characteristic finding of Trichomonas vaginalis infection. If the patient had this infection, her presentation may include dysuria, vaginal pruritus or odor, and abnormal vaginal discharge. Trichomonas infection is sexually transmitted and typically does not cause pelvic or abdominal pain. Hegar's sign is softening of the lower uterine segment, a physical exam finding of the lower uterus on bimanual exam; it is found in pregnant patients approximately 6-8 weeks after the LMP. This patient has a negative hCG, so she is not pregnant and she would not exhibit Hegar's sign.

Case A 25-year-old female single graduate student complains of a thin, malodorous vaginal discharge. A wet mount reveals many coccobacilli and many epithelial cells with indistinct cell border. Question What is the most likely etiology for the patient's vaginal symptoms?

Correct answer: Bacterial vaginosis Explanation Finding of clue cells (epithelial cells with indistinct cell margin) in vaginal smear is suggestive of bacterial vaginosis. The following four criteria are needed for the diagnosis of BV: Vaginal pH >4.5; the characteristic thin, gray, homogenous vaginal discharge; positive KOH whiff test (from amines liberated when KOH is added to a small amount of discharge); and the presence of at least four clue cells per high power field. White blood cells are not seen. Treatment may be accomplished with either metronidazole (given intravaginally or orally) or vaginal clindamycin. Trichomoniasis is a protozoal infection caused by T. vaginalis, and is also a sexually transmitted disease. Motile trichomonads must be seen on wet mount for a diagnosis to be made. Vulvovaginal candidiasis causes extreme vulvar pruritus and produces a thick, cheesy white discharge. It may be caused by three forms of Candida: C. albicans, C. glabrata, or C. tropicalis. Gonorrhea and chlamydia are both cervical infections and do not infect the vagina.

Case A 20-year-old woman presents with a malodorous vaginal discharge. She has had this discharge for approximately 2 weeks. She has been sexually inactive for the past year. She does not note any itching or abdominal/pelvic pain. Physical examination reveals a homogeneous gray discharge. There appears to be no redness or ulceration of the vulva and surrounding area. Some of the vaginal discharge is obtained and mixed with 10% KOH, producing a fishy amine odor. A Gram stain of the vaginal discharge reveals diagnostic "clue" cells. Refer to the image. Question Based on the clinical presentation and test results, what is the most likely diagnosis?

Correct answer: Bacterial vaginosis Explanation Bacterial vaginosis (nonspecific vaginitis) is a disease that is characterized by a malodorous vaginal discharge. The diagnosis of bacterial vaginosis is based on the presence of at least 3 of the 4 following signs: characteristic homogeneous gray discharge presence of "clue" cells vaginal pH greater than 4.5 the release of a fishy amine odor from vaginal secretions mixed with 10% potassium hydroxide (KOH) Gonorrhea cervicitis is caused by Neisseria gonorrhoeae and is a common sexually transmitted disease. Clinical manifestations include vaginal discharge, dysuria, intermenstrual bleeding, purulent or mucopurulent endocervical discharge, and erythema. Many women have mild or asymptomatic infections. The organism is a gram-negative diplococcus that is oxidase positive, glucose positive, sucrose negative, ONPG negative, and nitrate negative. When a gram-negative diplococcus is detected in the Gram-stained smear of discharge material, it is not diagnostic for a woman of Neisseria gonorrhoeae because there are other normal gram-negative diplococci present in the vaginal tract that are part of the normal flora. DNA probes, culture with biochemical identification, and EIA methods are used to detect and identify the organism.

ase The patient is a 26-year-old woman who presents to her gynecologist's office with a 4-month history of amenorrhea. She has had some breast tenderness, but she denies nausea, vomiting, fatigue, and abdominal pain. She was sexually active until about 6 weeks ago when she broke up with her boyfriend, but states that they used condoms. Question What choice would best describe the appearance of her cervix during an internal examination if the patient is 10 weeks pregnant?

Correct answer: Bluish appearance to cervix Explanation Bluish appearance to the cervix is the correct answer. While some pregnant women can have a normal appearance to their cervix, most will have a bluish appearance. In addition, bimanual palpation will reveal the cervix to be soft and flexible. The uterus also increases in size with gestational age and will fill the pelvis around 12 weeks. Strawberry-like appearance to the cervix is not the correct answer, as this appearance is more likely to be seen in patients who have some form of cervicitis. In particular, patients with cervicitis caused by trichomonas usually have purulent vaginal discharge and a friable cervix with punctate hemorrhages that give it a "strawberry-like" appearance. Cervical os slightly open is not the correct answer, as it is not the most likely appearance of the cervix, but could actually be seen in the cervix of a pregnant female. However, the os is usually only slightly open if the patient is experiencing a spontaneous abortion, has an incompetent cervix, or is in the early stages of labor. Purulent discharge from the cervical os is not the correct answer. If purulent discharge is seen coming from the cervical os, the likely diagnosis has something to do with an infection. The patient may have cervicitis or pelvic inflammatory disease.

Case A 52-year-old nulliparous woman presents with dyspareunia, persistent mucoid vaginal discharge, and soreness of the vaginal introitus. She has had several miscarriages, and she was told that they happened because her uterus is T-shaped. She is in a stable relationship with 1 man, has never had a sexually transmitted disease, and has no other complaints. On examination, her vaginal discharge is white and odorless; 2 bright red, fragile, hyperemic, superficial lesions are detected on the vaginal wall. The surrounding tissue initially appears normal, but fails to stain with Schiller's iodine. Biopsy of both lesions and the surrounding tissue reveals vaginal adenosis. A colposcopy, surgical excision, and cauterization of the lesion is planned. Question What other management step should be undertaken?

Correct answer: Breast cancer screen Explanation The correct response is a breast cancer screen. Being 52 years old and having vaginal adenomatosis, structural malformation of the reproductive tract, and problems with her pregnancies, this patient most probably was exposed to DES (diethylstilbestrol). In the 1950s and early 1960s, DES was prescribed for early pregnancy bleeding. The exposure to DES in utero before 18 weeks of pregnancy interferes with the growth and development of the uterus, cervix, vagina, and fallopian tubes, but many women are not aware of having been exposed. Exposure also raises the risk of otherwise rare, clear cell adenocarcinoma of the vagina and cervix. Patients may have an ectopic pregnancy, premature birth, infertility, or other pregnancy complications. They need a yearly exam (colposcopy and cytology), even after a hysterectomy or menopause. DES exposure in utero also is associated with an increased risk of breast cancer. Over age 40, the risk of breast cancer is 2 times higher than in unexposed women; after age 50, the risk is even higher. Women who took DES during pregnancy also have an increased risk of breast cancer. Both groups (mothers and daughters) should be screened by mammography. Dilation and curettage (D & C) is a diagnostic gynecological procedure, commonly performed to resolve cases of abnormal uterine bleeding or as a method of abortion.

Case A 26-year-old woman at 32 weeks gestation is brought via ambulance to the emergency department with a 30-minute history of convulsions; they occurred while she was at work. Her vital signs include a blood pressure of 160/95 mmHg, heart rate of 84 beats/min, and respiratory rate of 22 breaths/min. On physical examination, she is noted to have lower extremity edema and is hyperreflexic. She was treated with intravenous magnesium sulfate at a rate of 2 g/hour for 12 hours, and she is now is hyporeflexic; her respirations are decreased. Question What would you prescribe this patient?

Correct answer: Calcium Explanation The clinical picture is suggestive of eclampsia. In this instance, she became hyporeflexic and her respirations decreased, indicating possible magnesium toxicity. This can be reversed by giving calcium gluconate. Diazepam does not reverse the effects of magnesium toxicity. Oxygen will also not reverse the effects of magnesium toxicity. Continuing the magnesium would worsen the condition. Potassium does not reverse the effects of magnesium toxicity.

Case A 52-year-old woman presents to her gynecologist's office with a 6-month history of hot flashes, night sweats, mood swings, and vaginal dryness that interferes with intercourse. The symptoms seem to be worsening and are now interfering with her productivity at work and with her relationships with family and friends. Her LNMP was 8 months ago; the patient notes: "after that period, they just stopped." She denies any history of tobacco use, and she drinks one glass of red wine daily. She has a history of coronary heart disease (CHD) with stent placement 2 years ago. She recently read an article about hormone replacement and would like you to prescribe this for her. Question What is true regarding hormone replacement for this patient?

Correct answer: Certain antidepressants could be an alternate therapy for treatment of her menopause related mood swings. Explanation This patient is presenting with classic symptoms of natural menopause and could be a candidate for hormone replacement therapy based on the history information that is provided. CHD is not a contraindication for HRT. Some studies have shown a decrease in CHD in postmenopausal women on HRT. Estrogen therapy alone is contraindicated in this patient because her uterus is intact and estrogen that is unopposed by progesterone increases the risk of endometrial cancer. Hormone replacement will in fact diminish vasomotor symptoms, as well as vaginal dryness. Modest alcohol use is not a contraindication for hormone replacement. SSRIs would be an alternative therapy option for the treatment of this patient's mood swings. However, SSRIs would not result in relief of vasomotor symptoms or vaginal dryness.

Case A 22-year-old woman presents with high fever, prostration, vomiting. Her blood pressure is recorded at 66/42 mm Hg. A diffuse generalized macular rash is noted. She has no past medical history, except for menorrhagia. She has heavy menstrual periods and uses super absorbent tampons. Question Culture of what specimen will most likely lead to the correct diagnosis?

Correct answer: Cervical secretion Explanation The association of a severe febrile illness with signs of GI dysfunction, and a diffuse macular rash, is strongly suggestive of toxic shock syndrome. This diagnosis is especially likely if clinical abnormalities are associated with concomitant menstruation and the use of highly absorbent tampons. The clinical abnormalities are due to staphylococcal infection of the tampon, demonstrable in vaginal secretions, and the elaboration of staphylococcal exotoxins.

Question A 24-year-old female presents to the urgent care complaining of increased vaginal discharge, and she says she thinks she may have a "yeast infection." She has recently begun a new sexual relationship and is taking oral contraceptives. Physical exam reveals an abdomen that is soft and non-tender. On pelvic exam, she has a yellowish discharge at the cervical os with mild erythema and friability. She has no tenderness or masses on bimanual exam. Wet mount of the cervical discharge reveals squamous cells and scattered leukocytes. Urine pregnancy test is negative. What is the most appropriate diagnosis and treatment for this patient?

Correct answer: Cervicitis: azithromycin and ceftriaxone Explanation This patient has a mucopurulent cervicitis on physical exam, which is usually caused by Neisseria gonorrhoeae or Chlamydia trachomatis. In all practicality, a patient should be treated with antimicrobial agents that will cover both pathogens, since there is a high risk of co-infection with both pathogens (at least 50%). Patients should be tested for infection with both Chlamydia and gonorrhea before treatment, usually with an immunoassay. In addition, a Gram-stain of the cervical discharge can help elucidate the responsible pathogen. In 60% of women with gonorrhea, Gram-negative intracellular diplococci may be identified. In infections with Chlamydia, the gram-stain will reveal many leukocytes (>10/high powered field) but no gonococci. These patients should also be screened for co-infection with syphilis and possibly HIV. This patient does not have signs of salpingitis or tubo-ovarian abscess (pelvic inflammatory disease). Specifically, she has no cervical motion tenderness or peritoneal signs and no adnexal tenderness or masses. She will qualify for outpatient treatment of cervicitis and can avoid the more aggressive treatment used for salpingitis. In either condition, both of the patient's sexual partners will require medical evaluation and treatment. Additionally, if either Neisseria gonorrhoeae or Chlamydia trachomatis is identified with culture or immunoassays, infection with these pathogens must be reported to the local health department. Acceptable regimens for the treatment of gonorrheal cervicitis include one-time treatments with ceftriaxone 125 mg I.M., cefixime 400 mg P.O., ciprofloxin 500 mg P.O., or ofloxacin 400 mg P.O.. However, there is a rising incidence of resistance to the quinolones of gonorrhea in Asia and in many metropolitan areas of the U.S.; therefore, one of the cephalosporin regimens is probably a better choice. Further options include spectinomycin 2 gm IM, and other regimens of single-dose cephalosporins; although azithromycin 1 gm and ceftriaxone 125 mg IM once (by mouth) are the most commonly used. Additionally, all patients with cervicitis should receive a regimen to treat Chlamydia. These regimens include azithromycin 1 gm P.O. once, or doxycycline 100 mg P.O. twice a day for 7 days. Metronidazole 2 gm P.O. is the regimen used to treat vaginitis due to Trichomonas, not mucopurulent cervicitis. Miconazole is useful in treating vaginitis due to infection with Candida albicans but would not treat a bacterial cervicitis.

Case A 25-year-old primigravida presents at 8 weeks gestational age for her first prenatal visit. She denies any abnormal vaginal discharge or pelvic pain. On pelvic exam, you note cyanosis of the proximal vagina and cervix. Question What is this finding consistent with?

Correct answer: Chadwick's sign Explanation Due to pelvic congestion early in the first trimester, the proximal vagina and cervix appear cyanotic; such cyanosis is termed Chadwick's sign. This physical exam finding was used as a reliable sign of pregnancy in the era before rapid laboratory-based pregnancy tests were available. Acute and chronic cervicitis are histologic diagnoses, but acute cervicitis may be recognized clinically. It may present with dyspareunia, postcoital bleeding, mild pelvic pain, mucopurulent cervical discharge, and an erythematous and tender cervix. Infectious etiologies include Neisseria gonorrhoeae and Chlamydia trachomatis. Hegar's sign is seen in early pregnancy and is also related to pelvic congestion, but it is noted on bimanual exam and refers to the softening of the uterine corpus caused by the additional blood flow to the pelvis in the first trimester. A cervical ectopic pregnancy represents about 1% of all ectopic pregnancies; it may present with abnormal vaginal bleeding, pelvic pain, and dyspareunia. It is best treated with methotrexate or with hysterectomy, but fertility may be preserved in some cases.

Case A 36-year-old gravida 5, para 3013, at 34 weeks gestational age, presents with a 2-hour history of painless, bright red bleeding per vagina. She states that after urinating, she found the toilet 'filled with blood' when she stood up. There is normal fetal movement. She had 1 Caesarean section for uncertain fetal status 2 years ago. Her other 2 term pregnancies resulted in normal vaginal deliveries. She denies recent intercourse, prior history of vaginal bleeding during this pregnancy, syncope, dizziness, and headache. BP is 100/72 mm Hg, maternal pulse is 110 bpm, and respirations are 18/min. The patient is afebrile. Fetal heart rate is 160 bpm with absent variability, absent accelerations, and occasional variable decelerations; therefore, it is a category II fetal heart rate tracing. Occasional irregular contractions are seen on tocodynamometer. Question What management step should be taken immediately?

Correct answer: Complete blood count, type and hold, administration of IV fluids Explanation The most likely diagnosis for this patient with 3rd trimester bleeding is placenta previa. Obstetrical hemorrhage is still one of the most common etiologies of maternal death in the United States. In a patient with significant blood loss, attention must first be paid to determination of the degree of blood loss, volume expansion with isotonic IV fluids, and access to blood replacement if necessary. Caesarean section is not indicated at this time, as maternal complete blood count is not yet available and the fetal heart rate tracing is category 2. A biophysical profile may be of benefit in determining fetal status, but is not the most critical next step. Sonography to confirm placenta previa is more critical than the biophysical profile at this time. Amniocentesis for lecithin and sphingomyelin may be of benefit in determining whether the fetus has achieved lung maturity, but it is not the next best step in managing this emergent situation. Induction of labor and normal vaginal delivery are contraindicated in placenta previa.

Question A 20-year-old woman who was 3 months pregnant aborted a mass which resembled a bunch of grapes. The specimen that she passed is shown in Figure G3.5. Karyotype analysis of the specimen revealed that it was 46, XX. The MOST likely diagnosis is which of the following?

Correct answer: Complete hydatidiform mole Explanation A hydatidiform mole occurs once in about 2000 pregnancies. Its incidence is higher among women at the two extremes of maternal age than in other women. The serum and urinary chorionic gonadotropin levels are usually abnormally high. In complete mole, the cause is fertilization of an ovum which has lost its chromosomes. Almost all have a 46XX diploid pattern from the sperm by androgenesis. There are no fetal parts. Almost every chorionic villus is cystic and avascular. There is diffuse trophoblastic proliferation. About 2 percent develop choriocarcinoma. Partial mole is due to fertilization of an ovum by 2 sperms. It has a triploid 69XXY chromosomal pattern. Only some villi are cystic, and there is slight trophoblastic proliferation. Fetal parts may be present. Choriocarcinoma rarely develops from a partial mole. The first manifestation of the presence of a mole is often the excessively rapid enlargement of the uterus due to growth of the placental mass. This is usually followed, at about the third to fifth month of pregnancy, by uterine bleeding or abortion. Placental tissue passed at this time show that the villi are cystic and grape-like. Microscopically, the mole shows cystic avascular villi together with irregular clumps of large syncytial and cytotrophoblastic cells. Hydatidiform mole may be complete or partial. It is characterized by cystic chorionic villi. Choriocarcinoma may develop in a mole, especially the complete type. About 50 percent of gestational choriocarcinomas arise in moles. Many of these have been treated successfully with chemotherapy.

Case A 27-year-old woman presents in active labor. She is G3P2 and at 39 weeks of gestation. She has been receiving prenatal care since 6 weeks gestation, and her pregnancy has been uncomplicated. Both of her prior births were normal spontaneous vaginal deliveries. Her cervix is 6 cm, 90% effaced, mid-position, and soft. The fetus is not engaged and is thought to be vertex. Initial fetal monitoring shows a heart rate in the 140s with good accelerations, and it is reassuring. Contractions are 4 minutes apart, and she is comfortable. 20 minutes later, she experiences a large gush of clear fluid, and severe variable decelerations appear on the fetal heart rate monitor. Question What is the most likely diagnosis?

Correct answer: Cord prolapse Explanation The most likely diagnosis is cord prolapse. Cord prolapse occurs in 2 out of 1,000 deliveries. It is diagnosed when fetal monitor recordings show that severe variable decelerations or bradycardia occur after membrane rupture. The cord is often palpable in the vagina. Cord prolapse happens most often at 5-cm cervical dilatation and in nonvertex presentations. 3 types of prolapse can occur: Overt cord prolapse is diagnosed when the membranes are ruptured and the umbilical cord falls through the cervix into the vagina ahead of the fetal presenting part. Funic presentation describes loops of umbilical cord between the presenting part and the cervical os prior to rupture of membranes. Occult cord prolapse is diagnosed when the cord is palpable alongside the presenting part on digital cervical exam. Immediate delivery is essential to prevent fetal compromise. Cesarean delivery is generally preferred when the cervix is not fully dilated. Vacuum or forceps delivery may be attempted if the cervix is completely dilated, although manual elevation of the fetal part and emergent cesarean are the most common management. Uterine rupture is less likely since the patient has no history of cesarean delivery. Abruption is often associated with tetanic contractions and bleeding. An increase in uterine tone is seen between contractions. Vasa previa usually presents with painless bleeding at rupture of membranes. The cervical exam is not consistent with a diagnosis of placenta previa.

Question Which of the following is an acceptable method for diagnosing endometriosis in a patient who describes a history of increasing pelvic pain which is cyclic in nature and increases in intensity just before menses?

Correct answer: Diagnostic laparoscopy Explanation Endometriosis refers to the presence of endometrial glands and stroma implanted in a location outside of the intrauterine cavity. There are multiple theories regarding the mechanism by which this occurs, they include: retrograde menstruation, metaplasia of coelomic epithelium, and embryonic rests or remnants. It is a progressive disease and often results in significant management problems. Interestingly, the extent of the disease often does not correlate well with the perceived degree of discomfort. Although various imaging techniques have been described as being helpful in following the progression of known pelvic disease, such as endometriosis, none are considered acceptable for the actual diagnosis of the condition. CA 125 is a cell surface antigen found in tissues derived from coelomic epithelium, including gastrointestinal and gynecologic tissues. Although it can be use to monitor the progress of a diagnosed inflammatory or malignant process, it is not diagnostic and should not be used as a screening test. The current standard of care regarding the definitive diagnosis of endometriosis is laparoscopic visualization, preferably with a confirmatory biopsy of a lesion.

Case A 36-year-old woman presents with lower pelvic pain. She noticed that pain worsens during intercourse. She also reports heavy menstruation for the past 4 months. Additionally, she has lost weight over the past few months. Upon physical exam, her gynecologist did a pelvic exam and felt a mass; however, cervical motion tenderness and suprapubic tenderness was negative. A transvaginal ultrasound showed an increase of greater than 4 mm of endometrial thickness. Furthermore, a D&C was then conducted and positive for dysplasia cells. Upon laboratory findings, the patient was negative for STDs and escherichia coli. Question What is the most likely diagnosis?

Correct answer: Endometrial cancer Explanation Endometrial cancer is the correct answer because the patient presents with pelvic pain and heavy bleeding during menstruation. Additionally, a mass was found on pelvic exam. Lastly, upon a transvaginal ultrasound endometrial thickness greater than 4 mm was found. Pelvic inflammatory disease is not the correct answer, as a mass would not be found on physical exam. Additionally, upon physical exam, physical motion tenderness was negative. Urinary tract infection is not the correct answer because a patient with a UTI presents with suprapubic tenderness, which the patient does not have. Additionally, a mass would not be present with a UTI. Urethritis is not the correct answer, as urethritis is most commonly caused by N. gonorrhoeae. The patient does not have any STDs. Pyelonephritis is not the correct answer, as the patient is negative for escherichia coli, which most commonly causes pyelonephritis.

Case A 27-year-old G1 P1 woman has recently given birth by caesarean section to a 36-week male infant. She did well throughout her pregnancy up until 34 weeks gestation. At that time, she presented with fever, abdominal pain, and wetness. She was diagnosed with preterm premature rupture of membranes (PPROM) and chorioamnionitis. She was treated with steroids and antibiotics, stabilized, and she then delivered by C-section with no complications. On postpartum day 7, she presents with sore breasts from breastfeeding and a sore abdomen. She also admits to an odorous vaginal discharge, but she denies any associated vaginal bleeding. On physical examination, she is having a moderate amount of lochia alba, and her temperature is 101.2°F. Question What is the most likely diagnosis at this time?

Correct answer: Endometritis Explanation Postpartum endometritis is most likely in this patient given her history of chorioamnionitis. Prolonged rupture of membranes, cesarean delivery, prolonged labor, and multiple cervical examinations are all risk factors for postpartum endometritis. The presence of intra-amniotic infection increases the risk of postpartum endometritis further. Antibiotics are not routinely continued for chorioamnionitis after a delivery because the "source" of the infection (the placenta) has been removed. Whenever fever occurs in the immediate postpartum period, endometritis should be suspected. Broad-spectrum antibiotics should be administered promptly by the parenteral route. Similar to chorioamnionitis, multiple bacterial organisms (usually normal vaginal and perineal flora) are likely to be responsible for this infection. Therefore, uterine cultures are unlikely to be helpful in guiding antibiotic therapy. Parenteral therapy should be continued until the patient has been afebrile for at least 24 hours. Mastitis is characterized by a swollen, firm, tender breast with systemic symptoms including inflamed breast, fevers, chills, and flu-like symptoms. Staphylococcus aureus is the typical pathogen. However, in the immediate postpartum period, breast engorgement without infection is the most likely reason for the patient's sore breasts. Pregnancy and the postpartum period increase a woman's risk of thrombogenesis. However, DVT is not a likely source of the fever. Septic pelvic thrombophlebitis is a diagnosis of exclusion and is usually entertained when fever spikes continue following treatment for endometritis. Infected retained placenta is unlikely in the absence of vaginal bleeding. Lochia This is the uterine discharge following delivery and lasts for 3 or 4 weeks. Foul-smelling lochia suggests infection. Types: (1) Lochia rubra. This blood-stained fluid lasts for the first few days. (2) Lochia serosa. This discharge appears 3 to 4 days after delivery. It is paler than lochia rubra because it is admixed with serum. (3) Lochia alba. After the 10th day, because of an admixture with leukocytes, the lochia assumes a white or yellow-white color. Puerperium: This period of 4 to 6 weeks starts immediately after delivery and ends when the reproductive tract has returned to its non-pregnant condition. Multiple anatomic and physiologic changes occur during this time; the potential exists for significant complications, such as infection or hemorrhage. Puerperal infection is defined as any infection of the genitourinary tract during the Puerperium accompanied by a temperature of 100.4°F (38°C) or higher that occurs for at least 2 of the first 10 days postpartum, exclusive of the first 24 hours. Prolonged rupture of the membranes accompanied by multiple vaginal examinations during labor is a major predisposing cause of puerperal infection.

Case A 23-year-old Hispanic female delivers a healthy male child by normal vaginal delivery. She had a small episiotomy which was clean cut. She is discharged home after 48 hours but returns to the ER on the fifth post partum day with fever and chills. She says she has had foul-smelling vaginal discharge and lower abdominal pain for the last 24 hours. On exam, she has a temperature of 102° F, BP 110/60 mm of Hg, pulse 118/min, and SPO2 92%. Lungs are clear, and she has no pallor, cyanosis, or icterus. Abdominal exam demonstrates tenderness in the suprapubic area and both lower quadrants. Pelvic exam shows foul smelling purulent vaginal discharge and tender uterus. The episiotomy site is non-tender. Labs show Hb 0f 11g/dl, WBC 13,500, bands 11%, and platelets 350,000/uL. Urinalysis is pending. Question The most likely diagnosis is:

Correct answer: Endometritis Explanation This female patient is suffering from post partum endometritis as obvious by recent delivery, fever, foul-smelling vaginal discharge, and tender pelvic exam. Other risk factors for endometritis include invasive gynecological procedures, retained products of conception, intrauterine devices, submucosal fibroids, multiple sexual partners, unprotected intercourse with infected partner who may be asymptomatic, etc. Leukocytosis with bandemia supports diagnosis. Blood cultures, urine culture, and cervical culture may be done but treatment should not be delayed. Broad spectrum antibiotics to cover beta lactamase and anaerobic organisms like clindamycin with an aminoglycoside are given. Pyelonephritis is unlikely since she has no back pain or costovertebral angle tenderness, and foul vaginal discharge is absent in pyelonephritis. UA and urine culture should rule it out. Wound infections are rare in a small, clean-cut episiotomy. Fever with foul vaginal discharge does not occur, and the wound area will be obviously erythematous, indurated, and tender. Even in third or fourth degree lacerations antibiotic therapy is usually unnecessary. Cervicitis is either asymptomatic or has purulent vaginal discharge without fever, chills, tender abdominal exam, or leukocytosis. Cystitis may be associated with low grade fever and suprapubic tenderness, but dysuria is predominant and vaginal discharge absent. UA confirms diagnosis.

Case A 48-year-old Caucasian woman presents due to feeling like she is losing her mind. She wants some tests done. Upon further questioning, she reports she is having multiple episodes daily in which she suddenly becomes very hot, flushed, and diaphoretic. These episodes last about 1 minute, then resolve. She has not measured a fever. The patient reports that these episodes occur during the day and at night, the latter causing her to awaken drenched in sweat. As a result, her sleep has been poor, and she feels fatigued and irritable both at work and at home. She has noticed these symptoms for about the last 2 months, and they seem to be increasing in severity. The patient has not had a period for 3 months; she recently did a home pregnancy test, which was negative. Prior to that, she had regular menses. This patient denies weight changes, palpitations, cold intolerance, bowel changes, as well as changes in her nails, skin, and hair. Although she admits irritability, she denies anxiety, depressed mood, and suicidal ideation. Her past medical history is significant for seasonal allergies, which are relieved with over-the-counter antihistamines and taken as needed. Her surgical history includes a tonsillectomy and bilateral tubal ligation. Her family history is remarkable for diabetes in her maternal grandfather and hypertension in her father. She is a G4P3Ab1. Social history reveals the patient is an office manager for a dental clinic; she is a married nonsmoker with 3 children living at home. She denies any major psychosocial stressors recently. She drinks alcohol rarely, and she denies use of other drugs. Vitals and a urine specimen for hCG were obtained prior to the physical exam.

Correct answer: Estradiol/norethindrone acetate (Activella) 1 mg/0.5 mg by mouth daily Explanation The most effective treatment for vasomotor symptoms in perimenopause and menopause is hormone therapy. Because this patient still has her uterus, a combination of an estrogen plus a progestin should be selected. Estradiol/norethindrone acetate (Activella) 1 mg/0.5 mg daily is the most appropriate choice listed. This patient's history reveals no contraindications to the use of hormone therapy. Alprazolam would be given for panic attacks. This patient's described episodes are consistent with hot flashes and night sweats, and she denies anxiety. Alprazolam will not effectively treat vasomotor symptoms. Clonidine is an alpha-2 receptor stimulator and is used primarily for treatment of hypertension. It has mild efficacy for vasomotor symptoms and may be reasonable if there are contraindications to the use of the more effective hormone therapy. Estradiol may alleviate this patient's vasomotor symptoms, but without an "opposing" progesterone, it is contraindicated. Estrogens by themselves can overstimulate the endometrial lining, leading to endometrial hyperplasia and cancer. Vaginal use of estradiol, such as in Estrace Vaginal, is useful for treatment of vulvar and vaginal atrophy associated with menopause, but the majority of the effect is local with topical use of vaginal cream. This choice would not be effective in alleviating this patient's hot flashes and night sweats. She does not have symptoms or a physical exam consistent with vaginal atrophy (vaginal dryness/dyspareunia or pale, dry mucosa, respectively).

Case You are performing a physical examination on a 14-year-old girl. She has no complaints, no problems at school, participates in school sport activities, and is not sexually active. Her past medical history is not contributing. Her mother is worried because she still looks prepubertal and still has not gotten her period (her mother had menarche at age of 12). Her height is 5 ft; weight is 79 lbs (BMI 15,46; percentile 3%); her Tanner stage is 1 for both breast and pubic hair development; and the rest of physical examination is normal. Laboratory results are all within normal limits (CBC, ESR, LFT, and basic metabolic panel). Question At this point, what test will you choose for the evaluation of the delayed puberty in this child?

Correct answer: FSH Explanation FSH levels will help you to decide if the problem in hypothalamic-pituitary-gonadal axis (low or normal FSH) or in the ovaries (high FSH). MRI is never the best initial test, unless you suspect the presence of intracranial tumor (headache, visual problems, galactorrhea) and accompanying hormonal dysbalance. Your patient has no signs and symptoms of space occupying intracranial process, and you should wait for the results of hormonal analyses before making a decision about the future diagnostics. Karyotyping will be indicated in a patient if her FSH is high or if a patient has dysmorphism and/or developmental delay. Prolactin tests can be helpful if a patient has symptoms related to prolactinoma (headache, visual problems, galactorrhea). That is not the case in your patient. Bone age test will not reveal the cause of the delayed puberty and low body mass.

Case Ico-delete Highlights A 20-year-old Lebanese woman presents to a family practice office because she wants to start birth control. She has never been sexually active, and she is engaged to be married in 2 months. She feels well and has no complaints. She thinks she wants "the pill". Her fiancé is also a virgin, and they are not interested in condoms or other barrier methods of contraception. She wants to delay childbearing for at least 2 years.

Correct answer: Factor V Leiden Explanation Generally, no screening tests are recommended prior to starting most patients on most forms of contraceptives. However, when risk factors suggest a possible contraindication, the clinician should delve deeper and individualize testing. In this case, the patient is healthy, but her family history is concerning for a heritable form of hypercoagulability. She should be tested for a Factor V Leiden mutation. Factor V Leiden mutations are estimated to be present in approximately 5% of the Caucasian population, and it is highest in those of Lebanese ethnicity, with a prevalence of over 14% of the affected population. This mutation is the most common of heritable causes of hypercoagulability. If the patient tests positive, then she should not use estrogens in either contraceptives or menopausal hormone therapy, due to increased risk of thromboembolism. This patient should be offered alternate methods of contraception. Bleeding time is a test of platelet function, not coagulation factors. Bleeding time is useful in evaluation of patients at risk of bleeding too much, not clotting. A complete blood count (CBC) is useful in evaluating several conditions, such as anemia and infection and alterations of platelet numbers. If the clinician ordered a CBC and it returned normal, it could provide false reassurance that this patient could safely use combination hormonal contraceptives; the CBC can be entirely normal in an individual with a Factor V Leiden mutation. There is another condition of inherited hypercoagulability, familial or essential thrombocytosis, which would show up on the CBC, but this condition is very rare; also, it often presents at birth, so this patient is unlikely to have it. A complete metabolic panel (CMP) provides information on liver and kidney function, glucose levels, and electrolytes. It does not provide information related to this patient's family history of hypercoagulability. Theoretically, the CMP could be done to ensure normal liver enzymes because many of the hormonal contraceptives are hepatically metabolized. However, routine testing of liver function is not recommended prior to starting birth control. The Pap smear (short for Papanicolaou) is a test for cervical cancer and pre-cancerous dysplasia. Many years ago, it was thought that birth control increased risk of cervical cancer. That myth has been disproved. There is no compelling reason to obtain a Pap smear in order to initiate birth control. Routine cervical cancer screening guidelines should be followed, which would indicate that this patient should wait until the age of 21 years before beginning Pap testing.

Question A 29-year-old woman, G0P0, and her husband present to your office after 1 year of infertility. Their histories elicit irregular menses in the woman and that the man is an avid cyclist. Apart from semen analysis in the man, what laboratory and/or diagnostic studies should be pursued in the woman to further evaluate this couple's infertility?

Correct answer: Fasting prolactin, TSH, FSH, LH, hysterosalpingogram Explanation Fasting prolactin, TSH, FSH, and LH should all be obtained in the female patient as her irregular menses more or less indicate a high probability of oligomenorrhea. It is also important to evaluate by hysterosalpingogram to look for evidence of uterine and/or tubal defects. It is also important to evaluate the male patient's semen to investigate male infertility related to oligospermia or hypomotility (1).

Case A 32-year-old, vegetarian woman in mid-pregnancy complains of lack of energy and says she becomes easily fatigued. Upon any strenuous movement, her heart pounds rapidly and she becomes short of breath. Question Nutritional supplement of what mineral may alleviate the patient's symptoms?

Correct answer: Fe2+ Explanation Pregnant females can experience iron deficiency anemia due to increased demands on their blood. Oxygen (O2) in the lungs binds to the iron ion, Fe2+ while complexed with the heme cofactor of hemoglobin in red blood cells. Iron supplements or foods with abundant iron, such as liver, lean meats, or vegetarian alternatives such as spinach, carrots, and raisins can alleviate anemic symptoms. The other minerals have biological roles but are not associated with anemic symptoms. Magnesium (Mg2+) coordinates with the negatively charged backbone of DNA and interacts with neurotransmitter receptors at excitatory synapses in the central nervous system. Mg2+ deficiency affects the nervous system, resulting in vasodilation, tremors, and depression. Calcium phosphate forms a hard material in bone and teeth. In addition, Ca2+ is a ubiquitous second messenger ion in cellular signaling coupled to G-protein signaling, hormone signaling, and ion channel activity. Ca2+ deficiency can give rise to muscle twitching or cramping and cardiac arrhythmias. Copper (Cu2+) participates in bone and blood formation and is an electron carrier in mitochondrial electron transfer proteins. Cu2+ deficiency is uncommon since the trace amounts needed are satisfied by most diets. Zinc (Zn2+) is a cofactor of many DNA and RNA binding proteins, including many transcription factors. Severe zinc deficiency can retard growth in children, can cause low sperm count in males, and can slow wound healing.

Case A 35-year-old woman presents with a single firm, well-delineated, round, non-tender nodule in her left upper breast. It is very mobile with respect to its surrounding tissue. Question What is the most likely etiology of this mass?

Correct answer: Fibroadenoma Explanation Fibroadenomas are benign lesions of the breast which usually occur before middle-age. The following table describes the characteristics of fibroadenomas, breast carcinomas, breast cysts, fat necrosis, and tuberculous lesions. Other breast masses include: 1) galactoceles, which occur during or after lactation; 2) acute mastitis, which is generally hot, swollen, painful, indurated, and erythematous; and 3) breast abscesses, which are hot, swollen, painful, fluctuant, and erythematous.

Case A 16-year-old girl presents with a mass palpated over the left breast. There are no associated signs or symptoms noted. Menarche was at 12 years of age; it is described as regular, with moderate flow, lasting about 3 - 5 days, with occasional dysmenorrhea. At present, she is in high school with good school performance, and she is currently in the top 5 of her class. Vital signs are within normal limits. Weight and height are appropriate for age. Physical examination reveals a non-tender mass on the upper, outer part of the left breast measuring about 2x3 cm. Aspiration is done, but no fluid is aspirated. An excisional biopsy is contemplated. Question What is the most likely diagnosis?

Correct answer: Fibroadenoma Explanation The clinical picture is suggestive of fibroadenoma, which is the most common benign solid tumors containing glandular as well as fibrous tissue. They are mobile, well-defined and may be multiple. The exact etiology is unknown. but a hormonal relationship plays a role; it will persist until reproductive years, increase in size during pregnancy and estrogen treatment, and will diminish after menopause. Controversy exists with regards to the proper management of Fibroadenoma. Many believe that it has a characteristic ultrasound appearance and does not require biopsy. A reasonable approach is expectant management with ultrasound assessment since over time this fibroadenomas will regress. Regardless, many surgeons will recommend core biopsy to establish diagnosis. then advocate that no further follow-up is required. Breast Abscess is usually presented with a fluctuant palpable mass associated with systemic symptom like fever. It usually develops in 5 - 11% of women with mastitis and is commonly due to inadequate treatment. Breast pain is also common. Staphylococcus Aureus and Streptococcus are the most common etiologic agents. Management is with antibiotics and drainage if necessary. Cystosarcoma phyllodes is a low-grade malignancy in an adolescent and uncommon in an adult. It usually presents with asymmetric breast enlargement associated with a firm, mobile, circumscribed mass. This tumor can increase in size rapidly and become quite large. Excision is the initial mode of treatment in adolescents. Although most of the lesion is benign, malignant cases with metastases have also been reported. Fibrocystic diseases are multiple small lumps that usually require routine follow-up. They are diagnosed when fibrocystic changes occur, like thickening of breast tissue and cystic formation (lumpiness), nipple discharge and pain which may cause a suspicion of cancer. The cysts are usually due to blockage or dilatation of the ducts. The main goal of therapy is to alleviate pain or discomfort with the use of soft brassiere and acetaminophen or NSAIDS. Reassuring the patient that she does not have cancer is very comforting. Fat necrosis in adolescents is occasionally caused by trauma and inflammation in the form of contusion or hematoma. These later result in cystic changes or fibrosis with retraction of the skin and nipple over the injured area. These changes may mimic malignancy; therefore biopsy may be the only means of differentiating them.

Question A 24-year-old woman presents with 3 cm firm, painless, freely movable mass in her left breast. The mass does not change during her menstrual cycle and has grown slowly over the past year. The most likely diagnosis is which of the following?

Correct answer: Fibroadenoma Explanation This patient has fibroadenoma. These are the second most common form of benign breast disease. Fibrocystic changes are the most common benign breast disease. They occur mostly in young women. Fibroadenomas are firm, painless, freely movable breast masses that average 2-3 cm in diameter. These tumors do not change during the menstrual cycle and generally are slow growing. The management of these tumors is evaluation and then biopsy and excision

Case A 30-year-old woman presents with recurrent bilateral breast lumps and pain. She states that it seems to be worse during the last few days of her menstrual cycle; the lumps appear to get smaller after her cycle. She has felt different sized lumps in her breast that occur at the same time as the pain. On examination, several small, nodular lesions are noted in both breasts; they are freely movable. The axillary lymph nodes are unremarkable bilaterally. Question What is the most likely diagnosis?

Correct answer: Fibrocystic condition Explanation The clinical picture is suggestive of a fibrocystic condition; this occurs commonly in women 30-50. The patient may have an asymptomatic mass or may discover a mass while the pain is occurring. The pain usually worsens during the premenstrual cycle. Fluctuations in size and the disappearance of masses are common with this condition. Bowen's disease is a form of squamous cell carcinoma. It typically presents as well-demarcated, erythematous plaques with scaling on the skin surface; they are typically seen in sun-exposed areas. Phyllodes tumor is a fibroadenoma-like tumor. These tumors can become quite large due to their rapid growth; they may recur after excision. They commonly occur in women 40-50. Fibroadenoma is a common benign neoplasm seen in young women. The fibroadenoma is described as round, rubbery, discrete, relatively mobile, and nontender. Fibrocarcinoma is a malignant tumor of the breast. The tumor usually consists of a nontender firm or hard mass with poorly defined margins.

Case A 22-year-old woman presents due to feeling depressed, withdrawn, and irritable from 3 days prior to her menses until the day after her flow begins. She frequently misses her college classes, and she is concerned about her symptoms' potential impact on her academic performance. She is otherwise in a good state of health, and she has no history of chronic medical or psychiatric disorders. She is not taking any medication. She does not use tobacco, drink alcohol, or use illicit drugs. Menarche was at age 13, and her menses are currently regular each month. She denies cramping, bloating, and other associated physical symptoms. She is sexually active with 1 partner, and she uses condoms for contraception. Question Following prospective documentation and confirmation of her symptom pattern, what would be the single best first-line treatment for this condition?

Correct answer: Fluoxetine Explanation The clinical scenario presented here is describing premenstrual syndrome (PMS). PMS and premenstrual dysphoric disorder (PMDD) are a group of disorders and symptoms related to the menstrual cycle. At least 4 criteria must be met in order to make the diagnosis. Symptoms must: Be cyclic and have a consistent and predictable relationship to the luteal phase of the menstrual cycle, and they must be relieved within 4 days of the onset of menses. Be sufficiently severe to interfere with some aspects of life. Not be better explained by another diagnosis. Be present without drug, alcohol, or hormone use. The physiologic and the psychosocial aspects of PMS as well as the severity of the presentation must be considered when designing a therapeutic program. All treatments regimens for PMS/PMDD are ultimately aimed at alleviating symptoms and improving occupational and psychosocial functioning, leading to an enhanced quality of life. The first-line intervention of choice is prevention and includes lifestyle changes (e.g. regular exercise, a balanced diet, and avoidance of stressful situations in the premenstrual phase of the cycle). Research studies have demonstrated non-pharmacologic interventions such as nutritional supplementation (calcium, vitamin B 6 and vitamin E) to be effective This patient has classic PPMD with mood symptoms (feeling depressed, withdrawn, and irritable) predominating the clinical picture. The condition is affecting her school work performance. PPMD is considered the most severe form of PMS and is best managed by pharmacologic intervention with SSRIs such as Fluoxetine, as the serotoninergic antidepressants are currently recognized as first-line treatment of choice for PMDD with mood symptoms predominating. Calcium carbonate has been shown to be an effective non-pharmacologic option in the treatment of PMS. In this case, however, the patient appears to be presenting with classic PMDD with mood symptoms impairing her occupational performance. The best first-line treatment of choice in this case is pharmacologic intervention with SSRIs. Alprazolam is a benzodiazepine that acts on the gamma-aminobutyric acid receptor complex. This agent has been reported to be beneficial in PMS; however, because of the addictive potential, it is not a first-line therapy. Its use is reserved for patients who can be monitored reliably; when used, it should be restricted to the luteal phase of the menstrual cycle. Hormonal contraception (norgestimate plus ethinyl estradiol) is indicated for PMS physical symptoms, and it is most effective when taken continuously. These agents should not be used if mood symptoms are primary. Therefore, because she denies physical symptoms, they would not be appropriate in this patient. Leuprolide, an agonist at pituitary GnRH receptor, would suppress menstrual cycles; it is used as a treatment for PMS. However, it is associated with significant side effects. Therefore, it is not considered a first-line therapy.

ase A 15-year-old girl presents due to primary amenorrhea. She was always healthy, does not take any medications, tobacco, or drugs, and denies intercourse. She has never experienced cyclic abdominal and/or pelvic pain. Her family history is negative for gynecologic or fertility problems, autoimmune diseases, and endocrinopathies; her mother's and female relatives' menarche presented in the ages from 12-14 years. Her breasts are not budding. The rest of your examination is normal. The presence of her uterus is demonstrated by ultrasound. Question What should you obtain as your next step in the diagnosis of this patient?

Correct answer: Folliculostimulating hormone (FSH) levels Explanation Primary amenorrhea is the absence of menstruation by the age of 13 if the patient has no secondary sexual characteristics, or by the age of 15 if the patient has secondary sexual characteristics present. The absence of breasts indicates inadequate estrogen production. The initial workup includes a pregnancy test and serum luteinizing hormone, follicle-stimulating hormone, prolactin, and thyroid-stimulating hormone levels. If history or examination suggests a hyperandrogenic state, serum free and total testosterone and dehydroepiandrosterone sulfate concentrations are useful. FSH levels will show where the lesion is. If there are no functional ovaries, FSH will be high; and if there is no FSH, ovaries will not be stimulated. The next step should be obtaining the karyotype. The lack of X chromosome, which is necessary for developing ovarian follicles, will point to the gonadal dysgenesis, and low FSH will point to hypothalamic dysfunction (normal ovaries are not stimulated to produce estrogen). Autoimmune oophoritis with anti-ovarian antibodies should be considered when previous tests are normal. Patients with autoimmune oophoritis are at risk for the development of adrenal insufficiency and other autoimmune endocrinopathies (thyroid and parathyroid, diabetes mellitus, myasthenia gravis, pernicious anemia, etc.). There is no need to obtain estrogen levels; having no breasts, your patient most probably has no estrogen. The main question is why she does not have estrogen. FSH and karyotyping will help in the diagnosis. Testosterone levels should be obtained in the patient who has breasts but has no uterus in order to decide where the estrogen originates from; if testosterone levels are that of a normal female and ovaries are present, then the patient might have Mullerian agenesis (she also will have normal pubic and axillary hair and normal karyotype). If testosterone levels are that of a normal male, the estrogen source is probably the testes, as in androgen insensitivity syndrome, when a psychologically and physically female patient (with karyotype 46, XY) presents with primary amenorrhea with the lack of pubic and axillary hair and absent uterus. Total Testosterone would be a second-line test in the evaluation of primary amenorrhea, if the patient were to have been found to have an absent uterus on ultrasound. Progesterone challenge test is usually performed in the case of secondary amenorrhea when other causes are excluded (pregnancy, hypothyroidism, prolactinoma, medications). A positive test is when a single dose of progesterone or 7 days of oral medroxyprogesterone causes withdrawal bleeding, as in anovulatory cycles. A negative test will demand further evaluation with an estrogen-progesterone challenge (21 days of estrogen followed by 7 days of progesterone). A negative test will prompt endometrial problems (scars, adhesions) or outflow obstruction from other cause. Withdrawal bleeding will reveal inadequate estrogen levels, and your next step will be to find out why. This can be done by obtaining FSH levels. References McPhee S, Papadakis M, Rabow M. CURRENT Medical Dx & Tx 2011; Cha

Case Your patient is a full-term newborn with facial defects affecting her eyes, nose, and upper lips. She is the first child of non-consanguineous parents. Her mother has a history of gestational diabetes, which began at the start of her pregnancy. She contracted German measles a month before the delivery. During the course of infection, her self-prescribed daily treatment was 3 tablets of aspirin, at least 6 cups of herbal tea, and a double dose of folic acid. Imaging studies showed prosencephaly. Question What risk factor is responsible for the condition of this child?

Correct answer: Gestational diabetes Explanation The only possible risk factors during the period of the development of face and forebrain is gestational diabetes, because the forebrain is formed and the face begins to develop in the fifth and sixth weeks of human pregnancy. Other known risk factors for prosencephaly include transplacental infections, bleeding during the first trimester, a history of miscarriages, use of some drugs potentially unsafe in pregnancy (insulin, birth control pills, lithium, anticonvulsants, retinoic acid, cholesterol-lowering agents, and maternal hypocholesterolemia), and toxins (alcohol, nicotine). Rubella, or German measles, is not likely the cause of birth defects because infection occurred after organogenesis was finished. Rubella can cause miscarriage, stillbirth, or birth defects (most often deafness, brain damage, heart defects, and cataracts) if infection occurs during the first 16 weeks of pregnancy. Aspirin taken during the last trimester may increase the risk of bleeding in the newborn, but cannot cause birth defects. Herbal tea is not the cause of birth defects after organogenesis is finished. All women capable of pregnancy should take a daily vitamin supplement of folic acid to prevent neural tube defects. In the late pregnancy, however, folic acid can neither prevent nor cause birth defects of the brain and face.

Question Some temporary disease processes are unique to pregnancy and may occur in one or all of a women's pregnancies. Which of the following would raise suspicion for pre-eclampsia?

Correct answer: Headache, visual disturbances, abdominal pain, vomiting Explanation Pre-eclampsia is defined by the new onset of hypertension (>140/90 mmHg) and proteinuria measured either a 24-hour urinary protein (>300 mg/24 h) or a protein-creatinine ratio (≥0.3), after 20 weeks of gestation. The patient may exhibit the symptoms of a headache, visual disturbances, abdominal pain, and vomiting. These are often overlooked as normal pregnancy symptoms. The patient should be monitored for further symptoms. Sinus pain, chest pain, bowel disturbances may indicate the presence of an enterovirus. Chest pain, shortness of breath, and pain in the arms would raise the suspicion of cardiac problems and should not be ignored. Bowel disturbances, pain in arms, and pain in the back may indicate a herniated disk with severe complications of cauda equine syndrome. Face swelling, edema of the limbs, and renal failure may indicate Acute nephritic syndrome.

Case A 24-year-old woman presents with a 3-month history of excreting fluid from her left breast. When her symptoms first started, she had noted clear fluid staining her bra, which recurred infrequently. If she compressed the left breast tissue around her nipple area, she could easily express clear fluid. Occasionally, with difficulty, she was also able to express the same fluid from her right breast. Medical and surgical history is positive for depressive disorder; it has treated with tricyclic antidepressants for the past 4 months; she had an appendectomy 2 years ago. Her menstrual history is regular, with dysmenorrhea present. She drinks alcohol occasionally. Family history is positive for diabetes. Question What history has the most significance in regard to this patient's condition?

Correct answer: History of tricyclic antidepressant use Explanation This patient seems to have galactorrhea. Causes of abnormal lactation can be broken down into the following categories: Any disruption of production of dopamine from the hypothalamus will remove the inhibition of dopamine on the pituitary gland; this will result in increased prolactin production. Thus, craniopharyngiomas, empty sella syndrome, and anti-dopamine drugs (like metoclopramide and antipsychotics) can all antagonize the normal production of dopamine from the hypothalamus. Fluctuating hormone levels of prolactin with a relative deficiency of estrogen and progesterone; therefore, pregnant women may lactate as early as the second trimester and continue to produce milk for up to 2 years after the cessation of breastfeeding. These fluctuating levels may also cause lactation during puberty and menopause. Similarly, stimulation of the breast (suckling, chest wall disorders, thoracic nerve irritation) and nipple tissue (poor fitting bras or irritating clothes) can increase the prolactin enough to stimulate lactation. Increased levels of TRH will stimulate prolactin production. Similarly, hypersecretion of cortisol and growth hormone may have associated hyperprolactinemia. Chronic renal failure will decrease the excretion of prolactin and lead to hyperprolactinemia. Serotonin also seems to play a part in increasing prolactin and patients on selective serotonin reuptake inhibitors (SSRIs, like fluoxetine) and cyclic antidepressants (like amitriptyline and clomipramine) have reported galactorrhea. Contraceptive hormones can cause galactorrhea; this side effect seems to occur more often after discontinuation of oral contraceptive pills than during their use (similar to the withdrawal effect of precipitous drop of estrogen and progesterone that occurs after delivery). Illicit drugs (amphetamines, cannabis, opiates, etc.) can all cause galactorrhea. Similarly, a number of herbs used in supplements have been reported to cause galactorrhea (fennel, anise, etc.). Pituitary tumors, such as a pituitary prolactinoma, can cause galactorrhea. About 20% of females with galactorrhea have evidence of a pituitary tumor on testing, but this rate increases to 34% if the woman has concomitant amenorrhea. Some cancers oversecrete prolactin (e.g., renal cell cancers). In summary, the causes of galactorrhea are: Physiologic conditions (14%) Neoplastic process (Benign or malignant 18%) Hypothalamic-pituitary disorders (non-prolactinoma/under <10%) Systemic disease (<10%) Medications and herbs (20%) Chest wall irritation (<10%) Idiopathic (35%) As can be seen from this breakdown, the most common cause of galactorrhea is idiopathic; this is a diagnosis of exclusion. The history must still be completed to rule out serious causes of galactorrhea. Tests to exclude hormone etiologies include prolactin level, human chorionic gonadotropin level, thyroid-stimulating hormone level, and BUN/Creatinine. In this patient, there is history of tricyclic antidepressant use, which points to the diagnosis of pharmacological galactorrhea. A menstrual history of amenorrhea, not dysmenorrhea, would be significant; a surgical history of a chest operation, not an appendectomy, would also be significant. A family history of thyroid disorder or multiple endocrine neoplasia type I, not diabetes, as well as illicit drug use, not alcohol intake, would have more significance.

Question A 28-year-old pregnant woman at 18-weeks gestation presents because she has been exposed to fifth disease. The patient is currently asymptomatic. What can you tell her about human parvovirus B19 and pregnancy?

Correct answer: If she develops an infection, she will need to be followed with serial fetal ultrasounds Explanation Fifth disease is caused by human parvovirus B19, which is a DNA virus. Fifth disease, also called erythema infectiosum, is usually a mild exanthem of childhood, but infection of a pregnant woman can have severe fetal complications Fetal (transplacental) infection with parvovirus B19 can result in a variety of fetal complications, including fetal loss, especially if the infection is between gestational weeks 10 and 20. A common complication includes fetal hydrops; it is caused by damage to fetal hematopoietic tissue, and it causes severe anemia and a resultant congestive heart failure. The virus can also cause a fetal viral myocarditis, which further worsens cardiac function and fetal hydrops. More rarely, first trimester infections with parvovirus B19 can cause teratogenic effects, including multiorgan abnormalities. Between 30-60% of adults are immune to parvovirus B19, as evidenced by the presence of IgG to B19 in their serum; therefore, most pregnant patients are probably immune to this virus. However, if a pregnant patient develops a rash or aplastic crisis that may be consistent with fifth disease, then igG and IgM serologies for parvovirus should be drawn to evaluate for acute infection. If parvovirus B19 infection is diagnosed in a pregnant patient, then her physician may choose to follow her with serial fetal ultrasounds to evaluate for the development of fetal hydrops. In some cases, fetal umbilical cordocentesis has been used to detect fetal infection. In children, fifth disease is characterized by a classic "slapped-cheek" facial erythema; it is associated with fever and often GI or other systemic symptoms. Adult patients will demonstrate a rash. The rash may be reticular, morbilliform, or even purpuric. Adult patients often have fever, lymphadenopathy, and/or arthritis. Parvovirus B19 is also associated with an acute transient aplastic crisis. Unfortunately, fifth disease is infectious for days before the onset of the rash; thus, many obstetrical patients may be exposed, particularly if they work closely with children. The virus is spread by aerosolized respiratory droplets, and it has an incubation period of 4 to 14 days.

Case A 32-year-old woman with no significant past medical history presents with a 4-day history of vaginal discharge. She describes the vaginal discharge as thin, fairly uniform in its consistency, and of a light grayish color. Her last sexual intercourse encounter was 1 week ago; she admits that it was unprotected. She is not in a monogamous relationship. She denies any fever, chills, swollen glands, dysuria, hematuria, urinary frequency, dyspareunia, or back pain. She further denies any vulvar or vaginal pruritus. Physical exam is significant for a pungent ammonia-like scent with an associated thin, gray-white vaginal exudate, but is otherwise unremarkable. Microscopic evaluation of the vaginal exudates is remarkable for the presence of clue cells. Question What additional laboratory findings would be expected in this patient?

Correct answer: Increased vaginal pH of 6.0 Explanation This patient's signs and symptoms are characteristic of bacterial vaginosis. While the infection is polymicrobial in etiology, an overabundance of Gardnerella vaginalis and other anaerobes are causative. Increased malodorous discharge without pruritus, dysuria, or inflammatory changes is typical for the infection. Sexual contact is thought to spread the infection. Clinical criteria for diagnoses include (1) homogeneous white, noninflammatory discharge; (2) microscopic presence of > 20% clue cells; (3) vaginal discharge with pH greater than 4.5; and (4) a fishy odor with or without addition of 10% potassium hydroxide (KOH). Furthermore, Gram staining will reveal small, nonmotile, nonencapsulated, and pleomorphic rods. Red blood cell casts are not typical of bacterial vaginitis; their presence should suggest acute glomerulonephritis, lupus nephritis, SBE, Goodpasture disease, aftermath of streptococcal infection (poststreptococcal glomerulonephritis), vasculitis, or malignant hypertension.

Case A 27-year-old woman has a history of recurrent ovarian cysts; she is being treated with a combination oral contraceptive (norethindrone/ethinyl estradiol 1mg/35mcg). Question Ico-delete Highlights A 27-year-old woman has a history of recurrent ovarian cysts; she is being treated with a combination oral contraceptive (norethindrone/ethinyl estradiol 1 mg/35 mcg). The norethindrone in this drug acts to suppress ovulation by what process?

Correct answer: Inhibiting luteinizing hormone (LH) secretion from the anterior pituitary Explanation Both the estrogen and progestin components of this drug decrease gonadotropin (LH and FSH) production by negative feedback to the anterior pituitary. Norethindrone is the progestin component of the oral contraceptive, which primarily suppresses luteinizing hormone (LH) secretion (and thus prevents ovulation). The LH surge is responsible for triggering ovulation. The LH is released from the anterior pituitary and is suppressed in a negative feedback cycle when progestin levels are increased. The norethindrone would be expected to increase circulating sex hormone-binding globulin (SHBG), resulting in decreased free androgens in the serum. Norethindrone thickens cervical mucus, which inhibits sperm penetration, but the thickened mucus does not affect ovulation. The estrogenic agents in oral contraceptives (ethinyl estradiol) suppress follicle-stimulating hormone (FSH), as well as provide stability to the endometrium and potentiation of the progestin's actions. The effect on FSH is primarily attributed to the estrogen, while the effect on LH is primarily attributed to the progestin, such as norethindrone. Estrogen stimulates proliferation in the endometrial lining. Progestins, such as norethindrone, are associated with endometrial thinning and a shift to the secretory phase. Neither effect on the endometrium leads to ovulation suppression.

Case A 19-year-old woman presents to the emergency room with a 2-day history of worsening pelvic pain. She describes the pain as sharp and throughout her pelvis, and unresponsive to over-the-counter analgesics. The pain onset was gradual, and she denies trauma. She admits to some discolored vaginal discharge and mild fever. She denies vaginal pruritus or burning, dysuria, hematuria, nausea, vomiting, and bowel changes. The pain does not change with meals, urinating, or bowel movements. She experiences dyspareunia. Her past medical history is unremarkable, with no known medical conditions; she has no allergies, she takes no medications, and she has not had any prior surgeries. She denies the use of alcohol, tobacco, and drugs. Her last menstrual period was approximately 3 weeks ago. She uses no contraception, and she admits to "multiple" sexual partners. On physical exam, the patient has an oral temperature of 100.2F (37.9C); she appears to be in discomfort on the exam table, and her skin feels warm. She is diffusely tender with guarding on abdominal and bimanual exam throughout the lower abdomen/pelvis. On speculum exam, some yellowish cervical discharge is noted. Her cervix is extremely tender with movement. The remainder of her exam and vitals were normal. Initial tests are obtained, with results as shown. Complete blood count (CBC) Elevated white blood cell count (WBC) Urinalysis Normal Complete Metabolic Panel (CMP) Normal Serum hCG Negative Abdominal and pelvic ultrasounds Free pelvic fluid, which is otherwise normal Chlamydia trachomatis Pending Neisseria gonorrhoeae Pending Question What intervention is the next most appropriate step in this case?

Correct answer: Initiate broad-spectrum empiric oral or parenteral antibiotics Explanation This patient is presenting with pelvic inflammatory disease(PID). PID can be an acute or chronic condition, in which inflammation occurs throughout the female genital tract (often from sexually transmitted organisms, such as Neisseria gonorrhoeae and Chlamydia trachomatis). Symptoms can range from subtle discomfort or vaginal discharge to an acute abdomen with a septic patient. PID is diagnosed clinically, and the diagnostic criteria are quite broad, only requiring 1 or more of the following: 1) cervical motion tenderness, 2) uterine tenderness, or 3) adnexal tenderness. Unrecognized, untreated PID can lead to chronic inflammation, scarring and infertility. Multiple organisms, including aerobes and anaerobes, can contribute to PID. Therefore, the next most appropriate step is to initiate broad-spectrum empiric oral or parenteral antibiotics. The Centers for Disease Control has excellent guidelines for PID treatment, and they include at least 2 antibiotics. If PID is very low on the differential, and the patient lacks findings consistent with PID, it can be acceptable to await culture results and treat only if positive. For example, if a patient presented with abdominal and pelvic pain most likely caused by a viral gastroenteritis (and her sexual history was low-risk), empiric antibiotics for PID would not be necessary. If this patient's vaginal discharge was found to be caused by a yeast infection (vaginal candidiasis), it would be appropriate to offer prescription or over-the-counter vaginal antifungal. Classic vaginal yeast infection would present with vaginal discharge, burning and/or pruritus. The pelvic pain is not usually associated with yeast infections. A vaginal antifungal has no role in treating PID. Ectopic pregnancy can present with pelvic pain, such as this patient's. She is at high risk for pregnancy. If ectopic pregnancy could not be ruled out, it would be appropriate to refer for urgent laparoscopy. However, her serum hCG (pregnancy) test is negative and ectopic pregnancy can be effectively removed from the differential. Laparoscopy may play a role in PID, mostly when appendicitis cannot be ruled out or if there is a large abscess. It is not needed for this patient. This patient could be referred to a gynecologist, but it is not the next step in this patient's care. Referral without antibiotic treatment immediately would put this patient at risk for worsened PID and complications.

Question A 16-year-old girl presents to her primary care physician after having discovered a breast mass while bathing. She reports no symptoms of any kind; the mass was simply discovered while taking a shower. There is no history of chest trauma. Exam reveals Tanner stage IV breast development, appropriate to age, and a 2 cm mass in the upper outer quadrant of the right breast. The mass is rubbery in character, mobile, with distinctly palpable borders, non-tender, not fixed to adjacent tissue, and there is no change in surface anatomy of the breast. What is the most likely diagnosis?

Correct answer: Juvenile fibroadenoma Explanation A simple, juvenile fibroadenoma is the most common breast mass in teens. Histology involves stromal proliferation surrounded by compressed or distorted ducts. A fibroadenoma is usually asymptomatic, although a few patients will report discomfort during menses. Typically, they are discovered while bathing or performing a self-exam. Most occur in teens age 14 years or older, and there is a large increase in prevalence in 15 and 16 year olds. For most patients, only 1 is present. Exam reveals a firm, non-tender, rubbery mass that is mobile, with well-demarcated margins. Average size is 2 - 3 cm. 2/3 will be in the upper outer quadrant. The diagnosis is made by clinical exam, fine needle aspiration, core biopsy, or excisional biopsy. Treatment is surgical excision. A giant fibroadenoma displays more rapid growth, more stromal cellularity, and greater potential for large size (more than 5 cm). There is compression of adjacent breast tissue Although encapsulated, a giant fibroadenoma has the consistency of normal breast tissue, so the margins may be difficult to define. Surface anatomy changes include a sensation of warmth over the mass, due to increased blood supply, and dilation of superficial veins. Diagnosis involves the same methods as noted above. Treatment is simple excision, sparing as much normal breast tissue as possible. Phyllodes tumors, also referred to as cystosarcoma phyllodes, are of low prevalence. Histology is similar to a fibroadenoma but with more cellular, hyperplastic stroma. Affected teens present with a large, bulky breast mass, up to 20 cm. The mass is firm and mobile, and may be either smooth or irregular. Its large size causes stretching of overlying skin, with dilation of superficial veins. Skin retraction, nipple retraction, and nipple discharge may be present. The mass lacks a capsule, and it may extend into surrounding breast tissue. Biopsy establishes the diagnosis. Treatment recommendations vary: simple excision should be done if possible; mastectomy should be done if the mass is found to be malignant (rare) or is extremely large compared with surrounding breast tissue. A breast abscess is caused by bacteria introduced from the skin into ductal tissue. It is not common in adolescents. The most common pathogen is Staphylococcus aureus, followed by Escherichia coli and Pseudomonas species. Abscess formation may be preceded by trauma, ductal obstruction or a pre-existing cyst. Presentation is similar to that of any other abscess, with redness, warmth, swelling, and pain. Treatment involves warm compresses and appropriate antibiotics. Breast malignancy is reported only sporadically in girls/women under the age of 20 years. 1/3 are primary malignancies of breast tissue; the remainder arise from non-breast tissue or represent metastases from other primary sites. The breast mass is usually hard, sub-areolar, fixed to deep tissue, and of varying size. Symptoms are uncommon. General principles of oncology apply: biopsy establishes a tissue diagnosis; then, depending upon that diagnosis, treatment involves combinations of simple excision, radical excision, radiation therapy, and chemotherapy.

Case A mother brings her 16-year-old son to your medical office for a comprehensive history and physical examination. She tells you she is concerned about his immature physical development and insecure behavior. She thinks these characteristics are markedly different from her other children. His IQ is 70, and he is in special education for a language-based learning disability. On physical examination, he is tall and thin; he has sparse body hair and a high-pitched voice. Heart, lungs, abdomen, and neurologic exam are unremarkable. Pertinent positive findings include disproportionately long arms and legs, gynecomastia, as well as small testes and phallus. Question What is the most likely diagnosis?

Correct answer: Klinefelter syndrome (XXY) Explanation The combination of hypogonadism, long extremities, decreased intelligence, and behavioral problems makes Klinefelter syndrome (also referred to as XXY syndrome, 47,XXY, and Klinefelter's syndrome) the most likely diagnosis. The original syndrome, as described by Dr. Klinefelter, consisted of gynecomastia, testicular atrophy, and infertility. Intelligence profiles can range from specific learning disabilities (language learning or reading impairment most common) to intellectual disability. The only constant feature of the syndrome is testicular atrophy with resulting azoospermia and infertility. The atrophy of the testis is the result of fibrosis, which begins to appear in childhood and progresses until all the seminiferous tubules are replaced by fibrous tissue. In males presenting with gynecomastia, MR/ID, and eunuchoidism (i.e., loss of male secondary sexual characteristics, small penis, loss of body hair, and a high-pitched voice), Klinefelter syndrome should be at the top of the list in the differential diagnosis. Most patients with Klinefelter's syndrome have 47 chromosomes instead of the normal 46 chromosome karyotype. The extra chromosome is an X chromosome, making the sex chromosome constitution XXY instead of XY. Klinefelter's syndrome is one of the most common chromosome abnormalities seen in males and occurs in 1 in 300 of the male population. Patients with this syndrome show that the Y chromosome is strongly sex-determining; thus, a patient who has an XXY chromosome constitution may have the appearance of a normal male, with infertility being the only incapacity, while the loss of a Y chromosome leads to the development of a bodily form that is essentially feminine. Fragile X syndrome is incorrect, as it is characterized by prominent jaw, large ears with soft cartilage, and macroorchidism in pubertal male patients. Turner syndrome (XO) is incorrect, as it is a genetic condition of females patients; it is usually characterized by a short stature, increased distance between the nipples, low hairline, low set ears, a webbed neck, amenorrhea, and sterility. Triple X syndrome (XXX) is incorrect. This condition only occurs in female patients. XYY syndrome (XYY) is incorrect. In this condition, IQ is normal, and there is normal sexual development as well as normal fertility.

Case A 33-year-old gravida 3, para 1011, whose last menstrual period (LMP) was 6 weeks ago, presents with a 3-day history of vaginal bleeding as well as intermittent, non-radiating right lower quadrant pain. She states that her pain scale is a 7/10 and that the pain is worse today than it has been previously. Her past medical history is significant for gonorrhea at age 16; she was treated with antibiotics. Vital signs are: BP, 80/42 mm Hg; pulse is 120 BPM, and respirations are 22/min. The patient is afebrile, and oxygen saturation is 95% on room air. The abdomen is distended, and bowel sounds are absent. There is rebound tenderness and guarding. Pelvic exam demonstrates a mass in the right adnexa. The uterus is approximately 4 - 6 weeks size. A serum beta human chorionic gonadotrophin (HCG) level is 3,723 mIU/ml. Hematocrit is 24%. Ultrasound demonstrates free fluid in the pelvis, and there is a 6 cm complex mass in the right adnexa. There is no evidence of an intrauterine gestation. Question What is the most appropriate definitive treatment for the patient at this time?

Correct answer: Laparoscopy Explanation The patient has signs and symptoms consistent with ectopic pregnancy. Approximately 2% of all pregnancies are ectopic pregnancies, and most are located in the Fallopian tube. The triad of amenorrhea, vaginal spotting, and unilateral adnexal pain is strongly suggestive of an ectopic pregnancy. This patient is hemodynamically unstable. Such patients should undergo laparoscopy as soon as is possible after the presumptive diagnosis has been made. Methotrexate is an acceptable alternative to surgical management of an ectopic pregnancy, but is contraindicated when an ectopic pregnancy is ruptured, or when the adnexal mass is >3.5 cm on ultrasound. Serial quantitative beta HCG levels may be obtained every 48 hours in order to confirm the diagnosis of ectopic pregnancy; however, in this case, the patient's unstable status makes this treatment plan dangerous and unnecessary. Laparotomy is rarely needed unless laparoscopic is unsuccessful and would not be the first-line approach. Oxytocin is used to treat uterine bleeding or stimulate contractions not to treat ectopic pregnancy.

Case A 40-year-old Caucasian woman presents with excessive menstrual bleeding, as well as bleeding between her menses; these symptoms have been occurring over her last 6 cycles. She also notes associated "pressure and fullness" in her pelvis, dysmenorrhea, urinary frequency, and generalized fatigue. She denies being pregnant and has never had children. She denies any weight loss, fever, chills, chest pain, shortness of breath, abdominal pain, early satiety, nausea, vomiting, diarrhea, changes to her urine color or odor, flank pain, hematuria, or dysuria. Her physical exam is remarkable for a 'lumpy-bumpy' cobblestone sensation above the symphysis pubis, with an enlarged, mobile, and irregular contour upon bimanual palpation. No adnexal masses, cervical motion tenderness, or vaginal discharge is noted. Question What medication would be most beneficial in the initial management of this patient?

Correct answer: Leuprorelin (Lupron) Explanation The correct response is leuprorelin (Lupron). This patient's history and physical exam are most consistent with a diagnosis of uterine leiomyoma (fibroid). Pharmacologic inhibition of the growth of fibroids can be achieved by suppression of the hypothalamic-pituitary-ovarian axis through the use of gonadotropin-releasing hormone agonists (GnRH analogs) that decrease estrogen production. This reduces fibroid size by as much as 40 - 60%. It is also commonly used prior to a planned hysterectomy to reduce blood loss. Raloxifene (Evista) may help reduce fibroid growth; however, whether efficacy in reducing symptoms is comparable to that of other drugs is unclear. Clomiphene (Clomid) is indicated in the treatment of infertility, as it stimulates ovulation. Danocrine (Danazol) is an androgenic agonist and can suppress fibroid growth; however, it is not as successful as GnRH analogs and is associated with a high rate of adverse, androgenic effects (e.g., weight gain, acne, hirsutism, edema, hair loss, deepening of the voice, flushing, sweating, and vaginal dryness). Because of these side effects, this is often less acceptable to patients. Estrogen replacement agents are not suitable as treatments for uterine fibroids since uterine leiomyomas are hormonally responsive benign tumors of the uterus; they proliferate in response to estrogen.

Case A 28-year-old woman presents 3 months after giving birth to her 1st child. She is interested in discussing her birth control options. She was on birth control pills when she got pregnant, but she was not taking them as prescribed. She states that it was hard to remember to take them; her life is even busier now than it was when she got pregnant. She is married and monogamous. She is emphatic that she does not want another child anytime soon. She also states she has had dysmenorrhea since she gave birth, which is very bothersome for her. Question Based on this patient's situation, what is the best contraception option to recommend for her?

Correct answer: Levonorgestrel-releasing intrauterine device (LNgIUD) Explanation Levonorgestrel-releasing intrauterine device (LNgIUD) is the correct response. This patient's main concerns seem to be having effective contraception and having less painful menstrual cycles. Of the above options, only the oral contraceptive pills and the levonorgestrel-releasing IUD will help with her periods. Since both of these options contain hormones (either estrogen-progestin, progestin only, or levonorgestrel) they are indicated for treatment of dysmenorrhea. Also, those 2 options are the most effective forms of contraception in the list. OCPs have a pregnancy rate of 9 per 100 women taking them each year. IUDs have a pregnancy rate of <1 per 100 women using them per year. Male condoms and female diaphragms have a pregnancy rate of 18 and 12 respectively per 100 women using them each year. Spermicides have the highest rate of pregnancy at 28 per 100 women using it each year. What makes the IUD a better option than OCP in this case is the patient's previous history of not taking the pills regularly. The IUD will eliminate the human error factor in birth control, and it can be effective for at least 5 years depending on the type of IUD selected. It also is rapidly reversible if the patient changes her mind about having another baby before then.

Case A 62 year old woman presents for her annual checkup. She indicates that she has heard alot lately in the news about breast cancer screening, and has decided she wants to have a breast ultrasound as her method of screening. Question Which test do you recommend that she undergo as the best screening option?

Correct answer: Mammography Explanation The breast is the most frequent site of occurrence for cancer in women. One out of nine women will have breast cancer in their lifetime. Mammography is the most effective tool for screening the general population of women. Breast Self Examination as a screening tool is not recommended by either the American Cancer Society or the U.S. Preventive Services Task Force. Annual physical examination of the breasts by a Health Care Provider is no longer recommended either. Thermography is ineffective as a screening method. Breast Ultrasonography has a place in follow-up examination of abnormal mammograms and also in the subpopulation of women with dense breast tissue. Premenopausal women are typically those that present with dense breasts. Once a woman is menopausal breast tissue becomes much less dense due to lack of estrogen.

Case A 52-year-old woman has a 12-month history of amenorrhea. She noticed she has been having hot flashes and night sweats, loss of libido, and vaginal dryness. Upon physical exam, no abnormal findings are found except some dry skin. Upon lab results, elevated serum FSH level >30 mIU/mL. Her 24-hour urinary free cortisol is 11μg/24 hr, within the normal range of 10 -100 μg/24 hr. Her serum IGF-1 level is in the normal range for a 42-year-old (90 to 360 ng/mL). Additionally, her levels of ACTH are within normal limits. Question What is the most likely diagnosis?

Correct answer: Menopause Explanation Menopause is the most likely diagnosis due to her age, amenorrhea over 12 months, vaginal dryness, mood swings, night sweats, and lab results. Polycystic ovary syndrome is not the most likely diagnosis, as FSH levels would be normal or low; however, this patient has elevated FSH levels. Pituitary adenoma is not the most likely diagnosis since the patient's 24-hour urinary free cortisol 11 μg/24 hr is within normal range. Acromegaly is not the most likely diagnosis, as her Serum IGF-1 is within normal limits as well. If the patient had acromegaly, the patient's Serum IGF-1 would be twice the upper limit. Cushing's syndrome is not the most likely diagnosis because a patient with Cushing's syndrome would have an elevated level of ACTH, and this patient is within normal limits.

Case Ico-delete Highlights A 50-year-old woman presents for an annual pelvic examination. She states her last menstrual period was over 6 months ago; the last few occurrences of menses were extremely irregular. The patient also describes having the sensation of intense heat in her face and trunk, and the sensation accompanied by sweating. She also states that her "heat episodes" have been occurring 1 or 2 times a week for the last several months. She has no other symptoms at this time. She has received her annual pap and pelvic examination yearly, as well as a clinical breast exam, without any issues. During the pelvic examination, you note obvious vaginal thinning, excessive dryness, and apparent vaginal wall atrophy. Question Based on the history and physical examination findings, what is most likely occurring in this patient?

Correct answer: Menopause Explanation The patient in this case is most likely experiencing menopause. Menopause is defined as a cessation of menstruation due to either natural aging or an external cause (surgical). There is usually a 1 - 3 year time period in which women will typically adjust physiologically to the diminished hormonal and menstrual actions as well as the effects they have on their body. These effects may include hot flashes, night sweats, vaginal dryness, and in the later stages, osteoporosis. In western societies, the average age at which a woman experiences menopause is roughly 51 years old. Ovarian failure is also referred to as premature menopause; it is defined as the cessation of menses before the age of 40, which does not fit the patient's case. Pregnancy is a possibility in this patient, but it is not as likely. Cervical cancer is not likely because this patient has cooperatively obtained her yearly pelvic exams without any significant abnormalities from these visits. Polycystic ovarian syndrome is another disease state that has a main symptom of amenorrhea; however, it typically presents in women younger than this case's patient. She has never had this diagnosis before, and it is usually found in women in their childbearing years who may be having infertility issues due to the hormonal dysfunction that occurs in it.

ase Your patient is a 35-year-old nullipara in the 19th week of gestation. She feels well; her laboratory findings and physical examination are unremarkable, except that her blood pressure is constantly around 165/105 (values measured at home were in average 160/100). Her lifestyle is healthy, and she has been on the recommended diet for about 2 months. Question Regarding the hypertension, what treatment will you most likely advise?

Correct answer: Methyldopa Explanation Hypertension is the most common medical problem in pregnancy. Your patient most probably has chronic hypertension, defined as blood pressure exceeding 140/90 mmHg before pregnancy or before 20 weeks gestation. Diastolic blood pressure greater than 110 mmHg is associated with an increased risk of placental abruption and intrauterine growth restriction, and systolic blood pressure greater than 160 mmHg increases the risk of maternal intracerebral hemorrhage. Therefore, if maternal blood pressure exceeds 160/100 mmHg, as is the case in your patient, drug treatment is recommended because the benefit of drug therapy may outweigh the risks. Commonly used drugs for hypertension in pregnancy include methyldopa, labetalol, and calcium channel blockers. The goal of pharmacologic treatment should be a diastolic blood pressure of less than 100-105 mmHg and systolic less than 160 mmHg. Your patient has blood pressure that mandates the use of pharmacological treatment. In normal pregnancy, women's mean arterial pressure drops 10-15 mmHg over the first half of pregnancy. Most women with mild chronic hypertension (i.e., systolic blood pressure 140-160 mmHg, diastolic 90-100 mmHg) have a similar decrease in blood pressures and may not require any medication during this period because pharmacologic treatment of mild to moderate hypertension does not reduce the likelihood of developing preeclampsia later in gestation and does not improve pregnancy outcomes (except in the case of hypertensive crisis). It actually increases the likelihood of intrauterine growth restriction. Magnesium sulfate is used to prevent eclampsia and to treat eclamptic seizures. Your patient is in her 19th week of gestation and does not satisfy criteria for preeclampsia (the new onset of elevated blood pressure readings after 20 weeks gestation). Even for very high blood pressure in pregnancy, magnesium sulfate should be avoided. ACE inhibitors should be avoided during pregnancy because they are associated with fetal renal dysgenesis or even death when used in the second and third trimesters, as well as with increased risk of cardiovascular and central nervous system malformations when used in the first trimester. Angiotensin II receptor antagonists/blockers are not used during pregnancy, because they have a mechanism of action that is similar to that of ACE inhibitors. Diuretics do not cause fetal malformations but are generally avoided in pregnancy, as they prevent the physiologic volume expansion seen in normal pregnancy. They may be used in states of volume-dependent hypertension, such as renal or cardiac disease. Your patient has no signs of those diseases.

Question During a routine follow-up of a 28-year-old G2 P1 woman at 24 weeks gestation, you note a blood pressure of 150/105 mmHg. You indicate that you are concerned, and you ask her to follow up in 1 week. On revisit, her blood pressure is 154/104 mmHg. This warrants antihypertensive medication use for the duration of her pregnancy. What antihypertensive agent is recommended in this patient?

Correct answer: Methyldopa Explanation Pregnant women with hypertension require medication only if their diastolic pressure is sustained at or above 100 mmHg. Because of its safety record, initial treatment includes methyldopa. Propranolol may be used, but it can cause growth restrictions. Captopril is contraindicated during pregnancy. Nifedipine can be used, but it should be avoided due to reflex tachycardia. Clonidine should not be used as a result of the potential of rebound hypertension with abrupt discontinuation.

Case A 13-year-old girl presents with a 3-day history excessive vaginal discharge without itching or burning. She denies ever being sexually active. Her last menstrual period was 10 days ago. On examination you find a thin, white, homogeneous discharge that has a distinct amine odor when potassium hydroxide is added. On saline wet mount, epithelial cells are covered with bacteria. Question What is the most appropriate management of this patient?

Correct answer: Metronidazole Explanation This patient has the classic signs of bacterial vaginosis (BV), including the thin discharge with the fishy odor on KOH prep, and clue cells on wet prep. Standard treatment of BV is metronidazole 500 mg po bid for 7 days. Trichloroacetic acid is used for the treatment of genital warts. Clotrimazole, fluconazole, and nystatin are all used in the treatment of candidiasis.

Case A 25-year-old woman has come in to see you today for her annual gynecological visit. You review her history. Menses onset was at the age of 12 years old, and the duration of menses is typically around 6 days in length and occurs every 30 days. She is G0P0 and has no history of abnormal pap smears or diagnosed STIs. The patient is a non-smoker, single, and has been in a monogamous relationship with 1 partner for the past year. A pertinent positive the patient mentions is a whitish gray vaginal discharge that increases after intercourse and is accompanied by a distinct musty odor; she denies any pain from this discharge. Physical examination and a positive result of a whiff test solidify your suspicions. Question Based on the findings so far, what would be the most appropriate therapy for the patient at this time?

Correct answer: Metronidazole 500 mg BID PO for 7 days Explanation The patient above is highly likely to be experiencing cervicitis caused by the disease state known as bacterial vaginosis. Many times this is considered a polymicrobial disease state and frequently is not viewed as a sexually transmitted disease. Its main etiology is an abundant overgrowth of Gardnerella and other anaerobes. These organisms produce a malodorous discharge that has a characteristic grayish to frothy appearance to it. An amine-like ("fishy") odor is created if the discharge is alkalinized with 10% potassium hydroxide (positive "whiff test.") On wet mount, the characteristic 'clue cells' will also help the healthcare provider confirm the diagnosis. Appropriate treatment for this diagnosis would be the Metronidazole 500 mg PO BID for 7 days. A patient with a Candida infection will have a malodorous, white curd-like discharge. It will also exhibit extreme pruritus and vulvovaginal erythema. A microscopic examination with potassium hydroxide will reveal both hyphae and spores. Both Miconazole and Fluconazole are treatment options for a patient with a confirmed Candida infection. Trichomoniasis cervicitis is known as "strawberry cervix" when examined by the healthcare provider. This type of pathology will have diffuse vaginal erythema. It accompanies red macular lesions as well as possible punctate hemorrhages that are identified on the cervix. It produces a malodorous frothy, yellow-green discharge as well as pruritus. Microscopic examination will reveal motile protozoan organisms with flagella. Tinidazole would be used if the patient were suspected of having this diagnosis. Doxycycline would be considered part of the treatment regimen in a patient who has pelvic inflammatory disease, which is also known as salpingitis/endometritis.Signs and symptoms may include lower abdominal pain, chills, fever, menses disturbances, purulent cervical discharge, and exquisite cervical and adnexal tenderness to palpation. This pathology is more common in nulliparous, young, sexually active women who have a history of multiple partners.

Case A 58-year-old woman presents with pelvic pressure and a bulging sensation in her vaginal region. During her pelvic examination, the Valsalva maneuver results in prolapse of the uterus. Question What risk factor is most closely related to the patient's diagnosis?

Correct answer: Multiparous Explanation The correct response is multiparous. Women who have carried children, and specifically those who have delivered children vaginally, are at a higher risk for uterine prolapse. About 30-50% of women experience some degree of uterine prolapse, which predominately occurs during the perimenopausal or postmenopausal period. People of West African or South African heritage experience uterine prolapse very infrequently. Caucasians have the highest incidence; uterine prolapse is less commonly noted among Asians and African Americans. Thin body habitus is not the correct response since conditions that result in an increased intra-abdominal pressure put patients at risk for uterine prolapse. Patients who are obese have increased strain on their pelvic floor and are at an increased risk for uterine prolapse. Abdominal or pelvic tumors, chronic constipation, and chronic coughing secondary to pulmonary disease are other disorders that can result in increased intra-abdominal pressure and uterine prolapse. There are few occupations that would increase a woman's risk of uterine prolapse, but any occupation that would include heavy lifting, with or without prolonged periods of time standing, can result in a significant increase in pelvic floor pressure and a higher incidence of uterine prolapse. Since medical secretaries would not be required to do a lot of heavy lifting, they would not necessarily be at a higher risk of uterine prolapse. While having a significant smoking history can put people at higher risk for many different disorders, it does not put a patient at a higher risk for uterine prolapse.

Case A 14-year-old girl presents due to what appears to be a delay in the onset of puberty; she has not yet experienced menarche. Family history is not available. On examination, her height is 150 cm; her weight is 36 kg; and her body mass index adjusted for the age and gender is 16 (underweight). Her left breast has adult contour and areolae forms secondary mound, but the right one is smaller than the left. Her pubic hair seems to be of adult quality but does not spread to the junction of medial thigh with perineum. Question What is the most likely diagnosis?

Correct answer: Normal development Explanation Normal menarche happens from age 10 to 14 years. It is the presence of secondary sexual characteristics that will lead you to make a decision about eventual management. This patient probably has normal development because her secondary sexual characteristics (pubic hair and breast development) can be considered normal. Increases in height velocity and breast development are usually the first signs of puberty in girls. Breast asymmetry may occur in normal teenage girls, even when both breasts began developing at the same time, but one breast is growing more rapidly than the other, causing the asymmetry. This difference between breasts usually becomes significantly less noticeable during further development. Medical intervention can be deferred until 16 years of age. Primary amenorrhea is defined as an absence of secondary sexual characteristics by age 13 or the absence of menarche by age 15 in the presence of otherwise normal secondary sexual characteristics. Your patient does not satisfy any of those criteria. Your patient cannot have secondary amenorrhea because she has never had menses. Turner syndrome is one of the most common chromosomal abnormalities. Typically, a patient will present with characteristic dysmorfic features (short stature, swelling of the hands and feet, widely spaced nipples, low hairline, increased weight, characteristic facial features, or webbed neck) together with the cardiovascular, visual, urogenital, and/or cognitive disturbances. Gonadal dysfunction is also present, with adrenarche (the beginning of pubic hair growth) occurring at a normal age, but breast development is absent (because ovarian failure occurs before puberty) and there are also streak ovaries (result of stromal fibrosis) and uterus showing signs of hypoestrogenic state (e.g., atrophic endometrium). Your patient has weight loss, but anorexia nervosa would be associated with disturbed body image and typical behavior (like the avoidance of foods and induction of regurgitation after ingestion). Inadequate caloric intake can cause decreased secretion of gonadotropin-releasing hormone. GnRH and patients can develop either primary or secondary amenorrhea, depending on a degree of weight loss, but your patient has neither.

Question While performing a Doppler ultrasound on a woman at 30 weeks gestation, you notice that the fetal heart rate is consistently within the range of 130 - 140 beats/minute. What can you conclude about the heart rate of the fetus?

Correct answer: Normal heart rate Explanation The fetus has a normal fetal heart rate, as it is within the range of 110 - 160 beats/minute. The diagnosis of structural heart disease cannot be established by a fetal heart rate. The fetus has neither tachycardia (an early sign of fetal asphyxia), nor bradycardia (a late sign of asphyxia). Fetal bradycardia is defined as a decrease in the baseline fetal heart rate to <100 beats per minute. Suspicious tachycardia is defined as the heart rate being between 150 and 170; a pathological pattern is above 170 beats/minute.

Case A 23-year-old G1P0 woman presents for her first prenatal visit. The first day of her last menstrual period (LMP) was February 23. Generally, she feels well and has no complaints. The patient has started a prenatal vitamin, is getting the appropriate amount of sleep, and is keeping up with a moderate exercise routine. A complete physical exam is performed, and there are no abnormal findings. Initial ultrasound documents fetal cardiac activity. Question Using Nägele's rule, what is the patient's estimated date of confinement (EDC)?

Correct answer: November 30 Explanation Nägele's rule is a mathematical way of calculating a pregnant patient's EDC using the first date of her LMP: EDC = LMP - 3 months + 7 days. Therefore, using this equation, this patient's EDC is November 30.

Case A 26-year-old woman presents with 20 weeks of amenorrhea. This is the first time she has been to the doctor in 5 years and has come to you because she believes she is pregnant. You perform a complete physical examination and cannot auscultate the fetal heart rate. The ultrasound reports a "snowstorm" pattern with placental and fetal remnants missing. The patient tells you she has history of miscarriage "that had something to do with high blood pressure." The doctor explains that she is not pregnant and has a disease related to the proliferation of trophoblasts. Question What is a risk factor in the development of this disease?

Correct answer: Nutritional deficiency Explanation Nutritional deficiency is a risk factor in the development of hydatidiform moles. Specifically, deficiency of animal fats and carotene are to blame, especially vitamin A deficiency (carotene), folic acid deficiency, and low protein. Hydatidiform mole is a disease that increases the proliferation of trophoblasts. Partial hydatidiform mole is diagnosed when there is a fetus (karyotype 69,XXY or 69,XXX). Complete hydatidiform mole is diagnosed when there is no fetus (karyotype 46,XX or 46,XY). Complete hydatidiform moles are more common than partial. Other risk factors besides nutritional deficiencies are previous molar pregnancy and young (teenage) or advanced maternal age. Risk for mole development is 5-10 times greater if the patient is older than 40 years old. Alcohol consumption is not a known risk factor for molar pregnancy; however, it can increase the risk of fetal alcohol syndrome (FAS). This syndrome is characterized by anatomic deformities and intellectual disability. Eclampsia is not a risk factor developing a molar pregnancy, but once a molar pregnancy exists it can be a risk factor for development of malignancy. 20% of complete hydatidiform moles become malignant. Hyperthyroidism, like eclampsia, increases the risk in malignancy once a mole exists but is not a risk factor in the initial development of a hydatidiform mole. Multiparity does not increase the risk of developing hydatidiform mole.

Case A 16-year-old girl presents with concerns over her "growth". She feels ashamed and left out because she is the only girl in her 11th grade class that hasn't yet had her 1st period. Her mother attained menarche at 11 years of age, and her younger sister attained it at 10. She is sexually active with her boyfriend of 1 year; she has always used contraception. On examination, her vital signs are normal. Physical examination reveals rudimentary breast buds at Tanner stage 2 and an absence of pubic and axillary hair. A pelvic sonogram shows a normal appearing vagina and uterus. An MRI of the brain shows a normal appearing pituitary gland and hypothalamus, with agenesis of the olfactory bulb. Question How do you best describe the pathophysiology of this condition?

Correct answer: Reduced gonadotropin secretion from pituitary Explanation Reduced gonadotropin secretion from pituitary is the correct response. Kallman's syndrome is a form of idiopathic hypogonadotropic hypogonadism. Pituitary secretion of FSH and LH is reduced to varying degrees, resulting in the clinical manifestations. Patients present with hypogonadism; infertility and varying degrees of pubertal development. Multiple genetic mutations have been associated with Kallman syndrome; the most common gene associated is the PAX1 gene. Both boys/men and girls/women are affected; boys/men are more commonly affected. The hallmark of the disorder is anosmia or severe hyposmia. As described in the vignette, patients have a reduced or absent sense of smell, which is present from birth. MRI of the brain may reveal multiple different malformations of the olfactory bulbs or sulci, including agenesis or partial agenesis. The pituitary and hypothalamus are structurally normal. It was previously thought to be associated with color blindness, which has now been disproved. Lab values typically show reduced levels of LH and FSH with reduced sex hormones as a result. In men, testosterone levels are below normal; in women, estrogen and progesterone are below normal. Treatment is with gonadotropin replacement, estrogen-progesterone in girls/women and testosterone in boys/men. Fertility options include induction of ovulation with clomiphene; another option is artificial insemination by a variety of techniques. End organ resistance is incorrect. End organ resistance to insulin is seen in polycystic ovarian syndrome; so are hirsutism and acanthosis nigricans. The pathogenesis is multifactorial, with an increased LH/FSH ratio, increased androgens, hirsutism, and polycystic ovaries. Menarche is not usually delayed, and sonogram studies show a 'chain of lakes' picture of multiple ovarian cysts. Adrenal hypofunction is incorrect. Adrenal sex steroids are not primarily responsible for puberty. Chromosomal anomalies is an incorrect response. Chromosomal anomalies resulting in delayed puberty is most commonly seen with Turner's syndrome. The XO genotype results in primary amenorrhea, failure of secondary sex characteristics, and streak ovaries. Coarctation of the aorta and renal abnormalities may be seen. Olfactory problems are not seen. Atresia of the genital outflow tract is incorrect. Mullerian agenesis results in varying degrees of atresia of the outflow tract. It results from defects in development in fusion and development of the Mullerian ducts with the fallopian tubes, uterus, and the upper third of the vagina. The pelvic sonogram shows a normal uterus and tubes, ruling it out.

Case A 51-year-old Caucasian woman presents to the clinic to follow up on multiple tests she had requested from another practitioner. She has had what she describes as "episodes", in which she feels overheated and diaphoretic. Her coworkers note that her face turns red. These episodes seem to occur multiple times during the day; they have been occurring for the last 2 - 3 months, and they last about 1 - 2 minutes before they resolve. She is also very warm when sleeping at night. She denies weight changes, palpitations, headaches, galactorrhea, acne, bowel changes, hair loss, and any changes to her skin or nails. Overall, she reports some mild malaise and irritability, but she denies depression and fatigue. Her past medical history is unremarkable, with no known medical conditions, no allergies, no medications, and no prior surgeries. She denies the use of alcohol, tobacco, and drugs. Her method of contraception is vasectomy in her husband. Her last menstrual period was about 4 months ago, but periods were regular and monthly prior to that. Physical exam and vitals are normal. Test results are shown. Complete blood count (CBC) Normal Thyroid stimulating hormone (TSH) Normal Urinalysis Normal Complete Metabolic Panel (CMP) Normal Follicle stimulating hormone Elevated Quantitative hCG Negative Prolactin Normal Free/total testosterone Normal Question What intervention would be most appropriate for this patient's condition?

Correct answer: Offer prescription hormone therapy Explanation The proper intervention is hormone therapy. This patient is experiencing the climacteric stage of life, which is often referred to as menopause. Technically, menopause is defined as the point in time in which a woman has had no menstrual periods for 1 year, which is due to decreased ovarian function, and the average age at which menopause occurs is 51.5 years. This woman would be considered peri-menopausal. The most common presenting symptoms of this normal transition of the reproductive phase of life are hot flushes (flashes) and night sweats. The rise in the 'upstream' follicle-stimulating hormone (from the pituitary) in response to very low ovarian output is easily measured. In most cases, with classic symptoms and a patient of age in her 40's-50's, testing for menopause is not necessary. If the patient desires, or additional symptoms, history, and/or physical exam suggest another disorder, testing can be done. Estrogens are the most effective treatment for menopausal vasomotor symptoms, and this patient has no contraindications for estrogen use. The provider should offer prescription hormone therapy, along with education about alternatives, risks, benefits and duration of therapy. Oral contraceptives are not necessary. This patient does not need another form of contraception and has no need for ovarian suppression. In some cases, such as women with recurrent ovarian cysts, it is reasonable to initiate oral contraceptives for ovarian suppression. If this patient presented with symptoms and labs suggestive of a pituitary adenoma or prolactinoma, it would be wise to order an MRI of the head as a next step. A prolactinoma could cause amenorrhea (not likely to cause her hot flashes and night sweats), and would be associated with elevated prolactin levels and possibly headaches and galactorrhea. Pituitary adenomas may be associated with a variety of endocrine symptoms and several abnormal pituitary hormones (TSH, FSH, prolactin). Post-menopausal bleeding and/or evidence of a thickened endometrium (usually seen on ultrasound) are the most common indications for which one should perform an endometrial biopsy, which is a procedure to sample a small amount of endometrial tissue. The sample is evaluated by pathologists to exclude malignancy and hyperplasia. Vasomotor symptoms and amenorrhea are not indications for an endometrial biopsy. The primary care practitioner can easily treat menopause and its related conditions. There is no reason to refer this patient to an endocrinologist, given her symptoms and test results. It would simply delay treatment of her symptoms.

Case A 51-year-old Caucasian woman presents to the clinic to follow up on multiple tests she had requested from another practitioner. She has had what she describes as "episodes", in which she feels overheated and diaphoretic. Her coworkers note that her face turns red. These episodes seem to occur multiple times during the day; they have been occurring for the last 2 - 3 months, and they last about 1 - 2 minutes before they resolve. She is also very warm when sleeping at night. She denies weight changes, palpitations, headaches, galactorrhea, acne, bowel changes, hair loss, and any changes to her skin or nails. Overall, she reports some mild malaise and irritability, but she denies depression and fatigue. Her past medical history is unremarkable, with no known medical conditions, no allergies, no medications, and no prior surgeries. She denies the use of alcohol, tobacco, and drugs. Her method of contraception is vasectomy in her husband. Her last menstrual period was about 4 months ago, but periods were regular and monthly prior to that. Physical exam and vitals are normal. Test results are shown. Complete blood count (CBC) Normal Thyroid stimulating hormone (TSH) Normal Urinalysis Normal Complete Metabolic Panel (CMP) Normal Follicle stimulating hormone Elevated Quantitative hCG Negative Prolactin Normal Free/total testosterone Normal Question You plan to educate the patient about her condition and offer treatment for her current symptoms. In addition, this patient should be counseled regarding what risk, for which she is now at increased risk due to her current diagnosis?

Correct answer: Osteoporosis Explanation This patient is experiencing the climacteric stage of life, often referred to as menopause. Technically, menopause is defined as the point in time in which a woman has had no menstrual periods for 1 year, due to decreased ovarian function and the average age for this is 51.5 years. This woman would be considered peri-menopausal. The most common presenting symptoms of this normal transition of the reproductive phase of life are hot flushes (flashes) and night sweats. The rise in the 'upstream' follicle-stimulating hormone (from the pituitary) in response to very low ovarian output is easily measured. In most cases, with classic symptoms and a patient of age in her 40's - 50's, testing for menopause is not necessary. If the patient desires, or additional symptoms, history and/or physical exam suggest another disorder, testing can be done. With the drop in natural estrogen production, menopausal woman can experience rapid bone loss and are at risk for osteoporosis. Many approaches can be used to address the risk, including prescription hormone therapy and/or bisphosphonates (if appropriate), weight-bearing activity, adequate calcium and vitamin D, and avoidance of tobacco products. Much has been debated about the role of menopause and prescription hormone therapy in the development of Alzheimer's disease. Currently, no definitive conclusions can be drawn that menopause is a definite risk factor for Alzheimer's. Endometrial hyperplasia is a condition in which the lining of the uterus (endometrium) over-proliferates. It can lead to malignancy. Endometrial hyperplasia can occur with polycystic ovarian syndrome, obesity and inappropriate unopposed administration of estrogens. It is not associated with normal menopause. In fact, with the decline in natural estrogen production, the endometrium atrophies and becomes very thin. Endometriosis is another condition in which estrogen effects can worsen the disease process, and a drop in estrogen (as with menopause) typically improves symptoms. Risk of endometriosis drops with menopause. This patient does not have a malignant condition associated with metastasis. If the patient is placed on hormone therapy, she should be counseled about the risks of breast cancer. However, her climacteric state does not incur a specific risk for metastasis.

Question Serosanguineous ascites is generally associated with which of the following processes?

Correct answer: Ovarian cancer Explanation The correct answer is ovarian cancer. Bloody or serosanguineous ascites is associated with intraperitoneal cancers, of which ovarian cancer is one. Lymphoma is incorrect because it will generally produce a chylous ascites. Congestive heart failure is incorrect because it will generally produce serous ascites. Cirrhosis is incorrect because it will generally produce serous ascites. Infection is incorrect because it will generally produce cloudy ascites.

Case A 29-year-old woman, G4P2011, LMP 9 months prior, presents with sudden onset of severe lower abdominal pain. The pain is sharp and tearing, and was not preceded by the contractions she recalls from previous deliveries. She also endorses vaginal spotting prior to presentation. There is no history of prior medical problems or surgery, and she is on no medications. All previous deliveries were vaginal. She has smoked 1 pack of cigarettes a day over the past 10 years. She denies alcohol use but does admit to a remote history of heroin abuse by insufflation. On physical examination: T = 99.4° F; BP= 110/70 mmHg; P= 85/min; RR= 20/min. Pertinent findings on the PE were relegated to the pelvic exam; fundal height measures 39 cm, and there is profuse bleeding from the vagina. Fetal monitor shows contractions every minute with elevated baseline uterine tone. Fetal tachycardia is evident at 180 beats/minute, and late decelerations are also present. Question What is the most likely diagnosis?

Correct answer: Placental abruption Explanation This patient has placental abruption. Abruption is a leading cause of 2nd and 3rd trimester bleeding. A meta-analysis has demonstrated that smoking increases the risk of abruption by 90%. Maternal and paternal smoking increases the risk of abruption 2-fold. Risk increases 5-fold when both parents smoke. Women with a history of abruption have a 15% increased risk during future pregnancies. Placenta previa, cocaine use, preeclampsia, and preterm premature rupture of membranes (PPROM) are also associated with placental abruption. Placental abruption is defined as a complete or partial separation of the placenta prior to delivery. The incidence of abruption is 5 or 6 out of 1000 deliveries. Obvious vaginal bleeding occurs if the hemorrhage develops between the membranes and the uterus. Concealed presentations occur when blood collects behind the placenta. Abruption is a leading cause of 2nd and 3rd trimester bleeding, and it causes significant maternal and neonatal morbidity and mortality. Classically placental abruption presents as painful third-trimester bleeding. Treatment: Emergent cesarean is generally indicated when the patient is not in labor and hemodynamic compromise is present. If delivery is imminent and the abruption is mild, vaginal delivery may be attempted. Cervical cancer is incorrect. Although cervical cancer can coexist with pregnancy, it is very unlikely in this case. The severity of the signs and symptoms, in this case, would be incongruent with achieving 9 months gestation. Chorioamnionitis is incorrect. Clinical features include maternal fever and uterine tenderness in the presence of confirmed premature rupture of membrane (PROM). Profuse vaginal painful bleeding is not a finding. Placenta previa is incorrect, as it classically presents with painless 3rd trimester bleeding. Uterine rupture is theoretically possible but is not the most likely diagnosis. A uterine rupture typically occurs during active labor, most commonly in women who have had a previous Cesarean section. As this patient is hemodynamically stable, had a sudden onset of pain (not preceded by contractions / early signs of labor), and has not had any previous c-section deliveries, this diagnosis is very unlikely.

Case A 24-year-old obese woman presents because she cannot become pregnant. She and her husband have been trying to have a child since they were married 4 years ago. She claims to not have had her period for quite some time. She denies any pelvic pain or discomfort. On physical examination, you notice that she is afebrile and has quite a bit of body hair; her pelvic examination is unremarkable. Question What is the most likely diagnosis?

Correct answer: Polycystic ovary disease Explanation This patient has signs and symptoms of Polycystic ovary disease, which is also called Stein-Leventhal syndrome. This syndrome is characterized by sterility, obesity, hirsutism, and amenorrhea. However, irregular menses can sometimes be seen (instead of amenorrhea). In addition, the ovaries will have numerous cysts. Pelvic inflammatory disease (PID) can also lead to sterility. However, there would not be an association of PID with obesity, hirsutism, or amenorrhea. Dysmenorrhea can be seen with PID. In addition, a patient with PIDwould usually present with lower quadrant pain. On physical examination, there should be positive findings, which was not the case here. Endometriosis can present with dysmenorrhea and irregular menstrual cycles. However, there is not an association of endometriosis with obesity or hirsutism. Leiomyomas are usually asymptomatic. Obesity and hirsutism are not associated with leiomyomas. Menorrhagia, rather than amenorrhea, is a more frequent presentation with leiomyomas. The majority of women with endometrial cancer are postmenopausal. There is an association of polycystic ovarian syndrome with endometrial carcinoma. However, obesity and hirsutism, in and of themselves, are not associated with endometrial carcinoma.

Case A 23-year-old recently married woman presents for a periodic health examination. She has no complaints; she practices aerobic exercise at least 4 times a week, drinks a glass of wine or beer "sometimes," does not smoke cigarettes, and does not use illicit drugs. Both she and her husband want to have a baby as soon as possible. Her menstrual periods are regular, in 28-day intervals, usually lasting 4 - 5 days. Her last period was 7 days ago. Physical and pelvic examinations are unremarkable. Her Pap smear is normal. Question What should your next step be?

Correct answer: Prescribe daily folic acid Explanation All women planning to become pregnant should take a daily multivitamin supplement containing folic acid; they should begin taking it at least 1 month prior to conception and continue through the 1st trimester to reduce the risk of neural tube defects. A urine pregnancy test is unnecessary. Having had a period 7 days ago, she is very unlikely to be pregnant at this time. A vaginal wet mount is used to find the cause of vaginitis. Your patient has no symptoms of inflammation. The U.S. Preventive Services Task Force recommends against routine serological screening for herpes simplex virus in asymptomatic adolescents and adults. Pregnant women should be tested for asymptomatic bacteriuria at their first prenatal visits to prevent urogenital tract infections and related complications in pregnancy. Since she is most likely not pregnant, this test is unnecessary at this time.

Question A 16-year-old girl has had recent onset of painful menstrual periods. Age of menarche was at 12 years. Her periods were irregular for about 8-10 months, but have been fairly regular since then, with occasional mild crampy pains on the first 2 days of her period. She rates the pains now as being 8-9 on a scale of 10. Cramps will start about a half a day before the onset of her periods, worsen the following day, and then gradually subside over the next day. She also describes having a headache, looser stools, mild nausea, and low back and thigh pain. Bleeding is moderate on the first 2 days. She is otherwise healthy without history of abdominal surgery. Privately she denies any sexual activity. She is an average student and has missed 1-2 days of school with each period for the past 3 months. She also denies any fever or dysuria. There is also no family history of gynecological problems. On exam she has diffuse midline lower abdominal tenderness with some mild soreness to lower back and thighs, bowel sounds are normal, and there is no rebound tenderness. An external genital exam is normal. Based on this information, which of the following is the most likely cause of this girl's dysmenorrhea?

Correct answer: Primary dysmenorrhea Explanation Primary dysmenorrhea is associated with ovulatory cycles and typically occurs at an earlier age than secondary dysmenorrhea, as 1/3 of adolescents continue to experience anovulatory cycles in the first few years after menarche due to the time required for the hypothalamic-pituitary-ovarian cycle to mature. Disability is common with a reported 14% of girls reporting missing school or work because of pain and nearly 50% of those have pain described as moderate or severe. The physiologic basis relates to cell membrane phospholipids, endomyometrial prostaglandins, and leukotrienes. Defining symptoms include crampy midline lower abdominal pain beginning with menstrual flow or a short time before. As in this patient, cramps are typically intense on the first or second day and resolve before the end of the menstrual flow. The pain may be referred to the lower back and anterior thigh. Nausea or vomiting may occur as well as near-syncope or dizziness and complaints of weakness. Breast tenderness, bloating, headache, and mood changes can also be seen. Management involves the use of NSAIDs, which reduce the production of prostaglandins. Oral contraceptives should be considered for teens that have not experienced sufficient relief of symptoms with NSAIDs. Endometriosis occurs when functioning ectopic endometrial glands and stroma are present outside the uterine cavity, causing pain, irregular bleeding, and frequently infertility, though it may also be asymptomatic. The pelvic pain often cycles with menstruation and is associated with dysmenorrhea, dyspareunia, and infertility. The pain can be a deep constant ache with bilateral patterns of distribution and radiate to the buttock and perianal region. Pain may be associated with bladder or bowel function and there can be menstrual spotting. There is usually nonspecific pelvic tenderness on exam. Unlike this patient, it occurs more frequently in first-degree relatives of women whose endometriosis has been confirmed surgically. The pain of irritable bowel syndrome in adolescents is similar to that described in adults, with paroxysmal periumbilical abdominal pain that, unlike in this patient, is relieved by defecation or associated with an irregular pattern of defecation, including a change in frequency or consistency of stool, straining or urgency, feeling of incomplete evacuation, passage of mucus, and bloating or abdominal distention. Left lower quadrant fullness over the left colon may also be present. Periods of diarrhea alternate with periods of constipation. Imperforate hymen is recognized as a membrane covering the vaginal opening seen on external exam not evident in this patient. This may be bulging or a have a bluish hue from blood retained. There is a history of cyclic abdominal pain and a midline abdominal mass. A large enough mass may cause urinary obstruction. Ovarian cysts are fluid or semisolid filled sacs that develop on or within the ovary and are typically functional and disappear on their own. These develop from follicles that fail to rupture and release an egg and instead of being reabsorbed, the fluid within the follicle persists and forms a cyst. This may cause a sudden onset of pain that is constant or intermittent, dull or sharp aching pelvic pain that occurs, unlike the pattern seen in this patient, shortly after beginning or ending a menstrual period. There may be abnormal uterine bleeding, either longer, shorter or absent, abdominal bloating or distention, or no symptoms. Large or persistent cysts may require surgical removal. References:

Case A 16-year-old girl presents due to back pain. She has a 5-month history of experiencing pain in the lower midline of her back, which occurs around the time of her period. Additionally, cramping pelvic pain often radiates to her back and inner thighs. Cramps can last for more than 1 day and are sometimes present with headaches. Upon physical exam, her back is non-tender, as she is not currently on her period. Furthermore, the patient does not have any underlying diseases or conditions. All laboratory findings are in normal range. Question What is the most likely diagnosis?

Correct answer: Primary dysmenorrhea Explanation The clinical picture is suggestive of primary dysmenorrhea. The patient has a history of menstrual pain associated with her period in the absence of abnormal lab findings. Abnormal premenopausal bleeding is heavy bleeding that can be associated with clots. Premenstrual syndrome consists of physical and emotional symptoms that can alternate before a woman's period. Secondary dysmenorrhea is pain that comes from the woman's reproductive organs. Additionally, the pain usually begins earlier on in the menstrual cycle. Vaginitis is an inflammation of the vagina that results in discharge and itching.

Case A 20-year-old woman is brought to the emergency department by the police. She was assaulted and raped 3 hours ago. Her last menstrual period (LMP) began 22 days ago, and her period lasts for 4 to 5 days with a 28-day menstrual cycle. She is not using any contraceptive method. Question What hormone is mainly produced by the ovary at this point of her menstrual cycle?

Correct answer: Progesterone Explanation The correct response is progesterone. Day 22 of the menstrual period corresponds to the luteal phase of the menstrual cycle. The luteal phase starts from the day of ovulation to the first day of the menstrual period. In this phase, the ovarian corpus luteum begins to produce progesterone. If there is fecundation fertilization of the ovum, the production of progesterone continues for 4 to 5 weeks until the placenta can take over and produce enough of the hormone to maintain pregnancy. If there is no fertilization, progesterone levels decrease and the menstruation occurs. Progesterone helps develop the lining of the uterus in preparation for implantation. It also makes cervical discharge thick and more cellular. Estrogens are produced by the granulosa cells of the follicle. They are very important in the follicular phase; their level is very low at the beginning of this phase and reaches a maximum point at the middle of the cycle, just before the LH peak. They are the feedback for LH production and (in part) for FSH production. Day 5 day of the menstrual period corresponds to the follicular phase of the cycle, which starts from the first day of bleeding to the day of the LH peak. In this phase, FSH, which is produced by the pituitary, stimulates the development of follicles in the ovaries, with only 1 follicle dominant at the end. During the second half of this phase, the granulosa cells of the follicle begin to produce estrogen. Ovulation occurs approximately at the middle of the menstrual cycle (day 14). At the end of the follicular phase, a rise in the level of estrogens takes place, preceding the LH peak. Ovulation will succeed by an average of 30 hours after this peak. Inhibins are hormones produced by granulosa cells of the follicle, and they are part of the feedback for FSH. Inhibin B levels rise during the luteal-follicular transition, are highest during the mid-follicular phase, then go up again during the LH peak, and finally decrease in the late follicular phase. Inhibin A levels decrease during the late luteal phase. LH is a pituitary hormone. Its levels are fairly steady during the follicular phase, then peak sharply just before ovulation. The LH peak is the stimulus for ovulation. After ovulation, LH levels decrease during the luteal phase.

Case A 16-year-old girl presents with concerns over her growth. She feels ashamed and left out because she is the only girl in her 11th grade class that has not yet had her first period. Her mother attained menarche at 11 years of age, and her younger sister did so at 10. She is sexually active with her boyfriend of 1 year and has always used contraception. On examination, her vital signs are normal. Physical examination reveals rudimentary breast buds at Tanner stage 2 and absence of pubic and axillary hair. A pelvic sonogram shows a normal appearing vagina and uterus. An MRI of the brain shows a normal pituitiary gland and hypothalamus, with agenesis of the olfactory bulb. Question What pathophysiology is the most likely cause of this patient's condition?

Correct answer: Reduced gonadotropin secretion from pituitary Explanation Kallman's syndrome is a form of idiopathic hypgonadotrophic hypogonadism. Pituitary secretion of FSH and LH is reduced to varying degrees, resulting in the clinical manifestations. Patients present with hypogonadism, infertility, and varying degrees of pubertal development. Multiple genetic mutations have been associated with Kallman's syndrome, the most common gene associated being the PAX1 gene. Both males and females are affected, with males being more commonly affected. The hallmark of the disorder is anosmia or severe hyposmia. As described in the vignette, patients have a reduced or absent sense of smell, which is present from birth. MRI of the brain may reveal multiple different malformations of the olfactory bulbs or sulci, including agenises or partial agenesis. The pituitary and hypothalamus are structurally normal. It was previously thought to be associated with color blindness, which has now been disproved. Lab values typically show reduced levels of LH and FSH, with reduced sex hormones as a result. In men, testosterone levels are below normal; whereas in women, estrogen and progesterone are below normal. Treatment is with gonadotropin replacement: estrogen-progesterone in women and testosterone in males. Fertility options include induction of ovulation with clomiphene, as well as artificial insemination by a variety of techniques.

Case A 28-year-old G2P2 woman has just delivered a term male infant via normal spontaneous vaginal delivery. There was spontaneous rupture of the membranes 2 hours prior to delivery. Meconium stained amniotic fluid and thick particulate meconium was noted. The infant is brought under the radiant warmer where he appears peripherally cyanotic. He is crying and moving vigorously, and his heart rate is 160 beats per minute. Question What is the most appropriate next step in the resuscitation of this infant?

Correct answer: Routine neonatal evaluation since infant is vigorous Explanation The recommendations have changed with the publication of the 2005 AHA/AAP Neonatal Resuscitation Program guidelines. Although the presence of meconium in the amniotic fluid should alert the caregiver to the possibility of neonatal distress, a vigorous infant is a good sign and further intervention is not warranted. Routine suctioning of the nose and mouth as the head is delivered is no longer recommended because it does not reduce the risk of meconium aspiration syndrome. In the presence of meconium stained fluid, suctioning is recommended if the infant has a low heart rate (<100 bpm), depressed respirations, or poor muscle tone. In an infant with one or more of these signs of distress, suctioning should be performed. Prior to drying, the infant should have visible, residual meconium removed from the nose and mouth by bulb suction. Then, under direct visualization, the trachea should be suctioned with an appropriately sized endotracheal tube. The infant in this case scenario is clearly vigorous and therefore does not require suctioning. He should only receive routine care (standard positioning, warming, etc.).

Case An 18-year-old woman with a history of irregular menses since menarche presents for the evaluation of an 8-month history of absent menses. She has been a ballet dancer since childhood. On examination, her height is 150 cm; her weight is 36 kg; and her body mass index adjusted for the age and gender is 16 (underweight), with female pattern of body hair and normal secondary sexual characteristics. Pregnancy test is negative. Question What is the most likely diagnosis?

Correct answer: Secondary amenorrhea Explanation Secondary amenorrhea is the cessation of menses for at least 6 months in already cycling women. Having low body mass index and a history of physical training, your patient most probably has secondary amenorrhea due to hypothalamic dysfunction. Inadequate caloric intake can also cause decreased secretion of gonadotropin-releasing hormone. Girls who regularly participate in activities such as strenuous gymnastics and ballet dancing may have delayed thelarche, delayed menarche, and irregular or absent menstrual periods. Primary amenorrhea is defined as an absence of secondary sexual characteristics by age 14 or the absence of menarche by age 16 in the presence of otherwise normal secondary sexual characteristics. Your patient does not satisfy any of those criteria. Polycystic ovarian syndrome will present with signs of hyperandrogenism, including hyperseborrhea with acne, scalp hair loss, or excessive facial and body hair. Androgen insensitivity is a syndrome when a genetically male is resistant to androgens. As a result, the person has some or all of the physical traits of a woman, but his genetic makeup is of a man. Those patients will never menstruate and will have absent secondary terminal hair. A hydatiform mole is a type of gestational trophoblastic disease. This abnormal pregnancy is the result of a non-viable fertilized egg that is implanted in the uterus and does not come to term. Most commonly a hydatiform mole presents clinically with vaginal bleeding and a positive pregnancy test. Quantitative beta-hCG levels often show highly active trophoblastic growth; levels greater than 100,000 mIU/mL are suspicious for a molar pregnancy.

Case A 35-year-old woman presents with symptoms of depression; the depression starts a few days prior to onset of menses. She says that she feels fine once her menses ends, but she dreads her mid-cycle. At that time, she starts to get moody, irritable, and as the days approach the end of her cycle, increasingly depressed. She says that occasionally she has felt suicidal when her premenstrual symptoms are at their worst. She also has some symptoms related to breast and menstrual cramps, but she feels she can handle these with the use of nonsteroidal medication. She feels her premenstrual dysphoria has always been present, but manageable, but in the past 5 months, it has been severe and is affecting her functioning both at home and at work. The patient's past medical history is negative except for 2 pregnancies and deliveries. She has no history of a depressive disorder. She denies any major changes in her life over the past year. She does not use any medications or birth control because her husband had a vasectomy. She has a complete and focused exam where no abnormalities were found. The patient is about 30 lb overweight, but she seems to be in good health otherwise. Question At this point, what should be suggested to treat the patient?

Correct answer: Serotonin reuptake inhibitors Explanation The correct response is serotonin reuptake inhibitors. The diagnosis of premenstrual syndrome (PMS) is common in many women, with the experience of mild symptoms in the days preceding menstrual flow. In most cases, however, these symptoms are not severe, and they are managed with over-the-counter medication. In a small percentage of women, the experience is much more unpleasant and serious. They suffer from premenstrual dysphoric disorder (PMDD), a more severe form of PMS. To be labeled PMDD, the presence of significant emotional distress with other symptoms that interfere with social and occupational functioning must be present; symptoms can include insomnia or hypersomnia, lethargy, difficulty concentrating, etc. Finally, these symptoms have to be absent in the week post menses. Many women have depression or anxiety symptoms throughout the menstrual cycle with worsening during the luteal phase. In such a case, a diagnosis of the underlying disorder (anxiety, disorder, etc.) is more appropriate than PMDD. PMDD has a prevalence of 3-8% in women in their reproductive years. There seems to be some alteration in the function of neurotransmitters (such as epinephrine, norepinephrine, serotonin, and gamma-aminobutyric acid). For example, women with PMDD have diminished whole blood serotonin and platelet serotonin uptake. Moreover, women with PMDD have a higher incidence of prior rates of postpartum depression and mood disorders. Selective serotonin reuptake inhibitors (SSRIs) are first-line treatment for PMDD. Most women respond to a fluoxetine dose of 20 mg/day, and some even respond to lower doses. Other SSRIs have also demonstrated efficacy. Some reports have suggested that fluoxetine, which is a longer acting SSRI, may be effective when given only during the luteal phase of the menstrual cycle. Danazol is a synthetic testosterone that suppresses ovulation. Danazol has a number of adverse effects (including masculinization and hepatotoxicity) and is not commonly used in treating PMDD. Spironolactone has been used to help patients with fluid retention and weight gain. A review of studies evaluating vitamin B6 in premenstrual syndrome with depression was performed. This review showed a definite improvement in depressive symptoms in women who used vitamin B6 in doses up to 100 mg/day, but conclusions are limited by the low quality of most of the trials included in the review. Studies of oral contraceptives show them to have no effect—or occasionally worsen—symptoms of PMDD. Recent double-blind studies have demonstrated the effectiveness of calcium supplements in relieving premenstrual symptoms including depression; further studies are needed to evaluate the effectiveness of magnesium therapy in PMS and PMDD.

Case The patient is a 19-year-old woman who presents due to lower abdominal pain, chills, and vaginal discharge. She admits to being sexually active and has a history of irregular menstruation. Her pelvic examination reveals adnexal and cervical motion tenderness, as well as purulent discharge seen at cervical os. Question What other factor in the patient's case would influence you to admit her for inpatient intravenous antibiotic therapy instead of outpatient antibiotic therapy?

Correct answer: Serum β-hCG level of 28,000 mIU/mL Explanation Given the patient's history and physical examination, the most likely diagnosis is pelvic inflammatory disease (PID). Most cases of PID are treated on an outpatient basis, but there are certain criteria that warrant inpatient intravenous antibiotics. One of the indications for hospitalization is pregnancy, and a serum β-hCG of 28,000 mIU/mL is laboratory evidence that this patient is at least 5 weeks pregnant. In addition, the chance for ectopic pregnancy as a cause of the symptoms and elevated serum β-hCG still exists. Ectopic pregnancy is treated surgically, so this would be an added reason to treat the patient as an inpatient. Oral temperature of 100.8°F is not abnormal for a patient with PID. In fact, one of the additional criteria used to diagnose PID is an oral temperature over 101°F. If a patient has a high fever associated with the disease, then the health care provider may opt to treat as an inpatient. High fever may be a sign that there is a pelvic abscess or another undiagnosed surgical emergency, such as appendicitis. Lower abdominal tenderness is a symptom of PID; it is not a criterion for admission. N. gonorrhoeae can be found in the cervical culture of a patient with PID, and a C-reactive protein can be elevated. Neither is necessary for diagnosis, but both are common in patients with PID. They are not reasons for inpatient treatment in themselves.

Question A sexually active, 17-year-old patient presents alone to your office and requests contraceptive counseling. Because she is under the age of consent, what is an ethical concern of which you must be aware in seeing her?

Correct answer: She can give consent for contraceptive counseling and evaluation of STDs. Explanation This case highlights several ethical issues. Autonomy indicates that an individual may make his/her own health care decisions based on their own values. It is usually limited by a determination of whether the individual is competent and, in pediatrics, at least in part determined by the age of the individual. Competence is an individual's ability to understand the possible consequences of his/her decision and the available alternatives. Patients are generally considered to have a right to confidentiality, i.e., trust that information about them will not be disclosed. Depending on the age of the patient, there may be a conflict of interest between the physician's duty to the patient and the physician's duty to the parents. States either have laws allowing individuals who would otherwise be considered minors to obtain contraceptive services or have no law precluding the provision of such services.However, most states have placed the interests of the adolescents above those of the parents in issues relating to the management of sexually transmitted diseases and other issues of sexuality. Physicians have a duty to tell the truth to their patients. This duty can create quite a conflict when other family members, especially parents, request information about another family member that the physician is obligated to keep confidential.

Case You are evaluating a 22-year-old primiparous woman at 10 weeks gestation. Though she currently works as a design graphics consultant, she was a competitive swimmer in college and still maintains a regimen of endurance and resistance training 3 to 4 times per week. She has had no vaginal spotting, and the pregnancy is progressing normally. She would like to continue with her exercise routine but is concerned about reports from friends that exercise has an adverse effect on pregnancy. Past medical history is otherwise unremarkable. Vital signs are normal, and examination is consistent with a 10-week gestation. Question What advice would you give?

Correct answer: She may continue her current exercise regimen. Explanation Exercise during pregnancy provides benefits to both the mother and child. It is recommended that a pregnant woman engages in 2 ½ hours of moderate-intensity exercise per week during her pregnancy. The pregnant woman may safely engage in vigorous exercise as well. Maternal benefits include increased pain tolerance, lower total weight gain, and a decreased risk of preeclampsia and gestational diabetes. Fetal benefits include decreased incidences of preterm birth, Caesarean section, and large-for-gestational-age births. These recommendations apply to a normal pregnancy. Conversely, a sedentary lifestyle during pregnancy increases the risk of gestational diabetes, preeclampsia, peripartum complications, and post-pregnancy weight retention. Some activities should be avoided, such as those with a risk of abdominal trauma, scuba diving, or those in an overheated environment. As indicated above, a sedentary lifestyle during pregnancy is associated with an increased risk of adverse pregnancy outcomes. Vigorous activity during pregnancy is not associated with an increased risk of miscarriage. Moderate intensity and vigorous exercise actually decrease the rate of prematurity. Usual activities of daily living do not provide sufficient activity to achieve the benefits of 2 ½ hours of moderate intensity exercise per week during pregnancy.

Question A 21-year-old college student calls late one evening in a panic because she has just realized that she forgot to take her birth control pill the previous evening. What would be the most appropriate advice to give her?

Correct answer: She should take 2 pills now and then continue the rest of the package according to her usual nightly regimen Explanation Questions associated with missed pills and consequent contraceptive effectiveness are common for the practicing clinician. Even if appropriate instructions are provided to patients at the time of the initial prescribing, patients often forget them and call in a panic seeking reassurance. The basic guidelines regarding missed pills are as follows: After 1 missed pill, the patient should take the missed pill as soon as she remembers and take the pill for that day at the usual time. Should she not discover that she skipped a pill until the time that she usually takes her daily pill (usually before bedtime), she should take 2 pills at once. There is no need for "back-up" contraception after 1 missed pill. After 2 missed pills, the patient should take the 2 pills per day for 2 days, and should then return to her once daily regimen for the remainder of the package. A back-up method is generally recommended in this scenario. She should also be counseled regarding a slight increase in break-through bleeding. After more than 2 missed pills, the patient should again take 2 pills per day until she has caught up. A backup method is mandatory. A pregnancy test would be indicated should she skip her regular menses, although she may merely spot irregularly for the remainder of the cycle and not have her usual period.

Case A 32-year-old pregnant woman presents for a screening ultrasound at 16 weeks gestation. The ultrasound notes that the fetus is male, has shortened long bones (all below the 5%), and a large head circumference. Question You refer the patient to a tertiary care center for further evaluation and explain that you suspect that the fetus may have what condition?

Correct answer: Skeletal dysplasia Explanation In this case, there are signs of a skeletal dysplasia where bony growth of the long bones of the skeleton is not normal. Increased head circumference size can also be observed in the skeletal dysplasias. While achondroplasia is the most common recognized skeletal dysplasia, over 100 disorders have been reported and additional ultrasound imaging and possibly genetic testing will be needed to make a specific diagnosis. Trisomy 13 is often fatal in utero or in the first few days or weeks of life. Global growth retardation (not just long bone shortening), renal anomalies, cleft lip/palate, and severe central nervous system malformations may be noted on ultrasound. Trisomy 18 is often fatal in utero or in the first few days or weeks of life. Growth retardation may be present, along with renal anomalies, severe cardiac malformations, and omphaloceles. Skeletally, there may be a shortening of the sternum and prominent calcanei on the feet. Trisomy 21 sometimes leads to shortening of the long bones in utero, although head circumference is often decreased. Ultrasound markers would include the presence of congenital heart disease (1/3 of all live born cases), duodenal atresia, and increased nuchal translucency. 45 X, or Turner syndrome, has few major structural problems in fetal development in spite of the fact that many 45 X fetuses will spontaneously miscarry. Ultrasound imaging may identify a short neck, increased nuchal translucency (or even a cystic hygroma), and/or webbing of the neck. Additionally, the phenotype of the fetus in this problem is male, and the phenotype of 45 X is female.

Case A 22-year-old nulligravida presents to your office with a 3-day history of green-yellow, frothy, malodorous discharge; her last menstrual period was 2 weeks ago. She has also experienced mild pelvic pain during the same period of time. She is sexually active with 2 male partners, and she uses condoms occasionally. On wet mount, motile protozoans and white blood cells are noted. Question What may be seen with this type of vaginitis?

Correct answer: Strawberry cervix Explanation The patient has trichomoniasis, which is caused by the protozoan Trichomonas vaginalis. Vaginal discharge is often profuse and may be accompanied by mild pelvic pain and/or dyspareunia. In 10% of cases, a strawberry cervix may be seen. Pseudohyphae may be seen in wet mount examinations of vulvovaginal candidiasis. Ferning may be seen in microscopic examination of vaginal secretions in a pregnant patient with ruptured membranes. A positive whiff test is noted when vaginal secretions from a patient with bacterial vaginosis are mixed with potassium hydroxide (KOH), liberating amines, which have a characteristic fishy odor. The whiff test is 1 of the 4 diagnostic criteria for the diagnosis of bacterial vaginosis.

Case A 62-year-old obese woman presents due to urine leakage. She has had some symptoms for about 2 years, but they are getting worse. She leaks urine when she coughs or sneezes. She is not very active, but if she jumps, she also leaks urine. She wears a pad daily, as she has leakage daily. The amount varies from a few drops to a gush. The patient denies hematuria, dysuria, and pelvic pain. She sometimes feels vaginal pressure and fullness. She is considering quitting her job because she is embarrassed to be in public when she has urine leakage. Her past medical history reveals she is menopausal; she has had 4 vaginal deliveries. She has no other known medical conditions; she has not had any surgeries; she takes no medications and has no allergies. She is married and works part-time at a call center; she denies the use of tobacco, alcohol, and drugs. On physical exam, she is obese, with an atrophic vulva/vagina. The pelvic examination reveals downward and forward rotation of the vaginal wall, with an anterior bulging when the patient is asked to strain. The remainder of her exam is normal. A dipstick urinalysis is normal. Question What is the most appropriate intervention for this patient's current condition?

Correct answer: Surgical repair Explanation This patient is presenting with a cystocele, a herniation of the bladder wall into the vagina. Common symptoms may include stress urinary incontinence (SUI) and a feeling of vaginal fullness; other symptoms include incomplete voiding and dyspareunia. Of the choices listed, surgical repair is the most appropriate intervention. Other possible approaches to cystocele treatment include pelvic floor physical therapy and use of a pessary. The type of surgical approach to cystocele varies, from a less-invasive mesh sling to a more invasive colpopexy. Antibiotic therapy would be appropriate if this patient's urinary symptoms were due to a cystitis, or urinary tract infection (UTI). Common signs and symptoms of a UTI include urinary frequency and/or hesitancy, dysuria, and (possibly) gross hematuria; urinalysis shows white blood cells, red blood, cells and nitrites. Some women have both a cystocele and a UTI, so it is important to rule out infection when evaluating urinary tract problems. Daily cranberry juice and/or supplements are often used for urinary tract health. However, limited evidence supports cranberry's benefits related to UTI prevention. There is no evidence that cranberry juice or supplements is helpful for cystoceles and stress urinary incontinence. If this patient were suffering overactive bladder (detrusor instability) with resulting urge incontinence, it would be helpful to start prescription oxybutynin, an anticholinergic medication. However, this patient's problem is primarily anatomic, and most medications are ineffective in treating cystocele. If this patient was experiencing both stress and urge incontinence (mixed incontinence), the medication (along with surgery) would be indicated. Patients with frequent UTI may be instructed to use over-the-counter phenazopyridine (Azo) as needed. The phenazopyridine helps with bladder pain and has mild bacteriostatic properties. However, it does not have a role in cystocele and SUI. This medication also turns the urine bright orange, and it can interfere with urinalysis dipstick interpretation. References

Case A 28-year-old primipara presents with wide fever swings 6 days after the delivery of a healthy male infant. Her pregnancy was uneventful, but because of the prolonged membrane rupture and prolonged labor, an emergency Cesarean section was performed. On the 2nd postpartum day, she started having fever and complaining of uterine tenderness. She was treated with IV gentamycin and clindamycin. Today, her physical examination is normal; there is no uterine tenderness, and her WBC count is 11500, but she continues to spike fevers up to 39 C. Question What is the most likely diagnosis at this point?

Correct answer: Thrombophlebitis Explanation Your patient most probably has septic thrombophlebitis. It appears in day 5 - 6 in patients at risk (like those who had an emergency cesarean section after prolonged membrane rupture and prolonged labor). The patient has prolonged fever swings despite broad-spectrum antibiotics, and she has a normal physical and pelvic examination. Endometritis usually appears 2 - 3 days postpartum in patients with similar risk factors as thrombophlebitis, but patients present with clinical signs of exquisite uterine tenderness. Therapy that covers polymicrobial flora should result in the improvement of all signs and symptoms, including fever. Your patient developed symptoms 6 days after the surgery, and all signs disappeared except a fever. She initially had endometritis, a risk factor for developing septic thrombophlebitis. Septic shock will have a dramatic clinical picture of acute circulatory failure that is unexplained by other causes. Symptoms and signs of septic shock are not specific (fever, chills, rigors, altered mental status, dyspnea, fatigue, malaise, nausea, vomiting, hypotension, and/or signs of tissue hypoperfusion); however, your patient does not have symptoms or signs of circulatory failure. Appendicitis usually presents with RLQ abdominal pain, loss of appetite, nausea, vomiting, constipation or diarrhea, an inability to pass gas, low-grade fever, and abdominal swelling. Your patient does not have abdominal tenderness or high fever. Drug fever is a febrile response that coincides temporally with the administration of a drug, most often after 7 - 10 days after starting the drug, and it disappears after its discontinuation. To make a diagnosis of the drug fever, you have to exclude other causes. Therefore, you should obtain the image of pelvic structures before making a diagnosis of drug fever.

Case A 39-year-old woman who is 32 weeks pregnant presents with severe abdominal pain and vaginal bleeding. Her pregnancy has been complicated by hypertension, which manifested at 24 weeks; it is being treated with methyldopa. Examination reveals a cold and clammy woman in obvious discomfort. Her heart rate is 115 bpm, and blood pressure is 85/55 mm Hg. Question What study would be most helpful in establishing the diagnosis?

Correct answer: Transabdominal ultrasound Explanation The most helpful diagnostic study in patients presenting with antepartum hemorrhage is ultrasound, which may be performed transabdominally or transvaginally. In case of placental abruption, it characteristically reveals displacement of the placenta from the uterine wall by the presence of hematoma. Non-contrast CT is an unnecessarily time-consuming and costly test that exposes the mother and fetus to radiation. A biophysical profile is useful in evaluating fetal status during pregnancy, but it has no role in the acute diagnosis of antepartum hemorrhage. Insertion of an intrauterine pressure catheter is a means to monitor the progress of labor, but it is also not appropriate in this clinical context. Digital cervical examination may be of use at some point, but it is usually not recommended as a first-line intervention until the cause of bleeding is more readily established. Most causes of antepartum hemorrhage relate to abnormalities of the placenta. This patient's symptoms are most consistent with placental abruption, which is responsible for approximately 1/3 of all causes. Placental abruption (also called abruptio placentae) involves the premature separation of the placenta from the uterus. This occurs as a result of defective maternal vessels in the decidual layer of the uterus that bleed and cause separation of the placenta from the uterine wall. Clinical symptoms of placental abruption are vaginal bleeding, abdominal pain, and uterine tenderness occurring in late-term pregnancy. Bleeding may also be occult in the form of decidual hematoma.

Case A 16-year-old sexually active girl is seen for a 2-month history of amenorrhea. She denies having unprotected sex but always relies on her partner to use a condom. She has vomited in the early morning twice in the past week. She has also had vaginal spotting for 3 days, accompanied by cramping lower abdominal pain that became sharp. Onset of menses was at 12 years, with normal, regular periods since then. There is no history of sexually transmitted disease. Physical examination revealed normal vital signs. Slight right and left lower quadrant abdominal tenderness without guarding and rebound was present. The cervix was closed. No blood was seen in the vaginal vault. The uterus was not palpable. Serum β-HCG: 5,200 mIU/ml. Vaginal spotting has increased, and abdominal pain has become more frequent. Repeat examination 3 days after the initial visit is unchanged. The uterus is still not palpable. Repeat serum β-HCG is 6,800 mIU/ml. Transvaginal ultrasound failed to reveal an intrauterine pregnancy or gestational sac. Question What is the most likely diagnosis?

Correct answer: Tubal ectopic pregnancy Explanation The absence of fetal pole or gestational sac in the uterus with elevated serum β-HCG indicates an ectopic pregnancy, of which a tubal ectopic is the most common. The full triad of abdominal/pelvic pain, amenorrhea, and irregular vaginal bleeding is present in only one-half of patients. Abdominal/pelvic pain may be unilateral or bilateral, usually worse on the affected side. Cervical ectopic pregnancies are rare. Serum β-HCG is not adequately elevated to suggest choriocarcinoma. Serum β-HCG is not elevated in pseudocyesis (false pregnancy), a condition in which the female patient truly believes she is pregnant and may exhibit signs and symptoms of true pregnancy such as amenorrhea, hyperemesis, breast swelling and tenderness, weight gain, abdominal tenderness, and even quickening. Transvaginal ultrasound failed to reveal even a fetal pole or gestational sac in the uterus, which should be detected by 36 to 40 days.

Case A 25-year-old woman presents with severe abdominal pain for the last few hours. The patient admits to pelvic pain and vaginal discharge for over 1 month, but she refused to make an appointment with her provider to be evaluated. She last had unprotected intercourse with an ex-boyfriend 3 months ago. Her examination reveals a fever of 103°F and abdominally rigidity, with light palpation that is worst in the left lower quadrant. She does not tolerate a speculum exam. A pelvic ultrasound reveals a 4 x 5 cm left adnexal mass. Urine hCG is negative, WBC is elevated, with a left shift. Question What is the most likely diagnosis?

Correct answer: Tubo-ovarian abscess Explanation This patient present with an acute abdomen that was preceded by pelvic pain and high-risk intercourse. Her history, physical examination, and diagnostic findings are all consistent with a tubo-ovarian abscess. In cases of appendicitis and diverticulitis, you would not expect to find an adnexal mass; the age and presentation of the patient are not consistent with ovarian cancer, and her negative hCG rules out pregnancy, including ectopic pregnancy.

Question A 14-year-old girl had primary amenorrhea, a webbed neck, a broad, shield-like chest with widely spaced nipples, and lack of secondary sexual characteristics. A karyotype of this patient is shown in Figure G3.6. The MOST likely diagnosis is which of the following?

Correct answer: Turner syndrome Explanation In classic Turner syndrome (ovarian dysgenesis), there are only 45 chromosomes, 1 sex chromosome being absent (45, X). This syndrome includes bilateral ovarian agenesis (streak ovaries), infantile genitalia, short stature, short webbed neck, high arched palate, cardiovascular and renal defects (such as coarctation of the aorta and horseshoe kidney), and cubitus valgus (an outward deviation of the extended forearm).

Case A 26-year-old woman presents with an abnormal lump in her right breast. Physical examination demonstrates a well-circumscribed, palpable mass in the upper outer quadrant of the right breast. The patient denies a family history of breast carcinoma or any additional medical problems. Question What would be the best test to order for further evaluation of this palpable abnormality?Case A 26-year-old woman presents with an abnormal lump in her right breast. Physical examination demonstrates a well-circumscribed, palpable mass in the upper outer quadrant of the right breast. The patient denies a family history of breast carcinoma or any additional medical problems.

Correct answer: Ultrasound Explanation In women under 30 with a palpable lump in their breast, the initial imaging modality should be an ultrasound. Ultrasound is an excellent modality in this scenario because it can define the borders of the mass and determine whether the lesion is solid or cystic. If the lesion is solid, then the characteristics of the mass can be better defined before either following the lesion or proceeding to biopsy. Bilateral mammography is not indicated in this patient (who is under 30) at this point. If the patient were over 30 with a palpable breast lump, then a unilateral mammogram and focused ultrasound would be the best tests to order for further evaluation. However, in patients under 30, ultrasound is a better choice for the initial test because most of these "lesions" are fibroadenomas. Additionally, it is desirable to limit radiation exposure in patients under 30, supporting ultrasound as the best choice in this case. Breast MRI exams can be used to evaluate for lobular cancer; occult breast carcinoma; to define the extent of disease in a patient with a positive surgical margin; to evaluate for multifocal, multicentric, or bilateral cancers; and to evaluate for postoperative scar versus tumor recurrence. Breast MRI is not the initial test that should be ordered to evaluate a 26-year-old patient with a palpable breast mass. Galactograms are types of mammograms whereby a breast duct is accessed using a blunt needle to evaluate for unilateral or bilateral bloody nipple discharge, which is most often caused by an intraductal papilloma but could be caused by intraductal papillary carcinoma or invasive ductal carcinoma. Galactograms would not be the initial test in the workup of a palpable mass in this young patient. A stereotactic biopsy is performed to obtain a tissue sample after a suspicious mass or cluster of microcalcifications is found on either a diagnostic or screening mammogram. A stereotactic biopsy would not be performed before an ultrasound is ordered in this patient. If a suspicious mass is seen in ultrasound, then an ultrasound-guided biopsy would be the next best course of action in order to exclude a breast carcinoma.

Case A 32-year-old woman presents for a routine gynecological exam. She has been married for 5 years and plans to start a family with her husband in the near future. Her first menstrual period was at the age of 11. She is in a 28/4 cycle with no irregularities or discomfort. At age 18, she had an induced abortion and has used oral contraceptives since then. In the recto-vaginal exam, you palpate a mass on the left side. An ultrasound confirms a complex cystic tumor 6cm in diameter on the left ovary. There is no free fluid in the pelvis. There is no family history of any malignant tumors. Question What is your therapeutic approach?

Correct answer: Ultrasound examination after the next period Explanation Since this patient is pre-menopausal, with no family history and the tumor is cystic, unilateral, asymptomatic, and there is no ascites, the possibility of it being malignant is very small. Most of these cysts regress within 2 menstruation cycles; therefore, ultrasonic control after the next period is a legitimate approach. If, after 2 cycles, the cyst persists or increases in size, laparoscopy and cystectomy would be indicated. Since the patient expressed the wish for children, it is very important to try leaving the ovary intact. Hormonal treatment with progesterone has not shown any advantage in retrogression of ovarian cysts. Even with no indication of a malignancy, a control period of 6 months is too long and should always include a sonogram. An ovariectomy should only be performed if laparoscopy shows a highly suspicious result of the tumor being malignant

Case Ico-delete Highlights A 30-year-old woman presents with a 2-week history of vaginal discharge. She denies vaginal burning or itching, as well as urinary frequency, hesitancy, and dysuria. The discharge is reported as light yellow in color. She is hesitant to provide much detail, so a physical exam is performed. On physical exam, the patient is in no distress, afebrile, has normal vitals, and a normal exam except for pelvic. On speculum examination, the cervix is inflamed, bright red, and slightly friable with yellow discharge at the os. Vaginal mucosa is pink, moist, and without inflammation. Uterus and ovaries palpate to normal size and are nontender on bimanual exam. Samples are collected for analysis and the results that are available are listed in the table. Test Result Urine hCG Negative Urinalysis (dipstick) Normal, except for 1+ WBC Wet mount/KOH slide Negative for trichamonads, hyphae, and clue cells, >10 WBC per high-powered field Complete blood count Normal Question If the patient provided full details of her history, what would be most consistent with this patient's presentation and likely condition?

Correct answer: Unprotected vaginal intercourse with 1 or more partner(s) Explanation This patient presents with cervicitis, an inflammation of the cervix characterized by mucopurulent cervical discharge and/or endocervical bleeding (induced by swabbing the os). Cervicitis, especially with the finding of increased white blood cells (WBCs) in the vaginal fluid, is associated with sexually transmitted diseases (e.g., Chlamydia trachomatis and Neisseria gonorrhoeae). The increased vaginal leucorrhea may also explain the (mildly) positive leukocyte esterase on the urinalysis dipstick. Cervicitis may occur on its own or as part of pelvic inflammatory disease (PID). Of the choices listed, the most likely history of this patient would be unprotected vaginal intercourse with 1 or more partner(s). It is common to obtain somewhat incomplete sexual histories due to patient embarrassment, distrust, or other factors. Delayed menarche is not associated with cervicitis. The approximate average age for menarche is 12.4 years. For girls much older than this age with primary amenorrhea, genetic, anatomic, and hypothalamic/pituitary causes should be evaluated. In a diabetic patient who is a virgin to sexual intercourse with a complaint of vaginal discharge, the most common consideration is a vaginal yeast infection (candidiasis). Common symptoms include vaginal burning and pruritus; the exam reveals inflamed vaginal mucosa and thick white discharge. The wet mount slide with KOH (potassium hydroxide) reveals the characteristic branching hyphae. In a menopausal woman with vulvovaginal atrophy, common vaginal symptoms include dryness, irritation, and dyspareunia. On physical exam, both the vulva and vagina are noted to be pale and dry. The tissue may also be friable, but the cervix would not be inflamed. A history of recent antibiotic use and vaginal discharge would suggest a possible vaginal yeast infection, with signs and symptoms as noted above.

Case A 23-year-old primiparous woman with an estimated gestational age of 27 weeks presents following a motor vehicle accident. Another car struck the vehicle that she was driving on the passenger side at 40 mph. She was wearing her seat belt. She cannot recall striking her head, but her chest hurts where the belt was, and her chin is scraped from the air bag. Review of her prenatal charts shows that her laboratory work is up to date. Her Rh factor is negative and her initial antibody screen was negative. She has gained 18 pounds during the pregnancy, and her blood pressure and urine dipstick readings have been normal. Her dates were confirmed by an ultrasound done at 12 weeks, and the fundal height measurements have fit with the dates throughout the pregnancy. Her physical exam reveals a blood pressure of 130/85 mm Hg, pulse of 96 BPM, respirations of 16/minute, and temperature of 98 °F (36 °C). There is an irregular abrasion on the chin. All extremities move fully without pain. The fundal height is 28 cm and the abdomen is non-tender. The fetal head is ballotable above the symphysis. Question What would be a major warning sign of minor trauma in this patient?

Correct answer: Vaginal bleeding Explanation Pregnant women who are involved in motor vehicle accidents need to be evaluated for the likelihood of injury that could have adverse effects on the pregnancy. The workup should include a careful physical exam and lab testing to check for maternal or fetomaternal hemorrhage. Rhesus-negative mothers may need Rh° (D) immune globulin. The most important component of the evaluation is fetal heart monitoring of at least 4 hours duration. After this initial monitoring period, if the mother is stable and has no specified warning signs, she may be discharged. Vaginal bleeding may be indicative of placental abruption and doing a speculum examination to evaluate for the presence of amniotic fluid and/or blood is prudent. An urgent ultrasound to assess for abruption is not likely to be helpful (sensitivity is 40% and specificity is 25). Differentiating between normal 'venous lakes' and extravasation, indicating abruption, is difficult. Frequent uterine contractions with or without vaginal bleeding are a good indicator; later, a detailed biophysical profile might be helpful if initial cardiac monitoring is equivocal. In a recent related study, frequent uterine contractions of greater than 4 contractions/hr are associated with poorer outcomes (especially within the initial 4 hours after trauma). Those with fewer than 4 uterine contractions per hour had the same live birth rates and Apgar scores as the control patients. Evidence of a reactive fetal strip would include the presence of short and long-term beat-to-beat variability, and the presence of at least 2 accelerations in fetal heart rate of at least 20 beats/minute within a 20 minute strip. In addition, a fetal heart rate of between 160/minute and 120/minute (above or below which would be classed as a fetal tachycardia and fetal bradycardia, respectively). Other warning signs would include the presence of abdominal tenderness and the presence of fetal decelerations, especially late decelerations.

Case A 31-year-old Caucasian woman presents for genital itching that she has been experiencing for a while. She has treated herself with OTC medication for a yeast infection, which has not helped. Now she notices a lump, which appears to be raised and nodular on exam. Her menstrual cycles have remained unchanged since puberty. She is G0P0 and has been taking oral contraceptives for the past 11 years. Her family and social history are insignificant. Question The presentation is most consistent with what type of cancer diagnosis?

Correct answer: Vulva Explanation Often the first symptom of vulvar cancer is pruritus (itching), which is associated with many other conditions, often causing a delay in diagnosis. Localized areas of color differences, caused by increased pigmentation, hyperkeratosis, wart-like lesions, and shallow ulcerations are indicative of neoplastic changes of the vulva. Biopsies are performed to confirm diagnosis. Treatment can include excision of the lesion or lesions, local chemotherapy, cryosurgery, or laser. Cancer of the vulva can occur at any age. More than 50% of the cases occur in women over the age of 70. In recent years, the occurrence has been increasing in the 20 to 40 year age group. 33 percent of affected women have associated papillomavirus infection of the cervix, vagina, and vulva. Cancer of the vagina is most common in older women. The cause is unknown, but proposed causes are chronic irritation with procidentia (uterine prolapse), wearing pessaries, genital viruses, or metastasis from another site. Cancer of the vagina is increasing in younger women (women less than 50 years of age). The increased incidence in younger women may be related to concomitant or history of HPV. Vaginally, a carcinoma is usually asymptomatic, although the most common symptom is vaginal bleeding. Treatment is removal of the lesion by surgical excision. Local application of chemotherapeutic agents, cryosurgery, electrocautery, or CO2 laser may be used. Lesions of the cervix that are less than full thickness are called dysplasia. The lesions are categorized as mild, moderate, or severe. Full thickness invasion of the cervix by neoplastic cells is carcinoma in situ. Bleeding is the only significant symptom, but it does not necessarily occur early, so that cancers can be far advanced before they are discovered. The Pap smear is used as a screening test for detection of neoplastic changes. If diagnosed early, cervical cancers can be completely removed by cone biopsy or eradicated by laser, cautery, or cryosurgery. Endometrium cancer can occur at any time during adulthood, but is more common after menopause. Most carcinomas develop from a precancerous hyperplasia. Diagnosis is made by examination of tissue from biopsy curettage of the endometrial cavity. Bleeding is the major symptom of endometrial carcinoma and indicates ulceration. The woman often notes a serous discharge before frank bleeding. Pain is a late symptom and indicates widespread disease. Ovarian tumors are grouped into three broad categories: (1) epithelial tumors, (2) gonadal stromal tumors, and (3) germ cell tumors. In regard to most cases, but not in all, tumors are found in postmenopausal women, 55 to 60 years of age. Early diagnosis is difficult because neoplasms of the ovary are usually silent until the tumor has grown large enough to cause symptoms, to be palpable, or to have metastasized. Symptoms are vague; there may be a feeling of pelvic heaviness, urinary frequency, and changes in gastrointestinal functioning with feelings of abdominal fullness. Treatment is surgical, followed by chemotherapy.

Question Your female patient presents with dysmenorrhea that has become more bothersome over the last 3 months. She has difficulty taking ibuprofen (Motrin) due to a stomach ulcer. What is the most appropriate non-pharmacological measure that would relieve her symptoms?

Correct answer: Warm compresses locally Explanation Warm compresses relief cramping associated with dysmenorrhea by relaxing the spasm. Caffeine intake should be decreased to limit symptoms. Engaging in aerobic exercise aids in mitigating symptoms. Wine should be limited; alcohol can also make symptoms worse.

Case A 32-year-old woman is 2 hours status post-cesarean delivery of a twin gestation at 36 weeks. Her pregnancy was uncomplicated. She presented in early labor, which became prolonged despite oxytocin infusion. A cesarean section was performed when her labor became non-progressive. In the recovery area, she notes nausea and lightheadedness. On exam, her heart rate is 133 beats per minute and blood pressure is 76/42 mm Hg. Significant vaginal bleeding is noted; abdominal palpation reveals a soft uterus. Question What intervention would be most appropriate?

orrect answer: Bimanual uterine massage Explanation The most likely cause of bleeding in this patient is uterine atony. Uterine atony occurs when the uterine myometrium fails to contract following delivery. Contractions of the uterine muscles after delivery normally tamponade bleeding from uterine arterioles. Absence of this response causes continued bleeding, which usually becomes evident soon after delivery. Bimanual uterine palpation revealing a soft "boggy" uterus confirms the diagnosis. Initial treatment involves bimanual uterine massage, which helps promote uterine contractions. Uterotonic agents are also administered. The first-line of therapy is intravenous oxytocin. Second-line therapies, including ergot alkaloid derivates and prostaglandins (e.g., Hemabate), are used when oxytocin therapy is unsuccessful. When these measures are unsuccessful, surgical interventions are required, the most common of which is bilateral uterine artery ligation. Uterine atony can also occur as a result of retained products of conception; these products inhibit uterine contraction. In this case, manual or surgical extraction is necessary. Intravenous magnesium is a uterine relaxant used to inhibit uterine contractions in settings of premature labor. It has the opposite effect to that desired to treat uterine atony. Transfusion with fresh frozen plasma may be indicated as a temporizing measure in the setting of disseminated intravascular coagulation accompanied by severe bleeding or massive uncontrolled hemorrhage from uterine rupture, neither of which is suspected in this clinical scenario. Transvaginal ultrasound is rarely needed to diagnose postpartum hemorrhage, and it is not the most appropriate next step in light of the patient's condition.

Case Your patient is a 16-year-old girl who weighs 300 lb and is 5 ft 2 in tall. She has severe facial acne vulgaris, hirsutism, and irregular periods. Her pediatrician has sent her to your 24-hour endocrinology clinical research station because she thinks that the patient may have polycystic ovary syndrome. Question What laboratory finding would support the diagnosis of polycystic ovary syndrome?

orrect answer: Elevated LH:FSH ratio Explanation The correct answer is elevated LH:FSH ratio. Polycystic ovary syndrome is a condition marked by hyperandrogenism and anovulation. Patients often present with disordered menstrual cycles as well as hirsutism and acne, which are related to elevated testosterone levels. About half of patients are overweight; this is related to insulin resistance. Typical lab findings of polycystic ovarian syndrome include increased androgen levels, including testosterone and DHEA. The LH:FSH ratio is typically elevated (>2), not reduced. Insulin levels are also high, related to insulin resistance.

Case A 28-year-old woman presents with a 1-hour history of sudden onset extreme abdominal pain. She reports nausea and vomited once in the ambulance. Her last period was 26 days ago in a 29/5 cycle with no irregularities. Her menarche started at 13; she has one sexual partner, with whom she has painless intercourse about twice a week. Her last gynecological exam was about 2 years ago; no abnormalities were reported. About a month ago, she had a feeling of fullness inside her pelvis for some time. Upon examination, there is tenderness, especially on the right side of the lower abdomen; no ascites are palpable. Percussion does not reveal any pleural effusion; there are no enlarged lymph nodes in the cervical, supraclavicular, axillary, or inguinal areas. There are no tumors palpable in either breast. Temperature is 36.8°C axillary, 37.3°C rectally. The bimanual exam reveals normal sized uterus and a plum-sized tumor in the right lower quadrant. β-HCG: <5IE/L, CA-125: 20IE/L, WBC: 6.0 x 103/mm3. Question What is the most likely diagnosis?

orrect answer: Twisted ovarian cyst Explanation The description of the feeling of fullness inside the pelvis is a common symptom of ovarian cysts. It is the result of the growing tumor putting pressure on the neighboring organs. The sudden onset of pain with nausea and vomiting suggests torsion of the cyst and needs immediate attention. Since the patient does not have any dysmenorrhea and pain during intercourse, endometriosis is unlikely. β-HCG: <5IE/L lets you rule out an ectopic pregnancy that could cause the pain. Normal WBC and normal temperature point towards no inflammatory process, including acute salpingitis and appendicitis. Other symptoms of acute appendicitis may include local tenderness at McBurney's point (junction of the middle and outer thirds of the line joining the umbilicus to the anterior superior spine) and Rovsing's sign (pain felt in the right lower quadrant). In addition, psoas sign (increase in pain from passive extension of the right hip joint that stretches the psoas muscle), adductor pain (produced by passive internal rotation of the flexed thigh), and increased bowel movement may occur. In acute salpingitis, the pain usually does not begin suddenly but becomes progressively more severe. There is increased discomfort with cervical motion and vaginal discharge. Since mammary carcinoma can metastasize into the ovaries, it is important to check the breasts for enlarged lymph nodes and ascites as well as pleural effusions.


Set pelajaran terkait

Week 1 - Chapter 3: Health, Wellness, and Health Disparities

View Set

Civics Midterm Exam study Guide Grade 7

View Set

Business Result Pre-Intermediate - Unit 2

View Set

Nursing Concepts PrepU Practice Questions - Ch. 19

View Set

CHAPTER 13-15/16 US HISTORY TO 1877 MULTIPLE CHOICE

View Set

Assessment and Management of Patients with Diabetes PrepU

View Set

Consumer Behavior Test 3 (13-17)

View Set